Hechos Geométricos en el Triángulo (2014)

Angel Montesdeoca

(Última actualización: )

Hechos Geométricos en el Triángulo de:
(2013) (2015) (2016) (2017) (2018) (2019) (2020) (2021) (2022) (2023) (2024)

Cómo es el enlace a un Hecho Geométrico correspondiente a un día concreto:
http://amontes.webs.ull.es/otrashtm/HGt2014.htm#HGddmmaa
EJEMPLO: Martes, 13 de mayo del 2014
http://amontes.webs.ull.es/otrashtm/HGT2014.htm#HG130514

  • martes 23 de diciembre del 2014

    Triángulos pedales ortológicos. Cúbica K389. Cuártica Q098


      If P and Q are two points collinear with the circumcenter O of ABC, then the pedal triangles of P and Q wrt ABC are orthologic.
    (Luis González. Art of Problem Solving)

      If P and Q are points with the midpoint O, the circumcenter, then their pedal triangles are orthologic and their orthology centers share the same midpoint O. (Luis González. Art of Problem Solving)

      Si ABC es triángulo, se denota por O el circuncentro, por PaPbPc y por QaQbQc los triángulos pedales de dos puntos P y Q, respectivamente.

      Dado un punto P, el lugar geométrico de los puntos Q tal que los triángulos pedales de P y Q son ortológicos es la recta OP.


      Si P(p:q:r), las coordenadas baricéntricas de Pa son:

    (0 : (b^2-c^2)p + a^2(p+2q) : (-b^2+c^2)p + a^2(p+2r)).

      La perpendicular por Pa al lado XbXc, si Q(u:v:w), es:

    (b^2-c^2)p(c^2v+b^2w)+a^2(c^2(p+2q)v-b^2(p+2r)w)x -
    ((-b^2+c^2)p+a^2(p+2r))(c^2v+b^2w)y +
    ((b^2-c^2)p+a^2(p+2q))(c^2v+b^2w)z = 0.

    Y análogamente se obtienen las perpendiculares por Pb al lado XcXa y por Pc al lado XaXb.
      Estas tres perpendiculares son concurrentes (PaPbPc y QaQbQc son ortológicos) si y sólo si

    a^2(b^2r-c^2q) - (b^2-c^2)(c^2q+b^2r)u+... =0,

    es decir, el punto Q está en la recta OP.


      Sean P un punto con triángulo pedal PaPbPc y Q un punto variable sobre la recta OP, con triángulo pedal QaQbQc.

     • El lugar geométrico del centro de ortología X de QaQbQc respecto a PaPbPc es una recta (L) que pasa por el ortocentro HP de PaPbPc.
      La recta (L) pasa por O si y solo si P está en la cúbica K389.
      La recta (L) pasa por P si y solo si P está en la cuártica Q098.

     • El lugar geométrico del centro de ortología Y de PaPbPc respecto a QaQbQc es la hipérbola equilátera (H) circunscrita a PaPbPc que pasa por P.

      Si P(p:q:r), la ecuación de la recta (L), lugar geométrico del centro de ortología X de QaQbQc respecto a PaPbPc, es

    (L):    (b^2SBr-c^2SCq)x ⁄ (c^2q+b^2r)+... = 0.


      La recta (L) pasa por el ortocentro HP de PaPbPc:

    HP (a^2(c^2q+b^2r) (SA(c^2p q+b^2p r+a^2q r) - b^2c^2p(p+q+r)): ... : ... )


      La cúbica K389 ("Kosnita nodal cubic") es el lugar geométrico de los puntos P tales que la recta (L) pasa por el circuncentro.

      Si Q=P' es el simétrico de P respecto O, X e Y son simétricos respecto a O e Y es el segundo punto de interseccion D de (L) y (H).
    ( Mostrar/Ocultar figura )
      TriangulosPedalesOrtologicosK389.png
    Descargar fichero GeoGebra


      La cuártica Q098 ("An inversible circular quartic, the isogonal transform of Q044") es el lugar geométrico de los puntos P tales que la recta (L) pasa por P.
    ( Mostrar/Ocultar figura )
      TriangulosPedalesOrtologicosQ098.png
    Descargar fichero GeoGebra


    C A S O   G E N E R A L

      El lugar geométrico de los puntos P tales que la recta (L) pasa por el punto P0, que divide al segmento OP en la razón OP0 : P0P = m : n, es la cuártica

    m a^2b^2c^2 Q098 + n R^2 (x+y+z) K389 = 0.

    R es el radio de la circunferencia circunscrita a ABC. Todas las cuárticas de este haz están circunscritas a ABC, tiene al circuncentro como punto singular, pasan por los vértices del triángulo tangencial y por los puntos en el infinito de la hipérbola de Jerabek.
    ( Mostrar/Ocultar figura )
      TriangulosPedalesOrtologicosmn.png
    Descargar fichero GeoGebra


    CASO m:n=-1:1
      El lugar geométrico de los puntos P tales que las rectas OP y (L) son paralelas es una cuártica con punto triple en el circuncentro.
    Pasa además por los centros X186, X1319, X1691, X2574, X2575 (estos últimos en el infinito, conjugados isogonales de los puntos en que la recta de Euler corta a la circunferencia circunscrita).
    ( Mostrar/Ocultar figura )
      TriangulosPedalesOrtologicosQnnn.png
    Descargar fichero GeoGebra


    CASO m:n=-1:2
      El lugar geométrico de los puntos P tales que la recta (L) pasa por el simétrico de P respecto al circuncentro es una cuártica, que pasa por los centros X3, X1498, X2574, X2575, X3532.
    ( Mostrar/Ocultar figura )
      TriangulosPedalesOrtologicosQnnnb.png
    Descargar fichero GeoGebra


  • viernes 19 de diciembre del 2014

    Cúbicas circulares circunscritas con punto nodal en el ortocentro


      Si ABC es triángulo, sean DEF su triángulo medial y A'B'C' su triángulo antimedial. Dado un punto X, sean X' su anticomplemento y Xa , Xb, Xc las reflexiones de X' en D, E, F, respectivamente,

      • Las circunferencias circunscritas a los triángulos BCXa, CAXb y ABXc son concurrentes en un punto Y, sobre la circunferencia circunscrita al triángulo XaXbXc.
    ( Mostrar/Ocultar figura )
      CircularNodalCubic.png

      Como los triángulos ABC y XaXbXc son simétricos respecto a X, ellos son ciclológicos.
      Un centro ciclológico es Y, el cuarto punto de intersección de la circunferencia circunscrita a XaXbXc con la cónica circunscrita a ABC con centro en X.
      El otro centro ciclológico, Z, es el cuarto punto de intersección de la circunferencia circunscrita a ABC con la citada cónica.

      Si X=(u:v:w), las coordenadas baricéntricas de Y son:

    F(X) = Y = ((u-v-w) /(2 a^2 v w+(b^2 w+c^2 v) (u-v-w)) : ... : ...)

    Para X el circuncentro, F(X) no está definido. Si X está en el infinito F(X)=X

      • Si X recorre una recta d: px+qy+rz=0, que no pasa por el circuncentro, F(X) queda sobre en una quíntica circunscrita con puntos dobles en los vértices de ABC y en el ortocentro, con una asíntota paralela a d.
    ( Mostrar/Ocultar figura )
      CircularNodalCubic0.png


    ( Hyacinthos #9381, Jean-Pierre Ehrmann)
      Los puntos Xa , Xb, Xc son las reflexiones de A, B, C en X, respectivamente. Al punto Y, de concurrencia de las circunferencias circunscritas a los triángulos BCXa, CAXb y ABXc, se le denomina "syngonal conjugate" of X.
      Así, Y=F(X) es el antigonal (conjugado isogonal del inverso en la circunferencia circunscrita del conjugado isogonal) del anticomplento de X.

      • La imagen de una recta d que pasa por el circuncentro, mediante la aplicacion XF(X), es una cúbica circular Kd circunscrita a ABC con punto nodal el ortocentro.
    ( Mostrar/Ocultar figura )
      CircularNodalCubicOP.png

      Si la recta d corta a BC en el punto que divide al segmento BC en la razón m:n, el lugar geométrico de Y=F(X) cuando X se mueve sobre d es la cúbica K(d):

    2x(SB((a^2-b^2-a c)(a^2-b^2+a c)m+ b^2c^2n) y^2+SC(- b^2 c^2m-(a^2-a b-c^2) (a^2+a b-c^2)n)z^2)+
    2y(SC(a^2 c^2m-a^2 (a^2-b^2)n) z^2 +SA(-(a^2-b^2-b c)(a^2-b^2+b c)m -c^2 (b^2-c^2)n)x^2)+
    2z(SA(-b^2 (b^2-c^2)m+ (a^2-b c-c^2) (a^2+b c-c^2)n)x^2 +SB(a^2(a^2-c^2)m- a^2 b^2 n)y^2 )+
    ((a^2-b^2)(a^4+b^4-c^4)m-(a^2-c^2)(a^4-b^4+c^4)n) x y z =0,

    O bien, en función de las coordenadas (p:q:r), p+q+r=0, de su punto del infinito:

    2 x (SB (b^2 c^2 (a^2 q SA - b^2 p SB) + (a^2 - b^2 - a c) (a^2 - b^2 + a c) (a^2 r SA - c^2 p SC)) y^2 + SC (-(a^2 - a b - c^2) (a^2 + a b - c^2) (a^2 q SA - b^2 p SB) - b^2 c^2 (a^2 r SA - c^2 p SC)) z^2) +
    2 y (SA (-c^2 (b^2 - c^2) (a^2 q SA - b^2 p SB) + (a^2 - b^2 + b c) (-a^2 + b^2 + b c) (a^2 r SA - c^2 p SC)) x^2 + SC (-a^2 (a^2 - b^2) (a^2 q SA - b^2 p SB) + a^2 c^2 (a^2 r SA - c^2 p SC)) z^2) +
    2 z(SA ((a^2 - b c - c^2) (a^2 + b c - c^2) (a^2 q SA - b^2 p SB) - b^2 (b^2 - c^2) (a^2 r SA - c^2 p SC)) x^2 + SB (-a^2 b^2 (a^2 q SA - b^2 p SB) + a^2 (a^2 - c^2) (a^2 r SA - c^2 p SC)) y^2) +
    (-(a^2 - c^2) (a^4 - b^4 + c^4) (a^2 q SA - b^2 p SB) + (a^2 - b^2) (a^4 + b^4 - c^4) (a^2 r SA - c^2 p SC)) x y z = 0,
    donde SA=(b^2+c^2-a^2)/2,...

     - Si la recta d corta a la circunferencia de Euler en X1 y X2, el punto F(X1)=F(X2)=H es crunodal.
     - Si la recta d es tangente a la circunferencia de Euler en X, F(X)=H es tacnodal (de retroceso de primera especie).
     - Si la recta d no corta a la circunferencia de Euler, H es acnodal (aislado).

    Cúbicas K(d) particulares:

     • Si d es la recta de Euler, K(d) = K025 Ehrmann strophoid
    ( Mostrar/Ocultar figura )
      CircularNodalCubicK025.png

     • Si d es la recta de X3X523, K(d) = K186 Iona cubic
    ( Mostrar/Ocultar figura )
      CircularNodalCubicK186.png

     • Si d es la recta paralela por X3 a X4X6, K(d) = K288 Gigha cubic
    ( Mostrar/Ocultar figura )
      CircularNodalCubicK288.png

     • Si d es el eje de Brocard, K(d) = K289 Canna cubic
    ( Mostrar/Ocultar figura )
      CircularNodalCubicK289.png

     • Si d es la recta X3X1, la ecuación baricéntrica de K(d) es:

    c (a^2 - b^2 - c^2) (a^3 - a^2 b - a b^2 + b^3 + a b c - b c^2) x^2 y + c (a^2 - b^2 + c^2) (a^3 - a^2 b - a b^2 + b^3 + a b c - a c^2) x y^2 - b (a^2 - b^2 - c^2) (a^3 - a^2 c + a b c - b^2 c - a c^2 + c^3) x^2 z - (a - b) (a - c) (b - c) (a^3 + a^2 b + a b^2 + b^3 + a^2 c + 2 a b c + b^2 c + a c^2 + b c^2 + c^3) x y z + a (a^2 - b^2 + c^2) (-b^3 + a^2 c - a b c + b^2 c + b c^2 - c^3) y^2 z - b (a^2 + b^2 - c^2) (a^3 - a b^2 - a^2 c + a b c - a c^2 + c^3) x z^2 - a (a^2 + b^2 - c^2) (a^2 b - b^3 - a b c + b^2 c + b c^2 - c^3) y z^2=0.
    ( Mostrar/Ocultar figura )
      CircularNodalCubicOI.png



  • domingo 14 de diciembre del 2014

    Quártica Q23 y séptica asociada


    ( Anopolis #2155, Antreas P. Hatzipolakis)

      Sea ABC un triángulo, O su circuncentro y P un punto, el lugar geométrico de P tal que el ortocentro de su triángulo pedal queda en la recta OP es la cuártica Q023, del catálogo de Bernard Gibert).

      Si (x:y:z) son las coordenadas baricéntricas de P el ortocentro del triángulo pedal A'B'C' es:

    H' (a^2(c^2y+b^2z) (yza^4 + (b^2z(x-y)+c^2y(x-z))a^2 - x(b^4z+c^4y-b^2c^2(2x+y+z))) : ... : ...)

      Los puntos P que están alineados con H' y O (a^2(b^2+c^2-a^2):...:...) satisfacen a la ecuación de la cuártica Q023.
    ( Mostrar/Ocultar figura )
      Anopolis2155.png



      El lugar geométrico del ortocentro del triángulo pedal de un punto P que varía en la cuártica Q023 es una séptica (ecuación obtenida por Bernard Gibert), con el circuncentro como punto quíntuple, puntos dobles en los puntos medios de los lados de ABC y tangentes en los vértices las recta OA, OB y OC.

  • domingo 30 de noviembre del 2014

    Cúbica de Bataille como un caso particular

      Sean ABC un triángulo y DEF el triángulo ceviano de un punto U. Tomemos los puntos D1=BC∩EF, E1=CA∩FD y F1=AB∩DE y sus reflexiones D2, E2, F2 respecto a los puntos medios D0, E0, F0 de los lados de DEF.
      Sea Ka el punto de intersección de la paralela por D1 a AU con la recta AD0.

    NOTA:
      Las rectas KaD1 y KaD2 son las asíntotas de la hipérbola Φa tangente en E, F a AC, AB y con una asíntota paralela a AU. Así, lo expuesto da un procedimiento para construir la hipérbola dado la dirección de una asíntota y dos tangentes de dos de sus puntos. Ver otros en It_Pt_P, It_Pt_P2, (3P_12T)_2, este último de Paris Pamfilos, omitido en A Gallery of Conics by Five Elements. Forum Geometricorum, Volume 14 (2014) 295–348.

    ( Mostrar/Ocultar figura )
      BatailleCubicsdadbdc.png

      Se denota por da la recta paralela a KaD2 por A. Similarmente, se definen las rectas db y dc.

      Las tres rectas da, db y dc son concurrentes.

      Si U(u:v:w), las coordenadas baricéntricas del punto V, de intersección de las tres rectas da, db, dc, son:

    F(U) = V = (u(v+w)^2 : v(u+w)^2 : w(u+v)^2)


      A un punto U y a su conjugado isotómico U les corresponde el mismo punto V.

      Cuando U recorre la cónica circunscrita de perspector P(p:q:r), pyz+qzx+rxy=0, o la recta px+qy+rz=0, V=F(U) describe la cúbica:

    ( Mostrar/Ocultar figura )
      BatailleCubics.png

      El punto singular de esta cúbica es:

    D = (p/(q + r - p) : q/(r + p - q) : r/(p + q - r)).

    Es el punto que correponde, mediante la aplicación F, a los puntos de intersección de la cónica circunscrita de perspector P y la tripolar de P.

      Los puntos de intersección de las tripolares de P(p:q:r) y del punto (1/(-p+q+r)^2:1/(p-q+r)^2:1/(p+q-r)^2) con los lados de ABC, están en la cúbica.

    CASO PARTICULAR

      Si U(u:v:w) se mueve sobre la recta del infinito, x+y+z=0, o sobre la elipse circunscrita de Steiner, yz+zx+xy=0, V (u^3:v^3:w^3) o V (1/u^3:1/v^3:1/w^3) describe la cúbica de Bataille (K656):
    x^3 + 3 x^2 y + 3 x y^2 + y^3 + 3 x^2 z - 21 x y z + 3 y^2 z + 3 x z^2 + 3 y z^2 + z^3 =0
    ó (x + y + z)^3 = 27 x y z.

    El punto singular de K656 es el baricentro (aislado) y pasa por X3081=F(X30)=F(X1494).

  • sábado 29 de noviembre del 2014

    Cuadrados inscritos en un triángulo y cúbicas asociadas

    a Lolilla, por su "cumple"



      Sean ABC un triángulo, P un punto y DEF su triángulo ceviano, Se denota por Ai el centro del cuadrado inscrito en ABC con un lado sobre BC y los otros dos vértices sobre los segmentos AB y AC. Ae el centro del cuadrado inscrito en ABC con un lado sobre BC y los otros dos vértices fuera de los segmentos AB y AC. Similarmente, se define los centros Bi, Be, Ci, Ce.
    ( Mostrar/Ocultar figura )
      CentrosCuadradosInscritos.png

      En coordenadas baricéntricas, Ai=(a^2:SC+S:SB+S) y Ae=(a^2:SC-S:SB-S).

      Los triángulos AiBiCi y AeBeCe son perspectivos con centro de perspectividad el simediano.

    ( Mostrar/Ocultar figura )
      CuadradosInscritosABC.png

      El lugar geométrico de los puntos P tales que los triángulos DEF y AiBiCi son perspectivos es la cúbica K070-b, "Shoemaker's (inner) cubic" = pK(X4, X1585). El centro de perspectividad Qi queda sobre la cúbica isogonal de pivote X3068 (K424a, del catálogo de cúbicas de Bernard Gibert).

    ( Mostrar/Ocultar figura )
      CuadradosInscritosCevianoK070b.png
    Descargar fichero GeoGebra

      Si P=(p:q:r) la ecuación baricéntrica de la recta DAi es (q(S+SB)-r(S+SC))x + a^2ry - a^2qz = 0.
      Las rectas DAi, EBi, FCi son concurrentes si y solo si las coordenadas de P satisfacen a la ecuación: Dividiendo por S SASBSC: que es la ecuación de la cúbica pivotal pK(X4,X1585).

      La primera coordenada de Qi es:

    a^4(-p^2+p(q+r)+3q r) + 2a^2(p(c^2(p+q)+b^2(p+r))+2(p-q)(p-r)S) - b^4(p+q)(p-r) - c^4(p-q)(p+r) + 2b^2c^2(p^2-qr) - 4pS(c^2(p-q)+b^2(p-r))-4(p-q)(p-r)S^2

    Este punto está, cuando P=(p:q:r) recorre K070-b, en la cúbica K424a que pasa por los centros X(1), X(2), X(6), X(371), X(485), X(493), X(3068), por los excentros Ia, Ib, Ic y por los vértices de AiBiCi.


      Cambiando en las expresiones algebraicas anteriores S (doble del área de ABC) por -S, se obtiene:

      El lugar geométrico de los puntos P tales que los triángulo DEF y AeBeCe son perspectivos es la cúbica K070-a, "Shoemaker's (outer) cubic" = pK(X4, X1586). El centro de perspectividad Qe queda sobre la cúbica isogonal de pivote X3069 (K424b).

    ( Mostrar/Ocultar figura )
      CuadradosInscritosCevianoK070a.png
    Descargar fichero GeoGebra


    ∗  ∗  ∗  ∗  ∗  ∗  ∗  ∗  ∗  ∗ 



      Planteamos el estudio anterior sustituyendo el triángulo ceviano de P por su triángulo anticeviano.


      El lugar geométrico de los puntos P tales que su triángulo anticeviano DEF es perspectivo con AiBiCi y con AeBeCe, es la cúbica K006, "Orthocubic" = pK(X6, X4). Los centros de perspectividad Qi y Qe quedan, respectivamente, sobre la cúbicas isogonales K424a y K424b.

    ( Mostrar/Ocultar figura )
      CuadradosInscritosAntiCevianoK006.png
    Descargar fichero GeoGebra

    Los centros de perspectividad Qi y Qe están alineados con el baricentro y están sobre la hipérbola rectangular que pasa por P y circunscrita a su triángulo anticeviano DEF y al triángulo excentral IaIbIc.

  • lunes 24 de noviembre del 2014

    Cúbica asociada a una recta. Cúbica de Allardice

      Sean ABC un triángulo, U un punto y DEF su triángulo ceviano. Los puntos D1=BC∩EF, E1=CA∩FD y F1=AB∩DE están en el eje d de perspectividad ("perspectrix") de los triángulos ABC y DEF. Denotamos por d‍′ la recta que pasa por las reflexiones D2, E2, F2 de D1, E1, F1 respecto a los puntos medios de los lados de DEF
      Si U=(u:v:w), las coordenadas baricéntricas de V=d∩d‍′ son:

    V = (u(v-w)(2u+v+w) : v(w-v)(u+2v+w) : w(u-v)(u+v+2w)).

      El lugar geométrico del punto de intersección V de las la rectas d y d‍′, cuando U se mueve sobre una recta p : px+qy+rz=0 es la cúbica:

    ( Mostrar/Ocultar figura )
      HG241114.png
    Descargar fichero GeoGebra


      El punto singular de esta cúbica es S=(3p-2(q+r) : 3q-2(r+p) :3r-2(p+q)).
      Si P es el punto de coordenadas (p:q:r), es decir, el conjugado isotómico del polo trilineal de la recta p : px+qy+rz=0, el punto singular S es la imagen de P mediante la homotecia h(G,-5), de centro el baricentro y razón -5.

      La correspondencia

    p(p:q:r)  ↦  S(3p-2(q+r) : 3q-2(r+p) :3r-2(p+q))

    es una correlación. Si la recta p es tangente a la elipse x^2+y^2+z^2+4xy+4xz+4yz=0, homotética de la elipse inscrita de Steiner en la homotecia h(G,Sqrt[10]), el punto singular S es el punto de tangencia.
    ( Mostrar/Ocultar figura )
      HG241114T.png
    Descargar fichero GeoGebra


    CASOS PARTICULARES

     • Cuando el punto U está en la recta del infinito p : x+y+z=0, el punto V es el baricentro de las trazas (D1, E1, F1) de la polar trilineal de U en los lados de ABC. Así. el lugar geométrico de V es la cúbica de Allardice (K219, del catálogo de cúbicas de Bernard Gibert)

     • Cuando el punto U está en la recta de Euler el punto V describe la parábola, cuyo eje tiene la dirección de punto X525, de ecuación:

    (a^4+a^2(-3b^2+2c^2)+2b^4+b^2c^2-3c^4) (a^4+a^2(2b^2-3c^2)-3b^4+b^2c^2+2c^4)x^2+ (-12a^8 + 12a^6(b^2+c^2) + a^4(11b^4-34b^2c^2+11c^4) - 10a^2(b^2-c^2)^2(b^2+c^2) - (b^2-c^2)^2(b^4-10b^2c^2+c^4) )yz+.... =0
    ( Mostrar/Ocultar figura )
      HG241114Euler.png


     • Cuando el punto U está en la recta con coeficientes las coordenadas del ortocentro, p : x/SA+y/SB+z/SC=0, el punto V describe una cúbica, con punto doble X3529 y pasa por X1650 (ambos en la recta de Euler).
    ( Mostrar/Ocultar figura )
      HG241114X4.png


  • sábado 15 de noviembre del 2014

    Séptica que pasa por los "equilateral cevian points"



    ( Anopolis #2003, Antreas P. Hatzipolakis)

    Equilateral cevian points, X(370) and related curves (Bernard Gibert)
      A point P is an equilateral cevian point if the cevian triangle of P is equilateral (TCCT, p.267). The corresponding center of the triangle is denoted by Q. There are 6 such points P which lie on the Neuberg cubic: 4 are always real, one of them being X(370), the only one inside ABC. Each line through P and the corresponding Q passes through O.


      Sean ABC un triángulo, O el circuncentro, P un punto y A'B'C' el triángulo ceviano de P.
      El lugar geométrico de P, tal que el centro N' de la circunferencia de los nueve puntos de A'B'C' esté sobre la recta OP, es una séptica que pasa por los vértices de los triángulos ABC (triples), medial y excentral, y por X1, X2, X3, X370 (y los otros cinco -tres siempre reales- "equilateral cevian points"), X1138.


      Si P(u:v:w), la primera coordenada baricéntrica de N' es:

    a^2 c^2 u^3 v^3 - b^2 c^2 u^3 v^3 + c^4 u^3 v^3 - 3 a^4 u^3 v^2 w +
    6 a^2 b^2 u^3 v^2 w - 3 b^4 u^3 v^2 w + 5 a^2 c^2 u^3 v^2 w +
    5 b^2 c^2 u^3 v^2 w - 2 c^4 u^3 v^2 w - a^4 u^2 v^3 w +
    2 a^2 b^2 u^2 v^3 w - b^4 u^2 v^3 w + 4 a^2 c^2 u^2 v^3 w +
    2 b^2 c^2 u^2 v^3 w - c^4 u^2 v^3 w - 3 a^4 u^3 v w^2 +
    5 a^2 b^2 u^3 v w^2 - 2 b^4 u^3 v w^2 + 6 a^2 c^2 u^3 v w^2 +
    5 b^2 c^2 u^3 v w^2 - 3 c^4 u^3 v w^2 - 4 a^4 u^2 v^2 w^2 +
    6 a^2 b^2 u^2 v^2 w^2 - 2 b^4 u^2 v^2 w^2 + 6 a^2 c^2 u^2 v^2 w^2 +
    4 b^2 c^2 u^2 v^2 w^2 - 2 c^4 u^2 v^2 w^2 - a^4 u v^3 w^2 +
    a^2 b^2 u v^3 w^2 - a^2 c^2 u v^3 w^2 + a^2 b^2 u^3 w^3 +
    b^4 u^3 w^3 - b^2 c^2 u^3 w^3 - a^4 u^2 v w^3 + 4 a^2 b^2 u^2 v w^3 -
    b^4 u^2 v w^3 + 2 a^2 c^2 u^2 v w^3 + 2 b^2 c^2 u^2 v w^3 -
    c^4 u^2 v w^3 - a^4 u v^2 w^3 - a^2 b^2 u v^2 w^3 +
    a^2 c^2 u v^2 w^3 - 2 a^4 v^3 w^3.


      La condicion necesaria y suficiente para que P, el circuncentro y N' estén alineados, es que las coordenadas de P satisfagan a la ecuación:

    -c^4 x^4 y^3 + c^4 x^3 y^4 - a^2 c^2 x^4 y^2 z - 2 b^2 c^2 x^4 y^2 z + c^4 x^4 y^2 z - 2 a^2 c^2 x^3 y^3 z + 2 b^2 c^2 x^3 y^3 z + 2 a^2 c^2 x^2 y^4 z + b^2 c^2 x^2 y^4 z - c^4 x^2 y^4 z + a^2 b^2 x^4 y z^2 - b^4 x^4 y z^2 + 2 b^2 c^2 x^4 y z^2 - a^2 b^2 x^3 y^2 z^2 + b^4 x^3 y^2 z^2 + a^2 c^2 x^3 y^2 z^2 - c^4 x^3 y^2 z^2 - a^4 x^2 y^3 z^2 + a^2 b^2 x^2 y^3 z^2 - b^2 c^2 x^2 y^3 z^2 + c^4 x^2 y^3 z^2 + a^4 x y^4 z^2 - a^2 b^2 x y^4 z^2 - 2 a^2 c^2 x y^4 z^2 + b^4 x^4 z^3 + 2 a^2 b^2 x^3 y z^3 - 2 b^2 c^2 x^3 y z^3 + a^4 x^2 y^2 z^3 - b^4 x^2 y^2 z^3 - a^2 c^2 x^2 y^2 z^3 + b^2 c^2 x^2 y^2 z^3 - 2 a^2 b^2 x y^3 z^3 + 2 a^2 c^2 x y^3 z^3 - a^4 y^4 z^3 - b^4 x^3 z^4 - 2 a^2 b^2 x^2 y z^4 + b^4 x^2 y z^4 - b^2 c^2 x^2 y z^4 - a^4 x y^2 z^4 + 2 a^2 b^2 x y^2 z^4 + a^2 c^2 x y^2 z^4 + a^4 y^3 z^4=0.

      O bien,

    𝔖abc xyz y z ((b^2 - c^2) (-a^2 + b^2 + c^2) x^3 y z - 2 a^2 (b^2 - c^2) x y^2 z^2 - a^4 y^2 (y - z) z^2 - x^4 (c^2 (a^2 + 2 b^2 - c^2) y - b^2 (a^2 - b^2 + 2 c^2) z) = 0.



      Esta séptica ha sido incorporado a CCT Q105.
    ( Mostrar/Ocultar figura )
      Anopolis2001.png
    Descargar fichero GeoGebra


      Los 21 puntos comunes de esta séptica y la cúbica de Neuberg (K001, del catálogo de cúbicas de Bernard Gibert) son: los vértices de los triángulos ABC (triples) y excentral, X1, X3, X370 (y los otros cinco -tres siempre reales- "equilateral cevian points") y X1138.

  • martes 11 de noviembre del 2014

    Una nónica con puntos cuádruples en los vértices del triángulo de referencia



      Dados un triángulo ABC, un punto P y su triángulo ceviano DEF, sean Da, Eb y Fc los puntos medios de los segmentos EF, BE y CF.
      La circunferencia circunscrita a DaEbFc vuelve a cortar a la recta EF en el punto D'. Se denota por A' la reflexión de D' en la recta EbFc. El punto A' está en la circunferencia de los nueve puntos de ABC.
    ( Mostrar/Ocultar figura )
      Q066_P3p.png

      Procediendo cíclicamente, de definen los puntos B' y C'.

      Los triángulos ABC y A'B'C' son ortológicos si sólo si el punto P está en una nónica con puntos cuádruples en los vértices de ABC, puntos dobles los vértices del triángulo antimedial y que pasa por el baricentro, ortocentro y punto de Gergonne.

    ( Mostrar/Ocultar figura )
      HG111114Math.png


      Si (u:v:w) son las cordenadas baricéntricas de P, la ecuación de la circunferencia DaEbFc es:
    HG111114circDaEbFc.png

      La circunferencia circunscrita a DaEbFc vuelve a cortar a la recta EF en el punto D':

    D' = (u(c^2uv(v-w)+ w(b^2u(-v+w)+a^2v(2u+v+w))) : v(c^2uv(u+w)+ w(-b^2u(u+2v-w)+a^2v(u+w))) : -w(-(u+v)(b^2u+ a^2v)w+c^2uv(u-v+2w))).

      La reflexión de D' en la recta EbFc es:

    A' = (-u(v-w)(-b^2(u+v)w+c^2v(u+w)) : -v(u+ w)(c^2u(v-w)+a^2(u+v)w) : -(u+v)w(a^2v(u+w)+ b^2u(-v+w)).


      Los puntos para los cuales las perpendiculares por los vértices de A'B'C' a los lados de ABC son concurrentes, están en una nónica que, además de las propiedades enunciadas arriba, pasa por los pies de las cevianas de X264 (conjugado isotómico del circuncentro). Estos son los únicos puntos de intersección de la nónica con los lados de ABC, a parte de los vértices.
    Las rectas que unen los vértices del triángulo antimedial con el ortocentro, cortan a sus lados opuestos en puntos sobre la nónica. Estos son los únicos puntos de intersección de la nónica con los lados del triángulo antimedial, a parte de sus vértices.
      Tres de las tangentes en los vértices del triángulo antimedial se cortan el retrocentro (isotómico conjugado del ortocentro, simediano del triángulo antimedial, el punto X69 de ETC).
      Las otras tres tangentes delimitan un triángulo perspectivo con ABC, con centro de perspectividad en X253 (conjugado cicloceviano del X69).

      Los centros de ortología de ABC respecto a A'B'C' correpondientes a los puntos X2, X4 y X7 son X4, X3 y X1, respectivamente.
      Los centros de ortología de A'B'C' respecto a ABC correpondientes a los puntos X2, X4 y X7 son X3, X4 y X5, respectivamente.

  • domingo 9 de noviembre del 2014

    Otra caracterización de la cuártica de Stammler



    Art of Problem Solving
    Let triangle ABC, circrumcircle (O). UV is a chord of (O), UV cuts AB,AC at Q,P. Denote M,N,J,R is midpoint of BP,CQ,PQ,UV.
    a. Prove MNJR is cyclic
    b. Prove that the symmetry point of R with mirror MN is lies on Euler circle of ABC
    (a appear in a contest in Vietnam and b is i propose when solve a)

    Synthetic solution by Luis González
    ( Mostrar/Ocultar figura )
      HCM.png


      Dados un triángulo ABC, un punto P y su triángulo ceviano DEF, sean Da, Eb y Fc los puntos medios de los segmentos EF, BE y CF.
      La circunferencia circunscrita a DaEbFc vuelve a cortar a la recta EF en el punto D'. Se denota por A' la reflexión de D' en la recta EbFc. Por el resultado enunciado: el punto A' está en la circunferencia de los nueve puntos de ABC.
      Procediendo cíclicamente, de definen los puntos B' y C'.
    ( Mostrar/Ocultar figura )
      Q066_P3p.png

      Los triángulos ABC y A'B'C' son perspectivos si sólo si el punto P está en la circunferencia circunscrita o en la cuártica de Stammler (Q066)

      Si (u:v:w) son las coordenadas baricéntricas de P:

    D' = (u(c^2uv(v-w)+w(b^2u(-v+w)+a^2v(2u+v+w))) : v(c^2uv(u+w)+w(-b^2u(u+2v-w)+a^2v(u+w))) : -w(-(u+v)(b^2u+a^2v)w+c^2uv(u-v+2w))).

      El simétrico de este punto respecto a EF es:

    A' =( u(v-w)(-b^2(u+v)w+c^2v(u+w)) : v(u+w)(c^2u(v-w)+a^2(u+v)w) : (u+v)w(a^2v(u+w)+b^2u(-v+w)) ).

      Las rectas AA', BB', CC' son concurrrentes si las coordenadas de P satifacen a una de las ecuaciones:

    a^2 yz + b^2 zx + c^2 xy =0,
    Q066:   a^2(b^2-c^2)y^2z^2 + b^2(c^2-a^2)z^2x^2 + c^2(a^2-b^2)x^2y^2= 0.

      La primera es la circunferencia circunscrita y la segunda es la cuártica de Stammler, con puntos dobles en los vértices de ABC y que pasa por los centros X1, X2, X4, X254, X1113, X1114, X1138, X2184, X3223, X3346, X3459, X8049, X9510.

      Si el punto P recorre la cuártica de Stammler Q066, el lugar geométrico del centro de perspectividad de ABC y A'B'C' es la cuártica (Q) de ecuación:

    ( Mostrar/Ocultar figura )
      Q066_P3.png
    Descargar fichero GeoGebra

      Como la cuártica Q066 es una curva algebraica de orden 4 de género cero (los vértices de ABC son tres puntos nodales) es unicursal, es decir, se puede parametrizar en función racional de un parámetro (§9.Curvas unicursales).
      Esto se puede lograr tomando un haz de cónicas pasando por los tres puntos dobles y por un punto más (el baricentro) de la cuártica Q006: y(x-z) + tz(y-xy)=0.
      A cada "t" corresponde una cónica del haz y ésta tiene con Q066 ocho puntos comunes, de los cuales ya se conocen siete (tomando cada punto doble dos veces); lo cual indica que a cada "t" corresponde otro único punto de intersección de ambas curvas, a parte de los elegidos. Este parámetro nos va a determinar todos los puntos de Q066.

    Pt = (1/(b^2(a^2-c^2)-c^2(a^2-b^2)t^2) :
    1/(b^2(c^2- a^2)-2c^2(b^2-a^2)t+c^2(b^2-a^2)t^2) :
    1/(b^2(a^2-c^2)-2b^2(a^2-c^2)t+c^2(a^2-b^2)t^2)).

      Tomando este punto, la intersección de las rectas AA', BB', CC' es el punto:

    Qt = ((b^2-c^2)^2t(-a^2t+b^2(1+t))(a^2-c^2(1+t)):
    (a^2-c^2)^2(1+t)(b^2+c^2t)(a^2t-b^2(1+t)) :
    (a^2-b^2)^2t(1+t)(b^2+c^2t)(a^2-c^2(1+t))).

      Eliminando el parámetro "t" obtenemos la ecuación implícita de la cuártica (Q), que tiene puntos dobles en los vértices de ABC y que pasa por los centros X2, X4, X12, X68, X252, X1312, X1313, X5627.

      La cuártica (Q) es conjugada isogonal de la cónica, que pasa por X3, X6, X24, X60, X143, X1511, X1986:




      Si el punto P recorre la la circunferencia circunscrita, el lugar geométrico del centro Q de perspectividad de ABC y A'B'C' es la circunferencia de los nueve puntos. El punto Q es la imagen de P en la homotecia de centro el baricentro y razón -1/2.

    ( Mostrar/Ocultar figura )
      Q066_P3O(R).png
    Descargar fichero GeoGebra

    Cada par que se muestra a continuacion representa los índices en la Enciclopedia de los Centros del Triangulo (ETC) de centros en la circunferencias circunscrita (el primero) y de los nueve puntos (el segundo), correspondientes a P y Q, respectivamente:
    {74, 113}, {98, 114}, {99, 115}, {100, 11}, {101, 116}, {102, 117}, {103, 118}, {104, 119}, {105, 120}, {106, 121}, {107, 122}, {108, 123}, {109, 124}, {110, 125}, {111, 126}, {112, 127}, {476, 3258}, {675, 5513}, {691, 5099}, {805, 2679}, {815, 5509}, {835, 5515}, {901, 3259}, {925, 136}, {927, 1566}, {930, 137}, {932, 5518}, {934, 5514}, {1113, 1313}, {1114, 1312}, {1141, 128}, {1290, 5520}, {1292, 5511}, {1293, 5510}, {1294, 133}, {1296, 5512}, {1297, 132}, {1298, 129}, {1300, 131}, {1303, 130}, {1305, 5190}, {1309, 10017}, {1310, 5517}, {1379, 2039}, {1380, 2040}, {2373, 1560}, {3565, 5139}, {5606, 5952}, {5951, 5950}, {5970, 9152}, {6078, 5519}, {6079, 5516}, {6093, 6092}, {9080, 9193}, {9150, 9151}.

    Más sobre la cúartica de Stammler:
    Triángulos semejantes y homólogos
    Cuártica de Stammler como lugar geométrico
    Cuártica de Stammler e hipérbola de Jerabek
    Cuárticas tipo Stammler
    Caracterización de la cuártica de Stammler
    Otra caracterización de la cuártica de Stammler



  • viernes 7 de noviembre del 2014

    Conjugado antigonal del incentro respecto a su triángulo ceviano



    ( Anopolis #1978, Antreas P. Hatzipolakis)

    Antigonal conjugate
    Let P be a point not on a sideline of ABC and not X(4). The antigonal conjugate of P is the point h(P) isogonal conjugate of the inverse-in-circumcircle of the isogonal conjugate of P. If P lies on the circumcircle then h(P)=X(4); otherwise, h(h(P))=P. If P lies on the line at infinity, then h(P)=P.


      Sean ABC un triángulo y A'B'C' el triángulo ceviano del incentro X1. El conjugado antigonal Q de X1 respecto A'B'C' está en la recta X1O', donde O' es el circuncentro de A'B'C'. Se verifica que
             Q = 3R X1 + 2r X399,
    donde R y r son los radios de las circunfencias circunscrita e inscrita a ABC, respectivamente.

      X399 es el punto de concurrencia de las circunferencias circunscritas a los triángulos TaB'C', A'TbC', A'B'Tc; donde, TaTbTc es el triángulo tangencial y A'B'C' es el triángulo obtenido reflejando cada vértice de ABC en el lado opuesto (Cosmin Pohoata, "On the Parry reflection point," Forum Geometricorum 8 (2008), 43-48).

      La primera coordenada baricéntrica de Q es:

    a^2 (a^5 + a^4(b+c) - 2a^3(b^2+c^2) - a^2(2b^3-b*c(b+c)+2c^3)+ a(b^4+b^2c^2+c^4) + (b-c)^2(b^3+c^3))

    con (6,9,13)-número de búsqueda en ETC: 0.812149174855219822306671631


    Una propiedad geométrica de Q:

    ( Mostrar/Ocultar figura )
      Anopolis1975.png
    Descargar fichero GeoGebra

    Hyacinthos #21642 (Feb 26, 2013)
    Let ABC be a triangle and A'B'C' the cevian triangle of I.
    Denote:
    B'a, C'a = the reflections of B',C' in AA', resp.
    A'a = (perpendicular to BB' from B'a) /\ (perpendicular to CC' from C'a).
    
    C'b, A'b = the reflections of C',A' in BB', resp.
    B'b = (perpendicular to CC' from C'b) /\ (perpendicular to AA' from A'b).
    
    A'c, B'c = the reflections of A',B' in CC', resp.
    C'c = (perpendicular to AA' from A'c) /\ (perpendicular to BB' from B'c).
    
    Are the circumcircles of A'aB'aC'a, B'bC'bA'b, C'cA'cB'c 
    concurrent?.
    
     Antreas P. Hatzipolakis
    
    Hyacinthos #21651 (Feb 28, 2013)
    The circumcircles are concurrent at non-ETC 0.812149174855220, which is, 
    the antigonal conjugate, wrt incentral triangle, of X(1)
    
    Randy Hutson
    


  • jueves 6 de noviembre del 2014

    Ejes radicales concurrentes y cúbica de Neuberg



    ( Anopolis #1973, Antreas P. Hatzipolakis)

      Sean ABC un triángulo, P un punto y Pa, Pb, Pc las reflexiones de P en BC, CA, AB, respectivamente.
      Denotamos por (Oab) y (Oac) las circunferencias circunscritas a los triángulos ABPb y ACPc, respectivamente; y por ea el eje radical de (Oab) y (Oac). Similarmente, se consideran los ejes radicales eb y ec.

      Los ejes radicales ea, eb y ec concurren en un punto Q si y solo si P está en la cúbica de Neuberg (K001, del catálogo de cúbicas de Bernard Gibert).

    ( Mostrar/Ocultar figura )
      Anopolis1972.png
    Descargar fichero GeoGebra

      El lugar geométrico del punto Q de intersección de los radicales ea, eb y ec,cuando P recorre la cúbica Neuberg, es la cúbica pK(X50, X3) (K073, del catálogo de cúbicas de Bernard Gibert).
      El punto Q es el inverso de P respecto a la circunferencia circunscrita a ABC.

  • miércoles 5 de noviembre del 2014

    Reflexión de la recta de Euler y triángulo tangencial



    ( Complemento a Anopolis #1971, Antreas P. Hatzipolakis)

      Sean ABC un triángulo, TaTbTc el triángulo tangencial y P un punto. Se denota por da, db y dc las reflexiones de la recta de Euler en los lados BC, CA y AB, respectivamente; y por Pa, Pb y Pc las reflexiones de P en las rectas da, db y dc, respectivamete.

      Los triángulos TaTbTc y PaPbPc son perspectivos si P está sobre el díámetro de ABC que pasa por X110 (foco de la parábola de Kiepert) o en la circunferencia con centro en la reflexión de X110 en X156 (centro de la circunferencia de los nueve puntos de TaTbTc) y que pasa por X155 (ortocentro de TaTbTc).


      La ecuación de la reflexión da de la recta de Euler en el lado BC es:

    ((b^2-c^2)^3-a^2(b^4-c^4))x - a^2(a^2-c^2)(a^2-b^2+c^2)y + a^2(a^2-b^2)(a^2+b^2-c^2)z=0.

      Si P(u:v:w), las coordenadas baricéntricas de la reflexión Pa de P en da son:

    (-a^2(a^8u - a^6(b^2(u+v-w)+c^2(u-v+ w)) - a^4(b^4(u-v)+c^4(u-w)+b^2c^2(-3u+v+w)) - a^2(b^2-c^2)^2(c^2v+b^2w) + (b^2-c^2)^4u ) :
    (b^2-c^2)^5u - a^10w - a^2(b^2-c^2)^3(c^2(u+v)+b^2w) + a^8(b^2(-v+w)+c^2(v+w)) - a^4(b^2-c^2)(c^4(u-v-w)+b^4(2u+w)+b^2c^2(-u+2v+w)) + a^6(-c^4(u+v-w)-4b^2c^2w+b^4(u+v+2 w)) :
    -(b^2-c^2)^5u - a^10v+ a^2(b^2-c^2)^3(c^2v+b^2(u+w)) + a^8(c^2(v-w)+b^2(v+w)) + a^6(-4b^2c^2v-b^4(u-v+w)+c^4(u+2v+w)) + a^4(b^2-c^2)(c^4(2u+v)+b^4(u-v-w) + b^2c^2(-u+v+2w)))

      Las coordenadas de los vértices del triángulo tangencial TaTbTc son:

    {(-a^2:b^2:c^2), (a^2:-b^2:c^2), (a^2:b^2:-c^2)}


      La condición analítica para que los triángulos TaTbTc y PaPbPc sean perspectivos es que P satisfaga a la ecuación de la recta:

    d:  b^2c^2(b^2-c^2)(2a^4 - a^2(b^2+c^2) - (b^2-c^2)^2x+...=0

    o a la ecuación circunferencia Γ con centro en P0 (reflexión de X110 en X156, dato proporcionado por Peter Moses) y pasa por X155 (ortocentro de TaTbTc).

    •   La circunferencia Γ tiene centro en P0 ((reflexión de X110 en X156):

    P0 = ( a^2 (a^14 - 4a^12(b^2+c^2) + a^10(5b^4+11b^2c^2+5c^4) - 13a^8b^2c^2(b^2+c^2) + a^6(-5b^8+12b^6c^2+2b^4c^4+12b^2c^6-5c^8) + 2a^4(2b^10-4b^8c^2+b^6c^4+b^4c^6-4b^2c^8+2c^10) - a^2(b^2-c^2)^2(b^8+b^6c^2-6b^4c^4+b^2c^6+c^8) + b^2c^2(b^2-c^2)^4(b^2+c^2)) : ... : ... ).

      Cuando el punto P recorre la circunferencia Γ, los triángulos TaTbTc y PaPbPc son perspectivos y SEMEJANTES. Si T1 es uno de los puntos de intersección de la recta d con Γ (el antipodal de X155 en Γ), el centro de semejanza S es el segundo punto de intersersección de la recta PT1 con la circunferencia circunscrita al triángulo tangencial Γ'. El centro de pesrpectividad es el punto Q (distinto de S) de intersección de las circunferencias circunscritas a los triángulos TaTbTc y PaPbPc.


    ( Mostrar/Ocultar figura )
      Anopolis1970Tangencial.png
    Descargar fichero GeoGebra


    •   La recta d contiene al diámetro de la circunferencia circunscrita a ABC con extremos en X74 (conjugado isogonal del punto del infinito de la recta de Euler) y X110 (foco de la parábola de Kiepert, centro de la hipérbola equilátera que pasa por el centro de la circunferencia circunscrita y por los vértices del triángulo tangencial).
      En este caso, los TaTbTc y PaPbPc son HOMOTÉTICOS y, cuando P varía sobre d, el lugar geométrico de los centros de homotecia Q es la recta d' que pasa por X110 y por X26 (centro de la circunferencia circunscrita al triangulo tangencial).
      La correspondencia:

    P ∈ d ↦ Q ∈ d'

    es una perspectividad con centro en T1, el antipodal de X155 en Γ:

    T1 = (a^2(a^10 - a^8(b^2+c^2) + a^6(-2b^4+5b^2c^2-2c^4) + 2a^4(b^2-c^2)^2(b^2+c^2) + a^2(b^2-c^2)^2(b^4-b^2c^2+c^4)- b^10+b^2c^2(b^6+c^6)-c^10) : ... : ... ).

    con (6,9,13)-número de búsqueda en ETC: -6.312998437457754226552932181
    ( Mostrar/Ocultar figura )
      Anopolis1970Tangencial0.png
    Descargar fichero GeoGebra


      El conjunto d∪Γ se tansforma en d'∪Γ' mediante una semejanza directa de centro en T2, el otro punto de intersección (distinto de T1) de las circunferencia Γ y Γ'.

    T2 = (a^2(a^2+b^2-c^2)(a^2-b^2+c^2) (a^12 - 4a^10(b^2+c^2) + a^8(7b^4+11b^2c^2+7c^4) - a^6(8b^6+11b^4c^2+11b^2c^4+8c^6) + a^4(7b^8+3b^6c^2+8b^4c^4+3b^2c^6+7c^8) - a^2(4b^10-3b^8c^2+3b^6c^4+3b^4c^6-3b^2c^8+4c^10) + (b^6-b^4c^2+b^2c^4-c^6)^2): ... : ...).

    con (6,9,13)-número de búsqueda en ETC: -1.88300482666727983296569296

  • domingo 26 de octubre del 2014

    Una propiedad del centro X3574



    ( Anopolis #1947, Antreas P. Hatzipolakis)

    X(3574) = X(21)-of-the-orthic-triangle if ABC is acute. Let A'B'C' be the orthic triangle, which is the pedal (and cevian) triangle of the orthocenter, H = X(4) . The Euler lines of the triangles HB'C', HC'A', HA'B' concur in X(3574). (Michel Garitte, July 7, 2009)


      Sea ABC un triángulo y A'B'C' el triángulo ceviano de un punto P.
    Se denota por:
      (Nab) y (Nac) las circunferencias de los nueve puntos de los triángulos BC'P y CB'P, respectivamente.
    da el eje radical de las circunferencias (Nab) y (Nac). Similarmente, db y dc.

      El lugar geométrico de los puntos P tal que los ejes radicales da, db y dc son concurrentes es una séptica circunscrita a los triángulos ABC y antimedial, y que pasa por el baricentro y ortocentro.


    ( Mostrar/Ocultar figura )
      Anopolis1947.png


      La ecuación baricéntrica de la circunferencia de los nueve puntos de BC'P es:
    2(u+v)(u+v+ w)(a^2yz+b^2zx+c^2xy) -
    (x+y+z) ((u(c^2v+SAw)+2v(c^2v+SAw))x + u(c^2u+SBw)y + (2SCu^2+u(2a^2v+SC(2v+w))+a^2v(2v+w))z) = 0.
      La ecuación de la circunferencia de los nueve puntos de CB'P, sesulta de ésta permutando y por z, b por c, SB por SC, v por w.

      El eje radical da de las circunferencias (Nab) y (Nac) es:
    (a^2u^2v-b^2u^2v+c^2u^2v+a^2uv^2-b^2uv^2+3c^2uv^2-a^2u^2w-b^2u^2w+c^2u^2w-2b^2uvw+2c^2uvw+2a^2v^2w-2b^2v^2w+2c^2v^2w-a^2uw^2-3b^2uw^2+c^2uw^2-a^2vw^2-2b^2vw^2+2c^2vw^2)x +
    (-2a^2u^3+2b^2u^3-3a^2u^2v+3b^2u^2v-3c^2u^2v-a^2uv^2+b^2uv^2-c^2uv^2-5a^2u^2w+b^2u^2w+c^2u^2w-8a^2uvw+2b^2uvw-2c^2uvw-2a^2v^2w-3a^2uw^2-b^2uw^2+c^2uw^2-4a^2vw^2)y +
    (2a^2u^3-2c^2u^3+5a^2u^2v-b^2u^2v-c^2u^2v+3a^2uv^2-b^2uv^2+c^2uv^2+3a^2u^2w+3b^2u^2w-3c^2u^2w+8a^2uvw+2b^2uvw-2c^2uvw+4a^2v^2w+a^2uw^2+b^2uw^2-c^2uw^2+2a^2vw^2)z = 0.

    Las escuaciones de los ejes radicales db y dc surgen por permutación cíclica.

      Los ejes radicales da, db y dc son concurrentes si y sólo si P está en una séptica circunscrita al triángulo ABC y a su triángulo antimedial; que pasa por el baricentro y el ortocentro. Los vértices A, B y C son puntos dobles.
    Las tangentes en los vértices del triángulo antimedial concurren en el retrocentro (el simediano del triángulo antimedial, el punto X69 de ETC).

      Cuando P es el ortocentro la intersección de los ejes radicales da, db y dc es el centro X3574,


    ( Mostrar/Ocultar figura )
      Anopolis1947H.png
    Descargar fichero GeoGebra



    CONSIDEREMOS OTROS EJES RADIACALES DE PARES DE CIRCUNFERENCIAS DE LOS NUEVE PUNTOS DESCRITAS ARRIBA


    •  Sean los ejes radicales ea de (Nba) y (Nca), eb de (Ncb) y (Nab), y ec de (Nac) y (Nbc).

      El lugar geométrico de los puntos P tal que los ejes radicales ea, eb y ec son concurrentes es una séptica circunscrita a los triángulos ABC, órtico, antimedial y excentral, y que pasa por el incentro, ortocentro y X1138. Los vértices de ABC son puntos triples.


    ( Mostrar/Ocultar figura )
      Anopolis1947a.png


      Cuando P = X1, X4 y X1138, los ejes radicales ea, eb y ec concurren en los centros X3649, X5 y X30 (es decir, son paralelos a la recta de Euler), respectivamente.

      There are only two points X such that the pedal triangle of X is similar to the cevian triangle of X. They are X(4) and X(1138). (Jean-Pierre Ehrmann, 1/4/03)

      Let X(1) be the incenter and A'B'C' is a central triangle. Let LA be the line through A' parallel to the Euler line of triangle BCX, let LB be the line through B' parallel to the Euler line of CXA, and let LC be the line through C' parallel to the Euler line of AXB. If the lines LA, LB, LC concur, the point of concurrence is the Kirikami-Schiffler point of the triangle A'B'C', denoted by KS(A'B'C'). Seiichi Kirikami (February 1, 2011)
    X(3649) = KS(Intouch Triangle)



    •  Sean los ejes radicales fa de (Nbc) y (Ncb), fb de (Nca) y (Nac), y fc de (Nab) y (Nba).

      El lugar geométrico de los puntos P tal que los ejes radicales fa, fb y fc son concurrentes es la "O(X5) orthopivotal cubic" (K060, del catálogo de cúbicas de Bernard Gibert).


    ( Mostrar/Ocultar figura )
      Anopolis1947K060.png
    Descargar fichero GeoGebra

    Si P está sobre la cúbica K060, se designa por Q el punto de intersección de los ejes radicales fa, fb y fc.
    Pares de centros del triángulo {P,Q} son:
    {X(4), X(4)}, {X(5), X(140)}, {X(13), X(396)}, {X(14), X(395)}, {X(79), X(3649)},
    {X(80), ((b+c)(-2a+b+c)/(b+c-a) : .. : ..)},
    {X(265), X(30)}, {X(621), X(298)}, {X(622, X(299},
    {X(1141), ((2a^8+(b^2-c^2)^4-4a^6(b^2+c^2)-2a^2(b^2-c^2)^2(b^2+c^2)+3a^4(b^4+c^4)) / (-(b^2-c^2)^2+a^2(b^2+c^2)) : ... : ...)},
    {X(5627), (2a^8-(b^2-c^2)^4-2a^6(b^2+c^2)+4a^2(b^2-c^2)^2(b^2+c^2)+a^4(-3b^4+8b^2c^2-3c^4)) / (2a^4-(b^2-c^2)^2-a^2(b^2+c^2))}

  • viernes, 24 de octubre del 2014

    Bisectriz interior, altura y mediana concurrentes

      Dado un triángulo ABC, sea MaMbMc el su triángulo medial.
    Lab es la recta que pasa por Ma y por el punto de intersección de la bisectriz interior en B y la altura desde C.
    Lac es la recta que pasa por Ma y por el punto de intersección de la bisectriz interior en C y la altura desde B.
    Ab es el punto de intersección de AB con Lab y Ac es el punto de intersección de AC con Lac.
     Similarmente se definen los puntos Bc, Ba, Ca y Cb.

      Las mediatrices de los segmentos BcCb, CaAc y AbBa son concurrentes.

    ( Mostrar/Ocultar figura )
      AGBICH.png
    Descargar fichero GeoGebra

      En coordenadas baricéntricas:
    Ab = (-a(a^2-b^2+c^2) : a^3+a^2c-a(b^2+c^2)-b^2c+c^3 : 0),
    Ac = (a(a^2+b^2-c^2) : 0 : -a^3-a^2b+a(b^2+c^2)-b^3+bc^2 ).
      Permutando cíclicamente se tienen las coordenadas de los puntos Bc, Ba, Ca y Cb.
      La mediatriz de BcCb (las otras dos mediatrices en cuestión, se obtienen por permutación cíclica) es:
    b(a^4+2a^3(b-c)+2ac(b^2-bc+c^2)+b^4-2b^3c-2b^2c^2-c^4)x
    +(a-c)(a^4-2a^2c^2+2ab^3+b^4+2b^3c+c^4)y
    - b(-a^4+2a^3c-2a^2b(b+c)-2a(b^3-b^2c+c^3)+b^4+2bc^3+c^4)z = 0.

      El punto M de intersección de estas mediatrices es:

    (a(a^6 - a^4(b+c)^2 + 2a^3(b^3+b^2c+bc^2+c^3) - 2a(b^5+b^4c+bc^4+c^5) - (b-c)^4(b+c)^2+a^2(b^2+c^2)^2) : ... : ... )

    con (6,9,13)-número de búsqueda en ETC: -88.3239018332220355783645503
      El punto M está, al menos, sobre las rectas X3X1104, X4X345.

  • jueves, 23 de octubre del 2014

    Parábolas asociadas al triángulo de reflexión

      Dado un triángulo ABC, sea A'B'C' el triángulo de reflexión que resulta de reflejar cada vértice de ABC en su lado opuestos. Se denota por:
      ℘ab la parábola de foco C y directriz BC', Dab el polo de CB' respecto a ℘ab.
      ℘ac la parábola de foco B y directriz CB', Dac el polo de BC' respecto a ℘ac.
      Similarmente, se definen las parábolas ℘bc, ℘ba, ℘ca y ℘cb, y los puntos Dbc, Dba, Dca y Dcb.

      Las rectas DabDac, DbcDba y DcaDcb delimitan un triángulo perspectivo con ABC.

    ( Mostrar/Ocultar figura )
      TrianguloReflexionParabolas.png
    Descargar fichero GeoGebra
      Las coordenadas baricéntricas de los vértices del triángulo de reflexión son:

    A' (-a^2 : 2SC : 2SB),   B' (2SC : -b^2 : 2SA),   C' (2SB : 2SA : -c^2).

      Ecuaciones de las parábolas (R radio de la circunferencia circunscrita):

    ab : b^2R^2SB^2x^2 + a^2c^2SC^2y^2+a^4b^2R^2z^2 + 2a^2b^2R^2SBzx - a^2b^2c^2SCxy= 0,
    ac : c^2R^2SC^2x^2 + a^4c^2R^2y^2 + a^2b^2SB^2z^2 - a^2b^2c^2SBzx + 2a^2c^2R^2SCxy = 0.

      El polo de la directriz de ℘ab respcto a ℘ac es:

    Dab ((a^2-b^2+c^2)(a^4-2a^2c^2+(b^2-c^2)^2) : b^2(-a^4+(b^2-c^2)^2) : -c^2(a^4-2a^2c^2+(b^2-c^2)^2)).

      El polo de la directriz de ℘ac respcto a ℘ab es:

    Dac (-(a^2+b^2-c^2)(a^4-2a^2b^2+(b^2-c^2)^2) : b^2(a^4-2a^2b^2+(b^2-c^2)^2) : -c^2(-a^4+(b^2-c^2)^2)).

      La ecuación de la recta DabDac es:

    -b^2c^2(a^4(b^2+c^2)+(b^2-c^2)^2(b^2+c^2)-2a^2(b^4-b^2c^2+c^4))x + c^4(-a^6-a^4(b^2-3c^2)-(b^2-c^2)^3-a^2(b^4-4b^2c^2+3c^4))y + b^4(-a^6+(b^2-c^2)^3+a^4(3b^2-c^2)-a^2(3b^4-4b^2c^2+c^4))z = 0.

      El centro de perspectividad P de ABC y el triángulo delimitado por las rectas DabDac, DbcDba, DcaDcb tiene primera coordenada:

    a^2 / (a^14(b^2+c^2) -
      a^12(5b^4+3b^2c^2+5c^4)+
      2a^10(5b^6+2b^4c^2+2b^2c^4+5c^6) -
      a^8(10b^8+3b^6c^2-2b^4c^4+3b^2c^6+10c^8) +
      a^6(5b^10+b^8c^2-2b^6c^4-2b^4c^6+b^2c^8+5c^10)-
      a^4(b^4-c^4)^2(b^4+b^2c^2+c^4)+
      2a^2b^2c^2(b^2-c^2)^4(b^2+c^2)-b^2c^2(b^2-c^2)^6 )

    con (6,9,13)-número de búsqueda en ETC: -3.347657323661639630845858

  • domingo, 19 de octubre del 2014

    Bisectriz interior, altura y simediana concurrentes

    a Kake, por su "cumple"

      Dado un triángulo ACUTÁNGULO ABC, sean:
    Ab es el único punto sobre AB tal que, en el triángulo AbBC, la simediana en Ab, la bisectriz interior en B y la altura en C son concurrentes.
    Ac es el único punto sobre AC tal que, en el triángulo AcBC, la simediana en Ac, la bisectriz interior en C y la altura en B son concurrentes.
      Similarmente se define los puntos Bc, Ba, Ca y Cb.

      Las mediatrices de los segmentos BcCb, CaAc y AbBa son concurrentes.


    ( Mostrar/Ocultar figura )
      AK-BI-CHmediatrz.png
    Descargar fichero GeoGebra

      Dado un punto en el lado AB de coordenadas baricéntricas A'(t:1-t:0), la condición para que, en el triángulo AbBC, la simediana en Ab, la bisectriz interior en B y la altura en C son concurrentes es:

    (a-b-c)(a+b-c)c^2t^2 - (a^2-b^2+c^2)(a^2-b^2-ac+c^2)t + a^2(a^2-b^2+c^2) = 0.

      El único valor de 't' que anula esta ecuación que determina un punto Ab sobre AB tal que, en el triángulo AbBC, la simediana en Ab, la bisectriz interior en B y la altura en C son concurrentes es:

    t = (a^2-b^2+c^2)(a^2-ac+c^2-b^2) - ((a^2-b^2+c^2)f2(a,b,c))½

    donde
    f2(a,b,c)= a^6-2a^5c-3a^4b^2+ 4a^3c(b^2+c^2)+3a^2b^2(b^2-c^2)-2ac(b^2-c^2)^2-(b^2-c^2)^3.
      Se tiene:

    Ab = ((a^2-b^2+c^2)(a^2-ac+c^2-b^2) - ((a^2-b^2+c^2)f2(a,b,c))½ :
    -a^4+a^3c+2a^2b^2-ac(b^2+3c^2)-b^4+c^4 + ((a^2-b^2+c^2)f2(a,b,c))½ : 0).

    Permutando cíclicamente las coordenadas de Ab y permutando 'b' y 'c', se tiene:

    Ac = ( a^4-a^3b+2a^2(b^2-c^2)+a(-b^3+bc^2)+(b^2-c^2)^2 -((a^2+b^2-c^2)f3(a,b,c))½ : 0 :
    -a^4+a^3b+2a^2c^2+b^4-ab(3b^2+c^2)-c^4)-c^2 +f3(a,b,c))½).

    donde
    f3(a,b,c)= a^6-2a^5b-3a^4c^2+ 4a^3b(b^2+c^2)+a^2c^2(-3b^2+3c^2)-2ab(b^2-c^2)^2+(b^2-c^2)^3 .

      Procediendo cíclamente sobre los vértices de ABC, obtennemos las coordenadas de los puntos Bc, Ba, Ca y Cb.

      Las mediatrices de los segmentos BcCb, CaAc y AbBa (cuyas ecuaciones son muy largas para exponerlas aquí) concurren en el punto de primera coordenada baricéntrica:

    f0(a,b,c) - 2a^2(b+c-a)((b^2+c^2-a^2)f1(a,b,c))½ + (a-b+c)(a^2+b^2-c^2)((a^2-b^2+c^2)f2(a,b,c))½ + (a+b-c)(a^2-b^2+c^2)((a^2+b^2-c^2)f3(a,b,c))½

    donde
    f0(a,b,c)= -2a^7+a^6(b+c) + a^5(b^2+c^2) - 2a^4(b^3+c^3) + 2a^3(b^4-b^3c-2b^2c^2-bc^3+c^4) - a^2(b^5-b^4c-4b^3c^2-4b^2c^3-bc^4+c^5) - a(b-c)^4(b+c)^2 + 2(b-c)^4(b+c)^3,

    f1(a,b,c)=-a^6 + a^4(3b^2-2bc+3c^2) + a^2(-3b^4+4b^3c-3b^2c^2+4bc^3-3c^4) + b^6-2b^5c+4b^3c^3-2bc^5+c^6.

  • domingo 12 de octubre del 2014

    Centros de circunferencias de Euler y circunferencias concurrentes

    ( Anopolis #1891, Antreas P. Hatzipolakis)

      Sea ABC un triángulo y O su circuncentro. Se denota por Ab, Ac las reflexiones de A en OB y OC, respectivamente. Na es el centro de la circunferencia de los nueve puntos del triángulo AAbAc. Similarmente se definen Nb y Nc.

      Las circunferencias circunscritas a los triángulos ABC, ANbNc, BNcNc y CNaNb son concurrentes en X930.
      Las circunferencias circunscritas a los triángulos NaNbNc, NaBC, NbCA y NcAB son concurrentes en:

    Q = (a^2(a^6(b^2+c^2)- a^4(3b^4+4b^2c^2+3c^4)+ a^2(3b^6+2b^4c^2+2b^2c^4+3c^6)-b^8+b^6c^2+b^2c^6-c^8) : ... : ...)

    con (6,9,13)-número de búsqueda en ETC: 8.07910137097857081131659206
      Este punto ha sido inculido en ETC como el X6102.

    En coordenadas baricéntricas:
    Ab = (a^2(a-c)(a+c):-(a^2-c^2)(a^2+b^2-c^2):-c^2(a^2+b^2-c^2))
    Ac = (a^2(a-b)(a+b):-b^2(a^2-b^2+c^2):-(a^2-b^2)(a^2-b^2+c^2)
    Na = (a^2(-b^8+b^6c^2+b^2c^6-c^8+a^6(b^2+c^2)- a^4(3b^4+4b^2c^2+3c^4)+ a^2(3b^6+2b^4c^2+2b^2c^4+3c^6)) :
      -a^8c^2+(b^2- c^2)^3(b^4+c^4)+a^6(-b^4+b^2c^2+4c^4)+ a^4(3b^6+b^4c^2+b^2c^4-6c^6)+ a^2(-3b^8+2b^6c^2+2b^4c^4-5b^2c^6+4c^8) :
     -a^8b^2+ a^6(4b^4+b^2c^2-c^4)-(b^2-c^2)^3(b^4+c^4)+ a^4(-6b^6+b^4c^2+b^2c^4+3c^6)+ a^2(4b^8-5b^6c^2+2b^4c^4+2b^2c^6-3c^8) ).

    ( Mostrar/Ocultar figura )
      Anopolis1891.png


      Los triángulos ABC y NaNbNc son ortológicos.
    • El centro de ortología de NaNbNc respecto a ABC es el punto, sobre la circunferencia circunscrita a NaNbNc:

    D = (a^2(a^6(b^2+c^2)- 3a^4(b^4+c^4)+a^2(3b^6-2b^4c^2-2b^2c^4+3c^6)-b^8+b^6c^2+b^2c^6-c^8) : ... :...)

    con (6,9,13)-número de búsqueda en ETC: 5.485624417414120257712782094

      El punto D ha sido incorporado a ETC con el número X(6102) = 19th Hatzipolakis-Montesdeoca Point.

    • El centro de ortología de ABC respecto a NaNbNc es X1141 (sobre la circunferencia circunscrita a ABC).
    X(1141) = GIBERT POINT
    X(1141) was first noted (Hyacinthos #1498, September 25, 2000) by Bernard Gibert as a point of intersection of the circumcircle and certain cubic, denoted Kn. To define Kn, note first that the Neuberg cubic is the locus of a point M such that the reflections of M in the sidelines of triangle ABC are the vertices of a triangle perspective to ABC. The locus of the perspector is the cubic Kn, and X(1141) is the point, other than A,B,C, in which Kn meets the circumcircle.




  • sábados 11 de octubre del 2014

    Triángulos precevianos y rectas de Euler concurrentes

      Sean ABC un triángulo, P un punto y PaPbPc el triángulo preceviano de P.
      Se denotan por da, db y dc las rectas de Euler de los triángulos PaBC, PbCA y PcAB, respectivamente.

      Las rectas de Euler da, db y dc son concurrentes si y solo si P está en una nónica (ver su ecuación baricéntrica), que pasa por los vértices de los triángulos ABC (dobles), medial y excentral, por los centros X1, X2 y X6.

    Las rectas da, db y dc son concurrentes cuando los triángulos precevianos son los triángulos excentral, antimedial y tangencial. En el primer caso las rectas concurren en X100, en el segundo son paralelas a la recta de Euler de ABC y en el tercero concurren en el circuncentro.
    ( Mostrar/Ocultar figura )
      Anopolis1884math.png

      La ecuación de la recta de Euler del triángulo PaBC es:
    a^4v(v-w)w-(b^2-c^2)u(b^2(u^2+v^2-w^2-u(2v+w))+ c^2(u^2-v^2+w^2-u(v+2w)))+ a^2(-c^2(u^3-3vw(v+w)-u^2(3v+2w)+ u(3v^2+4vw+w^2))+ b^2(u^3-3vw(v+w)-u^2(2v+3w)+ u(v^2+4vw+3w^2)))x + ((-b^2+c^2)u+ a^2(u-2w))(-u(c^2(u-w)+b^2w)+ a^2(u^2-u(2v+w)+v(v+2w)))y + ((b^2-c^2)u+ a^2(u-2v))(u(b^2(u-v)+c^2v)- a^2(u^2+w(2v+w)-u(v+2w)))z = 0.

  • jueves 9 de octubre del 2014

    Cónicas homofocales y parábola envolvente de polares

      Sean ABC un triángulo, MaMbMc el triángulo medial y P un punto, la recta da que une P con Ma vuelve a cortar a la circunferencia circunscrita a PBC en Pa; sea Fa la reflexión de Pa en la mediatriz de BC. Similarmente, procediendo cíclicamente, se consideran los puntos Fb y Fc.

      Los triángulos ABC y FaFbFc son perspectivos si y sólo si el punto P está sobre la quíntica Θ de ecuación baricéntrica:

    b^2 c^4 x^4 y + 2 a^2 c^4 x^3 y^2 + b^2 c^4 x^3 y^2 - 2 c^6 x^3 y^2 - a^2 c^4 x^2 y^3 - 2 b^2 c^4 x^2 y^3 + 2 c^6 x^2 y^3 - a^2 c^4 x y^4 - b^4 c^2 x^4 z - b^4 c^2 x^3 y z + b^2 c^4 x^3 y z + 4 a^4 c^2 x^2 y^2 z - 4 b^4 c^2 x^2 y^2 z - 4 a^2 c^4 x^2 y^2 z + 4 b^2 c^4 x^2 y^2 z + a^4 c^2 x y^3 z - a^2 c^4 x y^3 z + a^4 c^2 y^4 z - 2 a^2 b^4 x^3 z^2 + 2 b^6 x^3 z^2 - b^4 c^2 x^3 z^2 - 4 a^4 b^2 x^2 y z^2 + 4 a^2 b^4 x^2 y z^2 - 4 b^4 c^2 x^2 y z^2 + 4 b^2 c^4 x^2 y z^2 - 4 a^4 b^2 x y^2 z^2 + 4 a^2 b^4 x y^2 z^2 + 4 a^4 c^2 x y^2 z^2 - 4 a^2 c^4 x y^2 z^2 - 2 a^6 y^3 z^2 + 2 a^4 b^2 y^3 z^2 + a^4 c^2 y^3 z^2 + a^2 b^4 x^2 z^3 - 2 b^6 x^2 z^3 + 2 b^4 c^2 x^2 z^3 - a^4 b^2 x y z^3 + a^2 b^4 x y z^3 + 2 a^6 y^2 z^3 - a^4 b^2 y^2 z^3 - 2 a^4 c^2 y^2 z^3 + a^2 b^4 x z^4 - a^4 b^2 y z^4=0.

    ( Mostrar/Ocultar figura )
      parabolaFFPmath.png

      Si P=(u:v:w), la recta da es (v-w)x-uy+uz=0, que vuelve a corta a la circunferencia

    a^2 y z + b^2 z x + c^2 x y - ((c^2 u v + b^2 u w + a^2 v w) x (x + y + z))/( u (u + v + w))=0

    en el punto

    Pa = ( a^2u(u+v+w) : -b^2u(u+v-w)-(u-v+w)(c^2u+a^2w) : -b^2u(u+v-w)-a^2v(u+v-w)-c^2u(u-v+w)).

      La reflexión de Pa en la mediatriz de BC es

    Fa = ( a^2u(u+v+w) : -2b^2u(u+v)+v(2c^2u-a^2(u+v-w)) : -2c^2u(u+w)+w(2b^2u-a^2(u-v+w)) ).

      Finalmente, la recta AFa tiene por ecuación:

    (2c^2u(u+w)+ w(-2b^2u+a^2(u-v+w)))y+(-2b^2u(u+v)+ v(2c^2u-a^2(u+v-w)))z=0.

    ( Mostrar/Ocultar figura )
      parabolaFFP1.png


      Puntos en la quíntica Θ:
    Los vértices de los triángulos ABC, medial MaMbMc y circunceviano del simediano A(K)B(K)C(K) (indicación de Bernard Gibert).
    Los centros X3, X6, X15, X16, X187, X524.
    Los puntos del infinito de la quíntica, además de X524, son los puntos cíclicos (dobles).
    Las tangentes en los vértices de ABC son la simedianas.
    La asíntota con punto en el infinito X524 es:

    (b^2-c^2)(11a^6 - 6a^4(b^2+c^2) - 3a^2(5b^4-7b^2c^2+5c^4) + 2b^6+3b^4c^2+3b^2c^4+2c^6)x+....=0

    Si concideramos los pares {P,Q}, siendo Q el centro de perspectividad de los triángulos ABC y FaFbFc, cuando son perspectivos, tenemos:

    {X3, X6}; {X6,X25}; {X15, X62}; {X16, X61}; {X187, X6}; {X524,X2}.

      El eje de Brocard interseca a la quíntica en los cinco puntos: X3, X6, X15, X16 y X187.

    ==============================================================



      Otra interpretación geométrica de los puntos Fa, Fb y Fc, surge del siguiente resultado, que ha compartido conmigo Francisco Javier García Capitán:
      Problem. Prove an generalise that the envolope of the polar of a given point for a system of confocals is a parabola touching the axes of the confocals and having the given point on its directrix.
    E. H. Askwith, A Course of Pure Geometry, p. 255, problem 21.


      Podemos aplicarlo a la Geometría del Triángulo cuando tenemos un triángulo ABC y un punto P.

      Sea ℘a la parábola envolvente de las polares de P respecto a las cónicas homofocales de focos en los vértices B y C; su foco lo denotamos por Fa. Similarmente, procediendo cíclicamente, se consideran los focos Fb y Fc de las parábolas ℘b y ℘c, respectivamente.
      Los triángulos ABC y FaFbFc son perspectivos si y sólo si el punto P está sobre la quíntica Θ.

    ( Mostrar/Ocultar figura )
      parabolaFFPPa.png

      Para determinar la cónica envolvente de las polares de P, respecto a las cónicas homofocales de focos B y C, nos basta con conocer cinco de ellas:
    Las dos tangentes, en P, a las dos cónicas de focos B y C y que pasan por P.
    Y las tres rectas (cónicas degeneradas de la familia de cónicas homofocales de focos B y C) BC, mediatriz de BC y la recta del infinito.

    Tenemos, así, cinco tangentes y la cónica queda determinada; tratándose de una parábola ℘a, al ser tangente a la recta del infinito.

    De la parábola ℘a conocemos dos pares de tangentes ortogonales, por lo que cada par se cortan en su directriz. Esta es, por tanto, la recta da que pasa por P y el punto medio Ma de BC. El foco Fa es el punto de intersección de las polares de P y Ma respecto a la parábola ℘a.
    ( Mostrar/Ocultar figura )
      parabolaFFP.png


  • miércoles 8 de octubre del 2014

    Rectas de Euler concurrentes

    ( Anopolis #1872, Antreas P. Hatzipolakis)

      Sean ABC un triángulo y A'B'C' el triángulo ceviano del incentro.
    Oab y Oac son los circuncentros de los triángulos ABA' y ACA', respectivamente.
    Oa es el circuncentro del triángulo AOabOac. Ob y Oc se definen de manera similar.

      Las recta de Euler de los triángulos ABC, AOabOac, BObcOba, COcaOcb concurren en el inverso del ortocentro en la circunferencia circunscrita, X186.
      El ortocentro del triángulo OaObOc es el punto sobre la recta de Euler:

    X = (r^2 + 2 r R - R^2 + s^2)*X(3) + R^2*X(4),
    R y r, radios de las circunferencias circunscrita e inscrita, y s el semiperímetro.

    ( Mostrar/Ocultar figura )
      Anopolis1872.png

      El circuncentro del triángulo ABA' es:

    Oab = (a^2(bc^2+cSA) : b^2cSB + b(S^2+SASB)) : c^2(-bSB+cSC)),

      El circuncentro del triángulo ACA' es:

    Oac = (a^2(cb^2+bSA) : b^2(-cSC+bSB): c^2bSC + c(S^2+SASC)) ),

      El circuncentro del triángulo AOabOac es:

    Oa = (a^2(b(3SA+SB)+c(3SA+SC)) : b^2(2bSB+c(SB-SC)) : -c^2(b(SB-SC)-2cSC) ).


      El baricentro del triángulo AOabOac es:

    ((-2a^2+(b-c)^2)(b+c) : b(-a^2+2b(b-c)) : c(-a^2+2c(-b+c))).


      La ecuación de la recta de Euler de AOabOac es:
    -a^2bc(b-c)(a^2-b^2-c^2)x - c(a^2-b^2+c^2)(a^4+a^2c(b-2c)+c(b-c)^2(b+c))y + b(a^2+b^2-c^2)(a^4+a^2b(-2b+c)+b(b-c)^2(b+c))z = 0.

      Cíclicamente, se obtienen las ecuaciones de las rectas e Euler de los triángulos BObcOba y COcaOcb. Las tres concurren en el punto X186},

    (a^2(a^2-b^2+c^2)(a^2+b^2-c^2)(a^2-b^2-bc-c^2)(a^2-b^2+bc-c^2) : ... : ...).




      El ortocentro del triángulo OaObOc

    (a^2(a^2-b^2-bc-c^2)(a^5(b+c)+bc(b^2-c^2)^2 - 2a^3(b^3+c^3) - a^2bc(b^2+c^2) + a(b^5-b^4c-bc^4+c^5)) : ... : ...).

    con (6,9,13)-número de búsqueda en ETC: 3.5037197568390148045586626416,

    Nota: El punto X ha sido incluido en ETC como X(6097) = 14th Hatzipolakis-Montesdeoca Point

  • martes 7 de octubre del 2014

    Rectas de Euler y triángulos ortológicos

    ( Anopolis #1881, Antreas P. Hatzipolakis)

      Sean ABC un triángulo, Q un punto sobre la recta de Euler y P su conjugado isogonal (queda sobre la hipérbola de Jerabek).
    Sean Qa, Qb, Qc los mismos puntos que Q en los triángulos PBC, PCA, PAB, respectivamente.
    Se denota por Mqa, Mqb, Mqc los puntos medios de AQa, BQb, CQc, respectivamente.

      Los triángulos ABC y MqaMqbMqc son ortológicos.
      El lugar geométrico de los centros de ortología de ABC con respcto a MqaMqbMqc, cuando Q varía en la recta de Euler, es la hipérbola de Jerabek.
      El lugar geométrico de los centros de ortología de MqaMqbMqc con respcto a ABC, cuando Q varía en la recta de Euler, es una cúbica, con punto doble en el centro de la circunferencia de los nueve pountos, X5, con puntos en la recta del infinito los dos de la hipérbola de Jerabek (X2574, X2575) y el conjugado isogonal (X5663) del antipodal del punto de Tixier; esta cúbica también pasa por el conjugado isogonal (X2883) del ortocentro respecto al triángulo medial.

    ( Mostrar/Ocultar figura )
      Anopolis1881.png

      Un punto Q sobre la recta de Euler tal que GQ:QO=m:n, tiene coordenadas baricéntricas:

    (a^4(3m+n) - a^2(b^2+c^2) (3m+2n)+(b^2-c^2)^2n:...:...)

      Para un punto P(u:v:w) arbitrario del plano, el punto Qa respecto al triángulo PBC, que es el mismo que Q respecto al triángulo ABC es:

    Qa = (-(b^2-c^2)^2nu^2 + a^2u(b^2(2nu+3m(u+v-w)) + c^2(2nu+3m(u-v+w))) - a^4(nu^2+3m(u^2+2vw+u(v+w))):
    -a^4(-3mvw + nu(u+2v+w)) - (b^2-c^2)u(-c^2(3mv+n(u+2v+w)) + b^2(3m(u+v)+n(u+2v+w))) + a^2(c^2(3mv(u-w) + 2nu(u+2v+w)) + b^2(2nu(u+2v+w) + 3m(u^2+uv+2uw+vw))):
    -a^4(-3mvw+nu(u+v+2w)) - (b^2-c^2)u(b^2(3mw+n(u+v+2w)) - c^2(3m(u+w) + n(u+v+2w))) + a^2(b^2(3m(u-v)w + 2nu(u+v+2w)) + c^2(2nu(u+v+2w) + 3m(u^2+2uv+uw+vw)))).

      Las coordenadas de los puntos Qb y Qc se obtienesn por permutación cíclica.
      El punto medio de A y Qa es:

    Mqa = (-(b^2-c^2)^2u(3m(u+v+w)+n(4u+3(v+w)))- a^4(6m(u+v)(u+w)+nu(4u+3(v+w)))+ a^2u(b^2(3m(3u+3v+w)+2n(4u+3(v+w)))+ c^2(3m(3u+v+3w)+ 2n(4u+3(v+w)))) :
    -a^4(-3mvw+ nu(u+2v+w))-(b^2- c^2)u(-c^2(3mv+n(u+2v+w))+ b^2(3m(u+v)+n(u+2v+w)))+ a^2(c^2(3mv(u-w)+2nu(u+2v+w))+ b^2(2nu(u+2v+w)+ 3m(u^2+uv+2uw+vw))) :
    -a^4(-3mvw+ nu(u+v+2w))-(b^2- c^2)u(b^2(3mw+n(u+v+2w))- c^2(3m(u+w)+n(u+v+2w)))+ a^2(b^2(3m(u-v)w+2nu(u+v+2w))+ c^2(2nu(u+v+2w)+3m(u^2+2uv+uw+vw))))



     • El centro de ortología de MqaMqbMqc con respecto a ABC, para cualquier punto P del plano, es:

    ( a^4n(v+w) + a^2(b^2+c^2)(3m(u+v+w)+2n(2u+v+w)) - (b^2-c^2)^2(3m(u+v+w)+n(4u+3(v+w))) : ... : ...).


      En particular, si P es el conjugado isogonal de Q, poniendo m=tn:

    ρ(x:y:z) = ( - a^8(b^2+c^2)(5+6t) + a^6(2b^2c^2(2+3t)^2+3b^4(4+5t)+3c^4(4+5t)) - a^4(b^2+c^2)(3c^4(2+3t)+b^4(6+9t) - a^2(b^2-c^2)^2(3b^2c^2(2+t)(2+3t)^2+b^4(4+3t) +3(b^2-c^2)^4(b^2+c^2)(1+t) -b^2c^2(28+78t+72t^2+27t^3)) + c^4(4+3t)) : ... : ...)

      Eliminado ρ y t, resulta la ecuación implícita de una cúbica.


     • El centro de ortología de ABC con respecto a MqaMqbMqc , para cualquier punto P del plano, es:

    (u/(a^2(3m(u+v)(u+w)+2nu(u+v+w))- u(b^2(3m(u+v)+2n(u+v+w))+ c^2(3m(u+w)+2n(u+v+w)))):...:...).

      En particular, si P es el conjugado isogonal de Q, poniendo m=tn:

    ρ(x:y:z) = (1/(2(a-b-c)(a+b-c)(a-b+c)(a+b+c)(a^2+b^2- c^2)(a^2+b^2+c^2) + 3(a^8-3a^6b^2+4a^4b^4-3a^2b^6+b^8-2a^6c^2- 12a^4b^2c^2-12a^2b^4c^2-2b^6c^2+13a^2b^2c^4+ 2a^2c^6+2b^2c^6-c^8)t - 9a^2b^2(a^4-2a^2b^2+b^4+3a^2c^2+3b^2c^2-4c^4)t^2) : ...:...).

      Eliminado ρ y t, resulta la ecuación implícita de la hipérbola de Jerabek:

    a^2(b^2-c^2)(-a^2+b^2+c^2)yz+ b^2(-a^2+c^2)(a^2-b^2+c^2)zx+(a^2-b^2)c^2(a^2+b^2-c^2)xy = 0.



  • domingo 5 de octubre del 2014

    Circunferencias tangentes a los lados de un triángulo

    (Inspirado en Problem JP27, Ercole Suppa,)

      Sean ABC un triángulo, P un punto y A'B'C' su triángulo ceviano.
    Se denota por:
      Φab la circunferencia tangente en B a BC y que pasa por C'.
      Φac la circunferencia tangente en C a BC y que pasa por B'.

    t'a la tangente exterior (distinta de BC) a las circunferencias Φab y Φac.

      De forma similar, procediendo cíclicamente, se definen las tangentes exteriores t'b y t'c a las correspondientes circunferencias.

      Las tangentes t'a, t'b y t'c delimitan un triángulo DEF perspectivo con ABC.

    ( Mostrar/Ocultar figura )
      SuppaP27Ceviano.png
      En coordenadas baricéntricas, si P(u:v:w), se tienen las ecuaciones de las circunferrencias:

    Φab: a^2yz+b^2zx+c^2xy-(x+y+z)(c^2vx/(u+v)+a^2z)=0
    Φac: a^2yz+b^2xz+c^2xy-(x+y+z)(b^2wx/(u+w)+a^2y)=0.

    Y la ecuación de la tangente exterior (distinta de BC) a estas circunferencias es:
    t'a: (a^4(u+v)^2(u+w)^2+(b^2w(u+v)-c^2v(u+w))^2- 2a^2(u+v)(u+w)(b^2w(u+v)+c^2v(u+w)))x + 4a^2c^2u(u+v)(u+w)^2y + 4a^2b^2u(u+v)^2(u+w)z=0.

      El centro de perspectividad Q de los triángulos ABC y DEF tiene primera coordenada baricéntrica:

    u/((v+w)(b^4(u+v)^2w^2 + c^4v^2(u + w)^2 + a^4(u+v)^2(u+w)^2 - 2a^2(u+v)(u+w)(b^2(u+v)w+c^2v(u+w))- 2b^2c^2(u+v)(u+w)(vw+2u(v+w)))).



      El centro de perspectividad Q de los triángulos ABC y DEF coincide con P si P está sobre la elipse circunscrita de Steiner.
      Además, en este caso:
      • Las seis circunferencias Φab, Φac, Φbc, Φba, Φca, Φcb concurren en el punto T donde la recta X99P vuelve a corta a la circunferencia circunscrita.
      • Los puntos de tangencia de las tangentes t'a, t'b y t'c con las circunferencias Φab, Φac, Φbc, Φba, Φca, Φcb están en la circunferencia reflexión de la circunferencia circunscrita en el punto T.

    ( Mostrar/Ocultar figura )
      SuppaP27Steiner.png

      La correspondencia P↦T, entre centros del triángulo incluidos actualmente en ETC, sobre la elipse circunscrita de Steiner y centros en la circunferencia circunscrita, está dada por los siguientes pares {P,T} de sus índices:
    {99,110}, {190,101}, {290,98}, {648,112}, {664,109}, {666,919}, {668,100}, {670,99}, {671,111}, {886,?}, {889,898}, {892,691}, {903,106}, {1121,2291}, {1494,74}, {2479,?}, {2480,?}, {2481,105}, {2966,2715}, {3225,699}, {3226,727}, {3227,739}, {3228,729}, {4555,901}, {4562,813}, {4569,934}, {4577,827}, {4586,825}, {4597,4588}, {5641,842}.

      Los tres puntos T, sobre la circunferencia circunscrita, que no figuran en ETC, son:
      • Si P= X886,

    (1/((b^2-c^2)(a^2(b^2+c^2)-2b^2c^2)) : ... : ....),

    con (6,9,13)-número de búsqueda en ETC: 6.784805747166842285745353786
    X(9150) = TRILINEAR POLE OF LINE X(6)X(99)
    Barycentrics 1/[(b^2 - c^2)(a^2b^2 + a^2c^2 - 2b^2c^2)] : :
    Let A' = BC∩X(2)X(39), and define B' and C' cyclically. The circumcircles of AB'C', BC'A', CA'B' concur in X(9150). (Randy Hutson, January 15, 2016)

      • Si P= X2479 (de los dos puntos de intersección de la recta de Euler con la elipse circunscrita de Steiner, X2479 es la más cercano a X3).

    (a^2 (a^8 - a^6(b^2+c^2) + a^4b^2c^2 - a^2(b^2-c^2)^2(b^2+c^2) + b^8- b^6c^2-b^2c^6+c^8 + (-2a^4+a^2(b^2+c^2)+(b^2-c^2)^2)Λ) : ... : ....),

    donde Λ=[a^8 - a^6(b^2+c^2) + a^4b^2c^2 - a^2(b^6-b^4c^2-b^2c^4+c^6) + b^8-b^6c^2-b^2c^6+c^8]½ y con (6,9,13)-número de búsqueda en ETC: 6.770997724920497989820816646
     • Si P= X2480 (de los dos puntos de intersección de la recta de Euler con la elipse circunscrita de Steiner, X2480 es la más alejado de X3).

    (a^2 (a^8 - a^6(b^2+c^2) + a^4b^2c^2 - a^2(b^2-c^2)^2(b^2+c^2) + b^8- b^6c^2-b^2c^6+c^8 - (-2a^4+a^2(b^2+c^2)+(b^2-c^2)^2)Λ) : ... : ....),

    donde Λ=[a^8 - a^6(b^2+c^2) + a^4b^2c^2 - a^2(b^6-b^4c^2-b^2c^4+c^6) + b^8-b^6c^2-b^2c^6+c^8]½ y con (6,9,13)-número de búsqueda en ETC: 0.11661014590270525314320708434


      Consideremos ahora los segundos puntos A'1, B'1, C'1 de intersección de los pares de circunferencias Φbc y Φcb, Φca y Φac, Φab y Φba, respectivamente.

      El triángulo A'1B'1C'1 es perpectivo con ABC, con centro de perspectividad en el producto baricéntrico Y de P y el simediano.
      También los triángulos A'1B'1C'1 y DEF son perspectivos.

    ( Mostrar/Ocultar figura )
      SuppaP27Ceviano1.png

      Se tiene que A'1 = (-a^2vw/(v+w) : b^2v : c^2w ); B'1 = (a^2u : -b^2uw/(u+w) : c^2w); y C'1 = (a^2u : b^2v : -c^2uv/(u+v)). Por lo que el centro de perspectividad de A'1B'1C'1 y ABC es:

    (a^2u : b^2v : c^2w).


      El punto de concurrencia de las rectas DA'1, EB'1, FC'1 tiene coordenadas (a^2u f(a,b,c,u,v,w):...:...), donde f(a,b,c,u,v,w) es un polinomio de grado 12 en las variables (a,b,c) y (u,v,w).

      En el caso particular de P=X69 los triángulos A'1B'1C'1 y DEF coinciden con la reflexión de ABC respecto al circuncentro.

  • jueves 25 de septiembre del 2014

    Lugares geométricos asociados a ejes radicales

    ( Anopolis #1845, Antreas P. Hatzipolakis)

      Sean ABC un triángulo, P un punto y A'B'C' su triángulo ceviano.
    Se denota por:
    Ab y Ac las proyecciones ortogonales de B' y C' sobre AA'.
      (Oab) y (Oac) las circunferencias circunscritas a los triángulos BB'Ab y CC'Ac, respectivamente.
      Similarlente se definen las circunferencias (Obc), (Oba), (Oca) y (Ocb).
      Ra es el eje radical de las circunferencias (Oab) y (Oac). Rb es el eje radical de las circunferencias (Obc) y (Oba). Rc es el eje radical de las circunferencias (Oca) y (Ocb).
      Finalmente, A* = Ra ∩ BC, B* = Rb ∩ CA, C* = Rc ∩ AB.

      Si (u:v:w) son las coordenadas baricéntricas del punto P,
    Ab = (c^2v(2u+w)(v+w)-w(a^2v(2u-v+w)-b^2(2u-v)(v+w)) : -vw(a^2v-c^2v-b^2(v+2w)) : w^2(-a^2v+c^2v+b^2(v+2 w))),
    Ac = (c^2v(2u-w)(v+w)+w(-a^2v(2u+v-w)+b^2(2u+v)(v+w)) : v^2((-a^2+b^2)w+c^2(2v+w)) : vw((-a^2+b^2)w+c^2(2v+w))).

      El punto de intersección del eje radical Ra con lado BC es:
    A* = (0 : (u+v)(c^2SCuv-b^2SBuw+SBSCvw-a^2b^2w^2)(b^2uw+v(SAu-SBv+SCw)) : (u+w)(-c^2SCuv-a^2c^2v^2+b^2SBuw+SBSCvw)(c^2uv+w(SAu+SBv-SCw)).

      El lugar geométrico de los puntos P tales que A*B*C* es el triángulo ceviano de un punto Q es una séxtica con puntos dobles en los vértices de ABC, pasa por los vértices del triángulo excentral IaIbIc y por los pies de las bisectrices interiores y exteriores.

    ( Mostrar/Ocultar figura )
      Anopolis1845.png
    ( Mostrar/Ocultar figura )
      Anopolis1845m.png

      Esta séxtica pasa por los centros del triángulo X1, X2, X3, X15, X16.
      Si P es el incentro, el punto Q es el X21, punto de Schiffler.
      Si P es el baricentro, el punto Q es el punto de primera coordenada

    (5a^2-b^2-c^2) (a^4-6a^2b^2+b^4+2a^2c^2+2b^2c^2-3c^4) (a^4+2a^2b^2-3b^4-6a^2c^2+2b^2c^2+c^4),

    con (6,9,13)-número de búsqueda en ETC: 0.9249381788103438608692305
      Si P es el circuncentro, el punto Q es el punto de primera coordenada

    a^2(a^4-(b^2-c^2)^2)^2 (a^12 - 5a^10(b^2+c^2) + 5a^8(2b^4+3b^2c^2+2c^4) - 2a^6(5b^6+7b^4c^2+7b^2c^4+5c^6) + a^4(5b^8+2b^6c^2+4b^4c^4+2b^2c^6+5c^8) - a^2(b^2-c^2)^4(b^2+c^2) - b^2c^2(b^2-c^2)^2(b^4+c^4) ),

    con (6,9,13)-número de búsqueda en ETC: -6.0305312940593606748466345


      El lugar geométrico de los puntos P tales que los ejes radicales Ra, Rb y Rc son concurrentes es una curva algebraica de grado once con puntos cuádruples en los vértices de ABC, pasa por los vértices del triángulo excentral IaIbIc y por los pies de las alturas.

    ( Mostrar/Ocultar figura )
      Anopolis1845C11.png

      Esta curva pasa por los centros del triángulo X1, X4, X110.
      Si P es el incentro, el punto de concurrencia de los ejes radicales es el circuncentro.
      Si P es el ortocentro, el punto donde se cortan los tres ejes radicales es también el ortocentro
      Si P es el X110 (foco de la parábola de Kiepert), los ejes radicales se corta en el punto Y, de primera coordenada

    a^2(3a^8 - 7a^6(b^2+c^2) + a^4(3b^4+13b^2c^2+3c^4) + a^2(3b^6-7b^4 c^2-7b^2c^4+3c^6) - (b^2-c^2)^2(2b^4+3b^2c^2+2c^4))

    con (6,9,13)-número de búsqueda en ETC: 4.824096254164793916494256409

  • miércoles, 24 de septiembre del 2014

    Propiedad geométrica del punto X2136

     The eigencenter of a triangle is defined in the Glossary and in TCCT, p. 192.
     Let T be a central triangle, and let U(T) be the unary cofactor triangle of T. Then T and U(T) are perspective, and their perspector is the eigencenter of T.
      In case the triangle is the cevian triangle of a point U = (x:y:z), in barycentric coordinates, the eigencenter is given by
    (a^2yz(a^2y^2z^2-b^2z^2x^2-c^2x^2y^2) : b^2zx(b^2z^2x^2-c^2x^2y^2-a^2y^2z^2) : c^2xy(c^2x^2y^2-a^2y^2z^2-b^2z^2x^2))
    This point is now named the eigentransform of U, denoted by ET(U).

    In particular:
    If X(57) is the perspector of the intouch triangle and excentral triangle, X(2136) is the eigentransform of X(57).


      Sean ABC un triángulo, IaIbIc el triángulo excentral, Φa la hipérbola circunscrita a IaIbIc y de asíntotas paralelas a las bisectrices interiores en B y C, y D el centro de Φa. De foma similar, de consideran las hipérbolas Φb y Φc y sus respectivos centros E y F.

    Las rectas DIa, EIb, FIc concurren en X2136.

    ( Mostrar/Ocultar figura )
      X(2136).png

      La ecuación de la hipérbola Φa es:
    b(a-b-c)cx^2 + 2a^2cy^2 + 2a^2bz^2 + a^2(a+b+c)yz + ab(a+b+c)zx + ac(a+b+c)xy=0.
    Y su centro D tiene coordenadas:
    (a(a^2+2ab+b^2+2ac-6bc+c^2) : -b(a^2+2ab+b^2-2ac+6bc-3c^2) : -c(a^2-2ab-3b^2+2ac+6bc+c^2))

      El punto donde concurren las rectas DIa, EIb, FIc es X2136: (a(a-b-c)(a^2+2a(b+c)+b^2-6bc+c^2) : ... : ... ).


      En general, si se toma el tiángulo anticeviano PaPbPc de un punto P=(p:q:r), y D es ahora el centro de la hipérbola circunscrita a PaPbPc con asíntotas paralelas a PPb y PPc, y lo mismo para E y F, las rectas DPa, EPb, FPc concurren en el punto:
    Q=(p(p-q-r)(p^2+2p(q+r)+q^2-6qr+r^2) : ... : ... ).

    Pares de centros del triángulo {P,Q} son:
    {X(1), X(2136)}, {X(2),X(2)}, {X(3), X(1498)}, {X(5), X(52)}, {X(9), X(2951)}, {X(10), X(72)}, {X(141), X(3313)}, {X(178), X(177)}, {X(1125), X(4065)}

  • viernes 18 de septiembre del 2014

    Hipérbolas circunscritas con asíntotas paralelas a cevianas



      Dados un triángulo ABC y un punto P, denotamos por Φa la hipérbola circunscrita a ABC y de asíntotas paralelas a PB y PC; sea Oa su centro (ver construccción en Pt_Pt_P). Similarmente, se consideran las hipérbolas Φb y Φc, y sus respectivos centros Ob y Oc.

      Las rectas AOa, BOb y COc son concurrentes, para cualquier punto P.

    ( Mostrar/Ocultar figura )
      HiperbolasCevianas.png

      Si las coordenadas baricéntricas de P son (u:v:w), la ecuación de la hipérbola Φa es u^2yz-v(u+w)zx-w(v+u)xy=0, y su centro es:

    Oa(u(u^2+u(v+w)+2vw) : v(u+w)(u+v-w) : w(v+u)(u-v+w))

      El punto Q de intersección de las rectas AOa, BOb y ACc es:

    (u(v+w)(u^2-(v-w)^2) : v(w+u)(v^2-(w-u)^2 : w(u+v)(w^2-(u-v)^2).

      Pares de puntos (P,Q) que son ambos centros del triángulo que figuran actualmente en ETC:

    (X(1),X(65))  (X(2),X(2))  (X(3),X(5562))  (X(4),X(64))  (X(6),X(1843))  (X(7),X(3062))  (X(8),X(3680))  (X(9),X(3059))  (X(10),X(4647))  (X(37),X(2667))



      Los centros Oa, Ob y Oc de las hipérbolas Φa, Φb y Φc están alineados si y solo si P está en la cúbica isotómica no-pivotal de raíz el baricentro y parámetro k=6 del triángulo medial.
      La envolvente de las rectas que contienen los centros, cuando P varía en esta cúbica, es la elipse inscrita de Stenier.

      La ecuación de la cúbica, en coordenadas baricéntricas respecto al triángilo medial, es x(y^2+z^2)+y(z^2+x^2)+z(x^2+y^2)+6xyz=0.

    ( Mostrar/Ocultar figura )
      HiperbolasCevianasCentrosAlineados.png



      Denotemos por Pa el cuarto punto de intersección de la hipérbola Φa con la circunferencia circunscrita. Similarmente se definen Pb, Pc.
      Si P(u:v:w), las coordenadas baricéntricas de Pa son:

    Pa = (1/(b^2(u+v)w-c^2v(u+w)) : -1/(c^2u^2+a^2w(u+v)) : 1/(b^2u^2+a^2v(u+w)).

      Si P está sobre la circunferencia circunscrita, las rectas AOa, BOb, COc son paralelas, de dirección perpendicular a la del diámetro que pasa por P.

    ( Mostrar/Ocultar figura )
      HiperbolasCevianasCuartoPuntoO(R).png

      Si P está sobre la cúbica isogonal no-pivotal Kjp= K024 = nK0+(X6,X6) , las rectas AOa, BOb, COc concurren en el conjugado isogonal P' de P, sobre dicha cúbica.

    ( Mostrar/Ocultar figura )
      HiperbolasCevianasCuartoPuntonK.png

      Tenemos así la siguiente caractización de la cúbica nK0+(X6,X6):

      Kjp = K024=nK0+(X6,X6) is the locus of point P such that AP and APa are isogonal lines with respect to the sidelines AB and AC, BP and BPb are isogonal lines with respect to the sidelines BC and BA, and CP and CPc are isogonal lines with respect to the sidelines CA and CB.



  • martes 16 de septiembre del 2014

    La isocúbica no-pivotal nK(X577,X2,X3)

    (Variación del Problema, propuesto por Dao Thanh Oai)

      Dado un triángulo ABC, sean DEF su triángulo antimedial y una recta δ de ecuación baricéntricas px+qy+rz=0, cuyos puntos de intersección con los lados EF, FD, DE son, respectivamente, A0(q-r:-p:p), B0(q:-p+r:-q), C0(-r:r:p-q).
      Sean M((q-r)(qr+μ) : (r-p)(rp+μ) : (p-q)(pq+μ)) y P((q-r)(qr+t) : (r-p)(rp+t) : (p-q)(pq+t) dos puntos sobre la recta δ.
      El inverso del punto A0 en la circunferencia C(M,P), de centro M que pasa por P, es:

    A1 = ((q-r) (-p^2q r + p q^2r + p q r^2 - p^2μ + p q μ + p r μ + q r μ + 2 t μ- t^2) :
    (p-r) (p^3r - p^2q r - p^2r^2 + p^2μ - p q μ - 2 p r μ - 2 t μ+ t^2) : -(p-q) (p^3q - p^2q^2 - p^2q r + p^2μ - 2 p q μ - p r μ - 2 t μ+ t^2)).

      Los inversos B1, C1 de los puntos B0, C0 se obtienen por permutación cíclica.

      Las afirmaciones siguientes surgen del estudio hecho en HG140914:

    ∗  Las rectas DA1, EA1, FA1 concurren en un punto N.

    ∗   Cuando el punto P queda fijo y M varía sobre la recta δ, el punto N describe la cónica δP, circunscrita a DEF tangente a δ en P.

    ∗ El lugar geométrico del centro Q de la cónica δP, cuando P se mueve sobre la recta δ es la cuártica Φδ con puntos singulares en los vértices de ABC y tangente a los lados de DEF en los puntos de corte con la recta δ, de ecuación:

      El cuarto punto común de la cuártica Φδ y la recta δ es el punto D0 de coordenadas baricéntricas:

    D0 = (qr(q-r) : pr(r-p) : pq(p-q)).

      Cuando la recta δ gira alrededor del circuncentro, el punto D0 recorre la isocúbica no-pivotal nK(X577,X2,X3).

    x (c^4 (a^2 + b^2 - c^2)^2 y^2 + b^4 (a^2 - b^2 + c^2)^2 z^2) + y (a^4 (b^2 + c^2 - a^2)^2 z^2 + c^4 (b^2 - c^2 + a^2)^2 x^2) + z (b^4 (c^2 + a^2 - b^2)^2 x^2 + a^4 (c^2 - a^2 + b^2)^2 y^2) + 2 (b^2 c^2 (b^2 - c^2)^2 + a^6 (b^2 + c^2) + a^2 (b^2 - c^2)^2 (b^2 + c^2) - a^4 (2 b^4 + b^2 c^2 + 2 c^4)) x y z =0
    ( Mostrar/Ocultar figura )
      ADGEOM1694AntiMedial.png

      La cúbica nK(X577,X2,X3) pasa por X3 (doble) y por X4240 (que es el cuarto punto D0, cuando δ es la recta de Euler). Sus asíntotas son paralelas a los lados de ABC, por lo que delimitan un triángulo homotético a ABC con centro de homotecia X577 (polo de la cúbica). nK(X577,X2,X3) no figura actualmente en el catálogo CTC, de Bernard Gibert

      Cuando δ es la recta de Euler, la cuártica Φδ y la circunferencia circunscrita tienen un sólo punto común X925 (aparte de A, B, C), con tangente común. En este caso, la cuártica contiene a X648, tripolo de la recta de Euler.

  • domingo 14 de septiembre del 2014

    Inversos de los puntos de corte de una recta y un triángulo

    ( ADGEOM #1698, Dao Thanh Oai)

    Problem , Dao Thanh Oai

    Let ABC be a triangle, let three points A0, B0, C0 lie on BC, CA, AB such that A0, B0, C0 are collinear. Let circle (P) which center P lie on the line A0B0C0. Denote A1, B1, C1 are the inverse (or inversion, or reflection) of A0, B0, C0 in (P), resp. Show that AA1, BB1, CC1 are concurrent.


      Dado un triángulo ABC y una recta δ de ecuación baricéntricas px+qy+rz=0, los puntos de intersección con los lados BC, CA, AB son, respectivamente, A0(0:-r:q), B0(r:0:-p), C0(-q:p:0).
      Sean M((q-r)(qr+μ) : (r-p)(rp+μ) : (p-q)(pq+μ)) y P((q-r)(qr+t) : (r-p)(rp+t) : (p-q)(pq+t) dos puntos sobre la recta δ.
      El inverso del punto A0 en la circunferencia C(M,P) de centro M que pasa por P es:

    A1 = ((q-r)(qr+t)(qr-t+2μ) : (r-p)(pqr^2-t^2+prμ+qrμ+2tμ) : (p-q)(pq^2r-t^2+pqμ+qrμ+2tμ)).

      Los inversos B1, C1 de los puntos B0, C0 se obtienen por permutación cíclica.
      Las rectas AA1, AA1, AA1 concurren en el punto:

    N = ((q-r)/(p^2qr+p(q+r)μ-t(t-2μ)) : ... : ...).

    ( Mostrar/Ocultar figura )
      ADGEOM1694.png

      Cuando el punto P queda fijo y M varía sobre la recta δ, el punto N describe la cónica δP, circunscrita a ABC tangente a δ en P:

    δP: p(q-r)^2+(qr+t)^2yz + q(r-p)^2+(rp+t)^2zx + r(p-q)^2+(pq+t)^2)xy=0.

      El centro de δP es el punto:

    Q = (p(q-r)(qr+t)^2 (p^2qr(q+r)-pqr(q^2-2qr+r^2-4t)+(q+r)t^2) : ... : ...).


      El lugar geométrico del centro de la cónica δP, cuando P se mueve sobre la recta δ es la cuártica Φδ con puntos singulares en los vértices del triángulo medial y tangente a los lados de ABC en los puntos de corte con la recta δ, de ecuación:

    p^2x^4 + (-2p^2+2pq)x^3y + (p^2-4pq+q^2)x^2y^2 + (2pq-2q^2)xy^3 + q^2y^4 + (-2p^2+2pr)x^3z + (2p^2-2qr)x^2yz + (2q^2-2pr)xy^2z + (-2q^2+2qr)y^3z + (p^2-4pr+r^2)x^2z^2 + (-2pq+2r^2)xyz^2 + (q^2-4qr+r^2)y^2z^2 + (2pr-2r^2)xz^3 + (2qr-2r^2)yz^3 + r^2z^4=0.


    ( Mostrar/Ocultar figura )
      ADGEOM1694N1.png

      El cuarto punto común de la cuártica Φδ y la recta δ es el punto D0 de coordenadas baricéntricas:

    D0 = (p(q^2-r^2) : q(r^2-p^2) : r(p^2-q^2)).

      Si D1 es el inverso de D0 respecto a C(M,P) la recta ND1 pasa por un punto fijo N1 sobre la cónica δP, cuando P queda fijo y M varía.

      Las coordenadas de D1 y N1 son

    D1 = ((q-r)(p(q+r)(qr+μ)+(qr+t)(qr-t+2μ)) : ... : ...),
    N1 = (qr(q-r)(qr+μ)/(q+r) : rp(r-p)(rp+μ)/(r+p) : pq(p-q)(pq+μ)/(p+q)).

      El lugar geométrico del punto N1, cuando P, es la cónica inscrita en ABC tangente a δ en D0.

    Esta cónica inscrita es la envolvente de las rectas PN1 y su perspector es:

    P1 = (q^3r^3/(q^2-r^2) : r^3p^3/(r^2-p^2) : p^3q^3/(p^2-q^2)).



    CASOS PARTICULARES:

     • Cuando δ es la recta de Euler, la cuártica Φδ la corta en X1650, que es la posición del centro de δP cuando P tiende al punto del infinito de la recta de Euler. El punto X136 también está en Φδ, que es el centro de δP cuando P es el ortocentro. En X136 la circunferencia de Euler y la cuártica asociada a la recta de Euler son tangentes, por lo que no tienen más puntos comunes que éste y los puntos medios de los lados.
      Cuando P es el ortocentro la cónica δP es la hipèrbola equilátera de centro X136, pasando por los centros de ETC con índices 4, 93, 225, 254, 264, 393, 847, 1093, 1105, 1179, 1217, 1300, 1826. Su ecuación es:

    (b^2-c^2)yz/SA + (c^2-a^2)zx/SB + (a^2-b^2)xy/SC = 0.

    Su perspector es el centro radical de las circunferencias circunscrita, de los nueve puntos y de Taylor, X2501.
    ( Mostrar/Ocultar figura )
      ADGEOM1694Euler.png


     • Cuando δ es la eje de Brocard, la cuártica Φδ lo corta en el punto:

    (a^2(b^2-c^2)^2(b^2+c^2-a^2)(a^2(b^2+c^2)-b^4-c^4) : ... : ...)

    (con (6,9,13)-número de búsqueda en ETC: 2.9801797228438530320754966389) que es la posición del centro de δP cuando P tiende al punto del infinito del eje de Brocard.
    El punto X1084 también está en Φδ, que es el centro de δP cuando P es el simediano.
      Cuando P es el simediano la cónica δP es la hipèrbola circunscrita que pasa por el baricentro y simediano, además de los centros de ETC con índices 25, 37, 42, 111, 251, 263, 308, 393, 493, 494, 588, 589, 694, 941, 967, 1169, 1171, 1218, 1239, 1241, 1383, 1400, 1427, 1880, 1976, 1989, 2054, 2165, 2248, 2350, 2395, 2433, 2963, 2981, 2987, 2998, 3108, 3228, 3444, 3457, 3458, 3572, 5638, 5639. Su ecuación es:

    a^2(b^2-c^2)yz + b^2(c^2-a^2)zx + c^2(a^2-b^2)xy = 0.

    Su perspector es el centro radical de las circunferencias circunscrita, de los nueve puntos y de Taylor, X2501.
    ( Mostrar/Ocultar figura )
      ADGEOM1694Brocard.png


     • Cuando δ es una recta que pasa por X5 (centro de la circunferencia de los nueve puntos), el cuarto punto D0 de interseccion de δ y la cuártica Φδ recorre la cúbica nK(X577,X2,X3) no-pivotal del triángulo medial, que pasa por X5 (singular) y por X1650 (X4240 del triángulo medial). Las asíntotas de esta cúbica son paralelas a los lados de ABC. El centro de homotecia de ABC y el triángulo delimitado estas las asíntotas es X233, cociente ceviano del baricentro y el centro de la circunfencia de los nueve puntos del triángulo medial.
    ( Mostrar/Ocultar figura )
      ADGEOM1694X5.png


  • viernes 12 de septiembre del 2014

    "Porismcircles" y ejes radicales



    The porismcircles, Floor van Lamoen

    The midcircle (circle of antisimilitude) of two circles is the circle that swaps these two circles by inversion.
    Inversion in the midcircle of incircle and circumcircle maps the sides of a triangle to a triple of circles tangent to the circumcircle and intersecting on the incircle. I call them porismcircles.


    ( Mostrar/Ocultar figura )
      porismcircles.png

      La circunferencia de antisimilitud ("midcircle") de dos circunferencias es una circunferencia respecto a la cual las dos son mutuamente inversas.
      Sean ABC un triángulo y Γ la circunferencia de antisimilitud de sus circunferencias inscrita y circunscrita que tiene centro en el centro de homotecia interior, X55, de estas dos circunferencias.
      Se denota por (Ka) la circunferencia inversa del lado BC en la inversión respecto a Γ, por (Ja) la circunferencia que pasa por B y C y es tangente a la circunferencia inscrita y por ea el eje radical de (Ka) y (Ja). Similarmente se toman los ejes radicales eb y ec.

      El triángulo delimitado por los ejes radicales ea, eb y ec es perspectivo con ABC con centro de perspectividad el punto de coordenadas baricentricas:

    (a/(a^4-2a^2b c-2a^3(b+c)+2a(b^3+c^3)-(b-c)^4) : ... : ...)

    con (6,9,13)-número de búsqueda en ETC: 2.2548851539334829652783884
    ( Mostrar/Ocultar figura )
      HG120914.png


  • miércoles 10 de septiembre del 2014

    Lugares geométricos asociados a rectas de Euler

    ( Anopolis #1834, Antreas P. Hatzipolakis)

      Sean ABC un triángulo, P un punto y A'B'C' su triángulo ceviano.
    Se denota por:
    Pab y Pac las paralelas por A a PB y PC, resp.
    Ab, Ac las proyecciones ortogonales de A' sobre Pab y Pac, resp.
    La, Lb, Lc las rectas de Euler de los triángulos AAbAc, BBcBa, CCaCb, resp.

    Si (u:v:w) son las coordenadas baricéntricas,

    Ab = (w(u+v+w)((-b^2+c^2)u+a^2(u+2w)) :
    (u+w)(-w(a^2(u-v+w)+b^2(-u+v+w))+c^2(w(v+w)+u(2 v+w))) :
    w(w(a^2(u-v+w)+b^2(-u+v+w))-c^2(w(v+w)+u(2v+w))) )

    Las coordenadas de Ac se obtienen partir de las de Ab, permutando sus dos últimas componentes y permutando "b" por "c" y "v" por "w".

      El lugar geométrico de los puntos P tales que las rectas La, Lb, Lc son paralelas es una séxtica con puntos dobles en los vértices de ABC.

    ( Mostrar/Ocultar figura )
      Anopolis1834lg6.png

      El lugar geométrico de los puntos P tales que las rectas La, Lb, Lc son concurrentes una séptica con puntos dobles en los vértices de ABC y que pasa por los centros X3, X4, X13, X14, X20.

    ( Mostrar/Ocultar figura )
      Anopolis1834lg7.png

    En particular, cuando P es el ortocentro (A'B'C' triángulo órtico) el punto donde concurren las rectas La, Lb, Lc es X973 (1st Ehrmann Point, nombre propuesto en Hyacinthos #3694, Antreas P. Hatzipolakis).
    
    
    Dear Jean-Pierre,
    
    [APH]:
     So, we have the Theorem:
    
     Let AA', BB', CC' be the three altitudes of ABC, and
     Let Ab, Ac be the orth. proj. of A' on AB, AC resp.
     Bc, Ba " B' BC, BA
     Ca, Cb " C' CA, CB
    
     Then the Euler lines of A'B'C', A'AbAc, B'BcBa, C'CaCb are
     concurrent.
    
     Which is the point of concurrence [a point lying on the Euler
     line of the orthic triangle A'B'C' of ABC]? Is it in ETC?
    
     
     [JPE]:
      X(442) of the orthic triangle (not in ETC, I think)
    
    [JPE]:
    Let's name them temporarily (or not!) as:
     
     Triangles A'AbAc, B'BcBa, C'CaCb = Orthiac Triangles
     
    [APH]:
    
      A PS for Clark Kimberling:
     
     Dear Clark,
     
     If the above  point of concurrence of the Euler lines
     of the Orthiacs (ie X442 of Orthic)  is not already
     in your list, then you may of course include them, but with these 
    name:
     1st Ehrmann Point.
     

      El lugar geométrico de los puntos P tales que los triángulos ABC y el delimitado por las rectas La, Lb, Lc son ortológicos es una curva algebraica de grado ocho, con puntos triples en los vértices de ABC y pasa por el foco de la parábola de Kiepert, X110..

    ( Mostrar/Ocultar figura )
      Anopolis1834lg8Orth.png


  • domingo 7 de septiembre del 2014

    El centro X4240 y cuatro rectas de Euler

    ( ADGEOM #1629, Dao Thanh Oai)

    Problem. Let ABC be a triangle, let the Euler line of the triangle ABC meets the sidelines BC, CA, AB at A1,B1,C1 respectively. Let Ab,Ac are reflection of A in B1,C1 respectively. Let Ba,Bc are reflection of B in A1,C1 respectively. Let Ca,Cb are reflection of C in A1,B1 respectively. Show that the Euler lines of four triangles ABC, AAbAc, BBcBa, CCaCb are concurrent.


      En coordenadas baricéntricas, usando la notación de Conway, se tiene:

    Ab = ((a^2-c^2) SB : 0 : -2(b^2-c^2)SA)
    Ac = ((a^2-b^2)SC : -2(c^2-b^2)SA : 0)

      Las coordenadas de los puntos Bc, Ba, Ca, Cb se obtienen permutando cíclicamente las de Ab y Ac.

      La ecuación de la recta de Euler del triángulo AAbAc es:

    2 (b^2-c^2)^2SA^2x + (a^2-b^2)(a^2-c^2)SBSC(y + z) = 0.

    Por permutación cíclica resultan las ecuaciones de las rectas de Euler de los triángulos BBcBa y CCaCb.
    Estas tres rectas concurren el centro X4240:

    ( (2a^4-a^2(b^2+c^2)-(b^2-c^2)^2)/((b^2-c^2)SA) : ... : ...).

    ( Mostrar/Ocultar figura )
      ADGEOM1625.png


      Los seis puntos Ab, Ac, Ba, Bc, Ca, Cb están en una misma cónica, tangente en X4240 a la recta de Euler y que pasa por X648 (tripolo de la recta de Euler). Estos dos puntos son antipodales en esta cónica.

      La ecuación de la cónica es:
    (a^2-b^2)(a^2-c^2)SBSC(8(b^2-c^2)^2SA^2x^2 + (4a^8- 4 a^6 (b^2+c^2) + a^4 (-5b^4+14b^2c^2-5c^4) + 6 a^2(b^2-c^2)^2(b^2+c^2) - (b^2-c^2)^2(b^4+ 6b^2c^2 +c^4)) y z) + cicl. = 0.

  • lunes 1 de septiembre del 2014

    Involución sobre la recta de Euler y centros ortológicos

    ( Anopolis #1822, Antreas P. Hatzipolakis)

      Sean ABC un triángulo, H su ortocentro y P un punto. Se denota por:
    Pa, Pb, Pc las reflexiones de P en los lados BC, CA, AB, respectivamente.
    Pab, Pac las reflexiones de Pa en BH, CH, respectivamente.
    Pbc, Pba las reflexiones de Pb en CH, AH, respectivamente.
    Pca, Pcb las reflexiones de Pc en AH, BH, respectivamente.
    A'B'C' el triángulo delimitado por las rectas PabPac, PbcPba y PcaPcb.

      Los triángulos ABC y A'B'C' son ortológicos si y solo si P está sobre la recta de Euler.
      El centro de ortología de A'B'C' con respecto a ABC es P; el centro de ortología de ABC con respecto a A'B'C' es un punto Q sobre la recta de Euler.
      La correspondencia P ↦ Q es una involución de centro en el inverso del ortocentro respecto a la circunferencia circunscrita de ABC.

    ( Mostrar/Ocultar figura )
      Anopolis1822.png
    En coordenadas baricéntricas si P=(x:y:z), la expresión del lugar geométrico de los puntos P tales que los triángulos ABC y A'B'C' sean ortológicos es:

    (x+y+z)*
    ((b^2-c^2)(b^2+c^2-a^2)x+(c^2-a^2)(c^2+a^2-b^2)y+(a^2-b^2)(a^2+b^2-c^2)z)*
    (a^2yz+b^2zx+c^2xy)*
    (a^4b^2c^2(b^2+c^2-c^2)^2x^2 + a^2(b^2+c^2-a^2)(c^2+a^2-b^2)(a^2+b^2- c^2)(a^4-2a^2(b^2+c^2)+b^4-b^2c^2+c^4 )yz+ ...)=0.

    • El primer factor representa la recta del infinito, para cuyos puntos la construcción no tiene sentido.
    • El segundo factor es la ecuación de la recta de Euler.
    • El tercer factor es la ecuación de la circunferencia circunscrita. En este caso, las tres rectas PabPac, PbcPba y PcaPcb concurren en P. Las perpendiculares a ellas desde los vértices de ABC concurren el punto antipodal de P.
    • Y el último fractor, igualado a cero, es la ecuación de una cónica imaginaria, degenerada en el producto de dos rectas imaginarias conjugadas, con punto de intersección X186 (inverso del ortocentro respecto a la circunferencia circunscrita).


      Si P es un punto sobre la recta de Euler que divide al segmento X3X4 en la razón t, el correspondiente centro de ortología Q, del triángulo ABC respecto a A'B'C', divide a X3X4 en la razón:

    t' = -(a^2b^2c^2)/((a^2+b^2-c^2)(a^4-2a^2b^2+b^4-c^4)t).


    Así, la correspondencia P ↦ Q (t↦t') es una involución sobre la recta de Euler.

    P=(a2SA+2SBSC t:...:...) ↦ Q=(a2b2c2SBSC+4a2SA2SBSC t:...:...).


    Los puntos fijos corresponden a los valos de t:

    ±abc/((-a^2+b^2+c^2)(a^2-b^2+c^2)(a^2+b^2-c^2))½,

    que serán reales si el triángulos ABC es acutángulo (involución hiperbólica) e imaginarios conjugados cuando es obtusángulo.
      El centro de esta involución (punto medio de los fijos) es X186, inverso del ortocentro respecto a la circunferencia circunscrita:

    Anopolis1822prod.png
    Anopolis1822prod2.png



      Pares de puntos conjugados en esta involución, que son centros del triángulo que figuran actualmente en la Enciclopedia ETC, son los que corresponden a los índices:
    {2, 378}, {3, 4}, {5, 3520}, {20, 24}, {25, 376}, {28, 3651}, {30, 186}, {403, 2071}, {550, 3518}, {631, 1593}, {1006, 4219}, {1113, 1114}, {1597, 3524}, {1598, 3528}, {3090, 3516}, {3515, 3529}, {4220, 4227}, {4221, 4231}.

  • sábado 30 de agosto del 2014

    Problema de Apolonio relativo a las circunferencias exinscritas

    ( Forum Geometricorum Volume 10 (2010) 35–40, Boris Odehnal)

    Se cree que fue Isaac Newton el primero
    que resolvió, por medio de la regla y el compás,
    el problema de encontrar la circunferencia tangente
    a otras tres circunferencias.



      Dado un triángulo ABC, las ocho circunferencias tangentes a las circunferencias exinscritas son:
    • La circunferencia de Apolonio (Ap) de ABC
    • La circunferencia de Euler (N)
    • Los tres lados del triángulo ABC, circunferencias con centro en la recta del infinito
    • La circunferencia (Ja) que encierra la A-exinscrita (Ia) y toca a las otras dos por el exterior
    • La circunferencia (Jb) que encierra la B-exinscrita (Ib) y toca a las otras dos por el exterior
    • La circunferencia (Jc) que encierra la C-exinscrita (Ic) y toca a las otras dos por el exterior
    ( Mostrar/Ocultar figura )
      Exinscritas Problema de Apolonio_0.png

      La circunferencia (Ja) es la inversa del lado BC en la inversión respecto a la circunferencia radical de las exinscritas (Re). En esta inversión (Ia) se transforma en sí misma.
      Las circunferencias (Ia) y (Ja) son tangentes en el punto Aa de coordenadas baricéntricas:

    Aa = ((a+b-c)(a-b+c)(b+c)^2 : -c^2(a-b+c)(a+b+c) : -b^2(a+b-c)(a+b+c)).

    Permutando cíclicamente estas coordenadas obtenemos las de los puntos de tangencia Bb de (Ib) y (Jb), y Cc de (Ic) y (Jc).

      El centro de la circunferencia (Ja) es:
    Ja = ( * : 1/(a^3(b^2+2bc+2c^2)+ a^2(b^3+b^2c+2bc^2+2c^3) + ab(-b^3-2b^2c+bc^2+2c^3) - b^2(b-c)(b+c)^2) : 1/(a^3(2b^2+2bc+c^2) + a^2(2b^3+2b^2c+bc^2+c^3) + ac(2b^3+b^2c-2bc^2-c^3) - c^2(c-b)(b+c)^2)).

    Por permutación cíclica, se deducen las coordenadas de los centros Jb y Jc de las circunferencias (Jb) y (Jc).

      De las expresiones de las coordenadas obtenidas se deduce que:

    Los triángulos ABC y AaBbCc son perspectivos con centro de perspectividad en X3596 (1st Odehnal Point):
    ((-a+b+c)/a^2 : (a-b+c)/b^2 : (a+b-c)/c^2).

    Los triángulos ABC y JaJbJc son perspectivos con centro de perspectividad en X3597 (2nd Odehnal Point):
    (1/(a^5+a^4(b+c) - a^3(b-c)^2-a^2(b^3+b^2c+bc^2+c^3) - 2abc(b^2+bc+c^2) - 2b^2c^2(b+c)) : .. : .. ).

    *************************************



     Las tangentes en los puntos de contacto Aa, Bb y Cc de las circunferenicias (Ia), (Ib) y (Ic) con las (Ja), (Jb) y (Jc) delimitan un triángulo T1T2T3 perspectivo con ABC. El centro de perspectividad T tiene coordenadas baricéntricas:

    T = ((b+c)/(a^3(b+c) + a^2bc - a(b-c)^2(b+c) - bc(b+c)^2) : ... : ...)

    con (6,9,13)-número de búsqueda en ETC: 3.2570453258373956346802382
    ( Mostrar/Ocultar figura )
      Exinscritas Problema de Apolonio_3.png
    La ecuación de la tangente común en Aa a las circunferencias (Ia) y (Ja) es:

    bc(a+b+c)x + b(a+b-c)(b+c)y + c(a-b+c)(b+c)z = 0.



    *************************************



      Sean Ab y Ac los puntos de contacto de la circunferencia (Ja) con (Ib) y (Ic), respectivamente. Sus coordenadas son:

    Ab = ((-a+b+c)(a+b-c)(b+c)^2 : c^2(a-b-c)(a+b+c) : (b(b+c)+a(b+2c))^2),
    Ac = ((a-b-c)(a-b+c)(b+c)^2, : -(c(b+c)+a(2b+c))^2 : b^2(-a+b+c)(a+b+c)).

    Procediendo cíclicamente, de deducen las coordenadas de los puntos Bc y Ba de contacto de la circunferencia (Jb) con (Ic) y (Ia), respectivamente; y las de los puntos Ca y Cb de contacto de la circunferencia (Jc) con (Ia) y (Ib), respectivamente.

     Las rectas AbAc, BcBa y CaCb delimitan un triángulo P1P2P3 perspectivo con ABC. El centro de perspectividad es:

    P = ( (b+c)(b+c-a)/(a^2(b^2+3bc+c^2) + a(b^3+b^2c+bc^2+c^3) + bc(b+c)^2) : ... : ... )

    con (6,9,13)-número de búsqueda en ETC: 4.57674437778156865008060726
    ( Mostrar/Ocultar figura )
      Exinscritas Problema de Apolonio_1.png
    La ecuación de la recta AbAc es:
    (b^3+2b^2c+2bc^2+c^3+a(b^2+3bc+c^2))x - c(b+c-a)(b+c)y - b(b+c)(b+c-a)z = 0.

     Las rectas BaCa, CbAb y AcBc delimitan un triángulo Q1Q2Q3 perspectivo con JaJbJc. El centro de perspectividad Q tiene primera coordenada baricéntrica:

    a^10 (b + c)^3 + a^9 (b + c)^2 (5 b^2 - b c + 5 c^2) -
      a^8 (5 b^5 + 28 b^4 c + 42 b^3 c^2 + 42 b^2 c^3 + 28 b c^4 + 5 c^5) -
    a^7 (33 b^6 + 101 b^5 c + 182 b^4 c^2 + 243 b^3 c^3 + 182 b^2 c^4 +
      101 b c^5 + 33 c^6) -
    a^6 (21 b^7 + 86 b^6 c + 257 b^5 c^2 + 411 b^4 c^3 + 411 b^3 c^4 +
      257 b^2 c^5 + 86 b c^6 + 21 c^7) -
    a^5 (-27 b^8 - 27 b^7 c + 83 b^6 c^2 + 290 b^5 c^3 + 454 b^4 c^4 +
     290 b^3 c^5 + 83 b^2 c^6 - 27 b c^7 - 27 c^8) +
    a^4 (29 b^9 + 104 b^8 c + 160 b^7 c^2 - 53 b^6 c^3 - 372 b^5 c^4 -
      372 b^4 c^5 - 53 b^3 c^6 + 160 b^2 c^7 + 104 b c^8 + 29 c^9) +
    a^3 (b^2 - c^2)^2 (b^6 + 73 b^5 c + 182 b^4 c^2 + 239 b^3 c^3 +
      182 b^2 c^4 + 73 b c^5 + c^6) -
    a^2 (b - c)^2 (b + c)^3 (4 b^6 - 11 b^5 c - 49 b^4 c^2 -
      99 b^3 c^3 - 49 b^2 c^4 - 11 b c^5 + 4 c^6) -
    a b c (b - c)^2 (b + c)^4 (8 b^4 - 21 b^3 c - 9 b^2 c^2 -
      21 b c^3 + 8 c^4) -
    b^2 c^2 (b - c)^2 (b + c)^5 (4 b^2 - 11 b c + 4 c^2)

    con (6,9,13)-número de búsqueda en ETC: -1.35127668285956378285916015
    ( Mostrar/Ocultar figura )
      Exinscritas Problema de Apolonio_2.png


      La circunferenica (J) tangente a las (Ja), (Jb) y (Jc), abarcándolas, es tangente a la circunferencia de Apolonio de ABC en el centro del triángulo X3030.

    ( ADGEOM #1541)
    ( Mostrar/Ocultar figura )
      X(3030).png
    Si D es el punto de contacto de la circunferencia inscrita con el lado BC, el punto de tangencia Sa de las circunferencias (J) y (Ja) es el inverso de D en la inversión respeto a la circunferencia (Re) radical de las exinscritas; sus coordenadas son:

    Sa = (-(b+c)(a^2(b+c)-2a(b-c)^2+(b-c)^2(b+c)) : (a(2b-c)+(b-c)c)(a(2b-c)+c(b+c)) : (a(b-2c)+b(b-c))(a(b-2c)-b(b+c))).

    Similarmente se obtienen las coordenadas de los puntos Sb y Sc de contacto de (J) con (Jb) y (Jc), respectivalente.

    La ecuación de la circunferencia (J), inversa de la circunferencia inscrita a ABC mediante la inversión respecto a la circunferencia radical de las circunferencias exinscritas, es;

    k(a,b,c)(a^2yz+b^2zx+c^2xy)-(x+y+z)(f(a,b,c)x+f(b,c,a)y+f(c,a,b)z)=0

    donde
    k(a,b,c) = 4(a^2(b+c)+a(b^2-7bc+c^2)+bc(b+c)) y
    f(a,b,c) = a^4(b+c) - a^3(b^2-5bc+c^2) - a^2(b^3+c^3) + a(b^4-7b^3c+16b^2c^2-7bc^3+c^4) + bc(b+c)^3.

    El centro de esta circunferencia es el punto J de coordenadas:

    J = ( a(a^2(b+c)-3bc(b+c)+a(b^2+bc+c^2)) : ... : ...),

    que tiene números de búsqueda en ETC (166.495388181518, 12.6773138209277, -81.9799627086661)

      El triángulo tangencial de SaSbSc es perspectivo con ABC, con centro de perpectividad el punto de Spieker X10.



  • martes 26 de agosto del 2014

    Puntos de Feuerbach y circunferencia de Apolonio

    ( ADGEOM #1525, Tran Quang Hung)

    a Clara, por su 13 "cumple", con 9 días de retraso



      Sean ABC un triángulo, (Ia), (Ib), (Ic) sus circunferencia exinscritas y Fa, Fb, Fc los puntos de tangencia con la circunferencia de los nueve puntos (puntos de Feuerbach).
      Se denota por (Ka) la circunferencia (distinta de la circunferencia de los nueve puntos) tangente a (Ia) y que pasa por Fb, Fc. Similarmente se definen las circunferencias (Kb) y (Kc).

    La construcción de la circunferencia (Ka) es el problema de Apolonio PPC.

    Los pasos para la construcción de (Ka) son los siguientes:
    • La recta que une los puntos Fb y Fc es el eje radical de todas las circunferencias que pasan por ellos.
    • A continuación dibujamos una circunferencia auxiliar que pase por los puntos Fb y Fc y que corte a la circunferencia dada: tomamos la circunferencia de los nueve puntos. Trazamos la tangente común a ésta y a (Ia). En la intersección de esta tangente con la recta FbFc está el centro radical La.
    • La otra tangente desde La a la circunferencia (Ia) es también tangente a la circunferencia (Ka), que estamos buscando.
    ( Mostrar/Ocultar figura )
      ADGEOM1525a.png


    Si consideramos los puntos Lb y Lc necesarios para construir las circunferencias (Kb) y (Kc), de forma similar a la construcción de (Ka), se verifica:

    Los puntos La, Lb y Lc están en una misma recta y su tripolo L tiene coordenadas baricéntricas:
          ((b+c)/(a^2+bc) : (a+c)/(b^2+ac) : (a+b)/(ab+c^2))

    con (6,9,13)-número de búsqueda en ETC: 3.4313407904038455162362

      Se conoce como circunferencia de Apolonio del triángulo ABC a la círcunferencia que toca sus tres circunferencias exinscritas y las abarca (Kimberling.-TCCT 1998, p. 102).
      La recta que une el centro radical (punto de Spieker, incentro del triángulo medial) de las circunferencias exinscritas con el punto de Feuerbach Fa, vuelve a corta a la circunferencia A-exinscrita en el punto F'a de tangencia con la circunferencia de Apolonio de ABC. Similarmente, se determinan los puntos de contacto F'b y F'c.

      Los ejes radicales de la circunferencia de Apolonio de ABC y cada una de las circunferencias (Ka), (Kb), (Kc) delimitan un triángulo perspectivo con ABC. El centro de perspectividad es X6042:
       (a(b+c)^2(a(b+c)+b^2+c^2)^2 : b(c+a)^2(b(b+a)+c^2+a^2)^2 : c(a+b)^2(c(a+b)+a^2+b^2)^2).

    ( Mostrar/Ocultar figura )
      ADGEOM1525b.png
      Las ecuación baricéntrica del eje radical de la circunferencia de Apolonio de ABC y la circunferencia (Ka) es (las ecuaciones de los otros dos ejes radicales se obtienen por permutación cíclica):

    bc(a+b+c)(4a^2bc+a(b+c)^3+(b^2+c^2)^2)x + ac(b+c)(b^2+c^2+a(b+c))^2y + ab(b+c)(b^2+c^2+a(b+c))^2z = 0.

    X(6042) in ETC

    X(6042) = Perspector of Montesdeoca-Hung Triangle and ABC
    Let (Ap) be the Apollonius circle, and let (KA), (KB), (KC) be the circles described at X(5973) in association with the Hung-Feuerbach circle at X(5974). Let LA be the radical axis of (Ap) and (KA), and define LB and LC cyclically. The lines LA, LB, (LC form a triangle T (here named the Montesdeoca-Hung triangle) that is perspective to ABC, and the perspector is X(6042). (Tran Quang Hung, ADGEOM #1506; Angel Montesdeoca, ADGEOM #1525, August 24, 2014)


    X5975, Touchpoint Hung-Feuerbach circle and Apollonius circle (Peter Moses, ADGEOM #1508)
    ( Mostrar/Ocultar figura )
      ADGEOM1525c.png
      Para construir la circunferencia que toca a las tres circunferencias (Ka), (Kb), (Kc) y las abarca (Hung-Feuerbach circle), consideramos la recta q que contiene a los tres centros externos, Qa, Qa, Qa de homotecia de las circunferencias (Ka), (Kb), (Kc).
    Sean Pa, Pb, Pc los polos de la recta q respecto a las circunferencias (Ka), (Kb), (Kc). Si R es el centro radical de las circunferencias (Ka), (Kb), (Kc), las semirrectas RPa, RPb, RPc las cortan en los puntos de tangencia con la circunferencia de Hung.

  • miércoles 13 de agosto del 2014

    Descripciones geométricas del centro del triángulo X542

    ( Anopolis #20, Antreas P. Hatzipolakis)

    X(542) in ETC

    Let ABC ge a triangle and X(98) the Tarry point (the point of intersection, other than A, B, and C, of the circumcircle and Kiepert hyperbola), then X(542) is the point at infinity defined by the direction of the vector AX(98) + BX(98) + CX(98).

    &ensp  Sean ABC un triángulo, P un punto y PaPbPc el triángulo pedal de P. Consideremos P'a, P'b, P'c los puntos sobre las rectas PPa, PPb, PPc tales que:

    PP'a / PPa = PP'b / PPb = PP'c / PPc = t, (t≠1)

    La paralela por P'a a BC interseca a BA, CA en Ba, Ca, respectivamente.
    La paralela por P'b a BA interseca a CB, AB en Cb, Ab, respectivamente.
    La paralela por P'c a AB interseca a AC, BC en Ac, Bc, respectivamente.

    El lugar geométrico de los puntos P tales que las mediatrices de BaCa, CbAb, AcBc son concurrentes es la recta que pasa por el baricentro y el simediano.


    ( Mostrar/Ocultar figura )
      Anopolis20a.png
    La ecuación baricéntrica de la mediatriz da de BaCa (las de las otras mediatrices db y dc se obtiene por permutación cíclica) es, si P=(u:v:w):

    (b^2-c^2)(tu+v+w)x + ((b^2-c^2)(t-1)u+a^2(u+v+w))y + ((b^2-c^2)(t-1)u-a^2(u+v+w))z = 0.

    La condición para que las tres mediatrices sean concurrentes es:

    (b^2-c^2)u + (c^2-a^2)v + (a^2-b^2)w = 0.

    Es decir, el lugar geométrico es la recta que pasa por el baricento G(1:1:1) y el simediano K(a^2:b^2:c^2).


    Si P es el punto Pt de intersección de las rectas GK y δt, el correspondiente punto Qt de intersección de las mediatrices da, db, dc, describe la parábola que contiene al baricentro, circuncentro y simediano y con punto en el infinito X542.

    ( Mostrar/Ocultar figura )
      Anopolis20c.png

    Cuando t varía y P queda fijo en la recta GK , el lugar geométrico que describe el punto Q de intersección de las mediatrices da, db, dc, es una recta δP que pasa por el circuncentro, X3.

    ( Mostrar/Ocultar figura )
      Anopolis20b.png

    Si P es un punto sobre GK de coordenadas (a^2(1-ξ)+ξ : b^2(1-ξ)+ξ : c^2(1-ξ)+ξ), la ecuación de la recta δP es:

    (b^2-c^2)((a^2-b^2-c^2+b^2c^2)ξ-b^2c^2)x + ... = 0.

    La recta δP contiene al punto P solamente si éste es el baricentro o el simediano.

    Cuando P varía en la recta GK, el lugar geométrico que describe el pie de la perpendicular, trazada por él a la recta δP, es una cúbica circunscrita al triángulo medial, que contiene a los centros X2, X3 (doble), X6, X542, X5108 y con asíntota pasando por X5099 (descrito por Seiichi Kirikami en Anopolis #846) y dirección la del punto del infinito X542.

    ( Mostrar/Ocultar figura )
      Anopolis20d.png


  • miércoles 06 de agosto del 2014

    Propiedad del centro del triángulo X427

    ( ADGEOM #1469, Tran Quang Hung)

    X(427) in ETC

    Let LA be the line tangent to the nine-point circle at the midpoint of segment BC, and define LB and LC cyclically. The triangle formed by the lines LA, LB, LC is homothetic to the orthic triangle, and the center of homothety is X(427). (Randy Hutson, 9/23/2011)

      Sean ABC un triángulo, A1 y A2 los puntos de intersección de la circunferencia circunscrita con la circunferencia A-exinscrita, Fa el punto de tangencia de la circunferencia de los nueve puntos y la circunferencia A-exinscrita.
      Sea Γa la circunferencia, distinta de la A-exinscrita, que pasa por A1 y A2 es tangente a la circunferencia de los nueve puntos y La el punto de contacto.
      Similarmente se definen los puntos Fb, Fc, Lb, Lc.
      Las rectas FaLa, FbLb, FcLc concurren en X427, sobre la recta de Euler.

    ( Mostrar/Ocultar figura )
      ADGEOM1469.png
    El centro radical Ea de las circunferencia que pasan por A1 y A2 es el punto de intersección de la recta A1A2 y la tangente en Fa a la circunferencia de los nueve puntos. La es el punto de contacto de la otra tangente a la circunfeencia de los nueve puntos desde Ea. Así, la recta FaLa es la polar de Ea respecto a la circunferencia de los nueve puntos.

    En coordenadas baricéntricas:

    Ea = ((b-c)(2a^3+a^2(b+c)+(b-c)^2(b+c)) : -(a+c)(a^3+2b^3+a^2c-b^2c-c^3+a(b^2-c^2)) : (a+b)(a^3+a^2b-b^3-bc^2+2c^3+a(-b^2+c^2))).

    La ecuación de la recta FaLa:

    (c-b)(-a^2+b^2+c^2)(a^3+b^3+c^3+a^2(b+c)+a(b^2+bc+c^2))x +
    (a+c)(a^2-b^2+c^2)(a^3+a^2b+b^3+ab(b-c)-b^2c+bc^2-c^3)y -
    (a+b)(a^2+b^2-c^2)(a^3-b^3+a^2c+b^2c-bc^2+c^3+ac(-b+c))z=0.

    Las ecuaciones de las rectas FbLb y FcLc se obtienen por permutacion cíclica. Estas tres rectas tienen punto común X_{427}:
    ADGEOM1469f.png


    Si consideramos las circunferencias Γb y Γc, definidas de forma similar a Γa, el centro radical de estas circunferencias tiene coordenadas baricéntricas:

    ((a^4(b+c) + 2a^3(b^2+c^2) + 2a^2(b^3+b^2c+bc^2+c^3) + 2a(b+c)^2(b^2+c^2) + (b+c)(b^2+c^2)^2)/(b^2+c^2-a^2) : ... : ...)

    con (6,9,13)-número de búsqueda en ETC: -23.546702367310941758314

  • martes 05 de agosto del 2014

    Homotecia con centro en el circuncentro

    ( BMO2 Jan 2014)

      Sea, ABC un triángulo, P un punto y DEF su triángulo circunceviano.
    Oa, Ob, Oc son los circuncentros de los triángulos BCP, CAP, ABP, respectivamente.
    Od, Oe, Of son los circuncentros de los triángulos EFP, FDP, DEP, respectivamente.
     Las rectas OaOd, ObOe, OcOf concurren en el punto P', imagen de P en la homotecia de centro el circuncentro y razón 1/2.



    ( Mostrar/Ocultar figura )
      BMO2Jan2014.png
    Si (u:v:w) son las coordenadas baricéntricas del punto P, los vértices de su triángulo circunceviano son

    D=(-a^2vw : v(c^2v+b^2w) : w(c^2v+b^2w)),
    E=(u(c^2u+a^2w) : -b^2uw : w(c^2u+a^2w)),
    F= (u(b^2u+a^2 v) : v(b^2u+a^2v) : -c^2uv).

    El circuncentro de BCP es:

    Oa =(-a^2(-u(b^2(u+v-w)+c^2(u-v+w))+ a^2(u^2+2vw+u(v+w))) : -(b^2-c^2)u(-c^2v+ b^2(u+v))+a^4vw+ a^2(c^2v(u-w)+b^2(u^2+uv+2uw+vw)) : a^4vw-(b^2-c^2)u(b^2w-c^2(u+w))+ a^2(b^2(u-v)w+c^2(u^2+2uv+uw+vw))).

    Y el circuncentro de DFP es:
    Od=(-(b^2-c^2)^2u^2-a^4(u^2-2vw)+ 2a^2u(c^2(u+v)+b^2(u+w)) : -a^4v(u+w)- b^2(b^2-c^2)u(v+w)+ a^2(c^2v(u+w)+b^2(2uv-uw-vw)):-a^4(u+v)w+ c^2(b^2-c^2)u(v+w)+ a^2(b^2(u+v)w-c^2(uv-2uw+vw))).

    Las coordenadas de los circunscentros Ob, Oc, Oe, Of se obtienen por permutación cíclica.

    El punto de intersección de las rectas OaOd, ObOe, OcOf es:

    P' = ( (3S^2-SBSC)u + a^2SAv + a^2SAw : ... : ... ).


    OP' : OP = 1 : 2.

      Los triángulos OaObOc y OdOeOf son simétricos, respecto a P'. Así, ellos están inscritos en una misma cónica. En el caso particular de que P sea el incentro, los vértices de su triángulo circunceviano coinciden los vértices de OaObOc; y tal cónica corta por cuarta vez a la circunferencia circunscrita en el cuarto punto de intersección con la hipérbola de Feuerbach (conjugada isogonal de la recta OI).
    ( Mostrar/Ocultar figura )
      BMO2Jan2014X1.png


  • lunes 04 de agosto del 2014

    Cuatro triángulos perspectivos

    ( Triángulos 208)

    Sean ABC un triángulo, Γa la circunferencia que pasa por B y C y es tangente internamente a la circunferencia inscrita y similarmente, las circunferencias Γb y Γc. Designamos por Pa el punto de contacto de Γa y la circunferencia inscrita; similarmente, sean Pb y Pc. Sea Qa el punto de concurrencia de las tangentes a la circunferencia inscrita en Pb y Pc; y similarmente, Qb y Qc. Finalmente, sea Ta el punto de intersección de las rectas BPc y CPb; similarmente se definen Tb y Tc.

    ( Mostrar/Ocultar figura )
      gtre2486.png
    •  El centro de perspectividad de los triángulos ABC y PaPbPc es el centro X479.
    •  El centro de perspectividad de los triángulos ABC y QaQbQc es el centro X57.
    •  El centro de perspectividad de los triángulos ABC y TaTbTc es el centro X55.
    •  El centro de perspectividad de los triángulos PaPbPc y QaQbQc es el centro X3598 (Primer punto de Liu).
    •  El centro de perspectividad de los triángulos PaPbPc y TaTbTc es el centro X3599 (Segundo punto de Liu).
    •  El centro de perspectividad de los triángulos QaQbQc y TaTbTc es el centro:

    (a(a^4 - 4a^3(b+c) + 2a^2(3b^2-2b c+3 c^2) - 4a(b-c)^2(b+c) + (b-c)^2(b^2+6b c+c^2))/(b+c-a)^2 : ... : ...)

    que tiene coordenadas trilineales exactas en el triángulo de ETC (-0.06403720987389680, -0.1561335793835701, 3.778312595268649).

  • lunes 07 de julio del 2014

    Propiedades del punto de De Longchamps

    Definition of De Longchamps point

    Let the given triangle have vértices A, B, and C, opposite the respective sides a, b, and c, as is the standard notation in triangle geometry. In the 1886 paper in which he introduced this point, de Longchamps initially defined it as the center of a circle Γ orthogonal to the three circles Γa, Γb, and Γc, where Γa is centered at A with radius a and the other two circles are defined symmetrically. De Longchamps then also showed that the same point, now known as the de Longchamps point, may be equivalently defined as the orthocenter of the anticomplementary triangle triangle of ABC, and that it is the reflection of the orthocenter of ABC around the circumcenter

    Reference:
    de Longchamps, G. (1886), "Sur un nouveau cercle remarquable du plan du triangle", Journal de Mathématiques spéciales, 2. Sér. (in French) 5: 57-60. See especially section 4, "détermination du centre de Γ", pp. 58-59.

      Sea ABC un triángulo y Φa la cónica circunscrita con tangentes en B y C perpendiculares a BC. Similarmente se definen las cónicas circunscritas Φb y Φc.
      Denotamos por A', B', C' los cuartos puntos de intersección de Φb y Φc, Φc y Φa, Φa y Φb, respectivamete.
      La tangente en B a Φb vuelve a cortar a Φa en Ba. La tangente en C a Φc vuelve a cortar a Φa en Ca. Similarmente se definen los puntos Cb, Ab y Ac, Bc.

    En coordenadas baricéntricas:
      Φa : a²yz+SCzx+SBxy=0,     Φb : SCyz+b²zx+SAxy=0,     Φc : SByz+SAzx+c²xy=0.

    Cuarto punto de intersección de la cónicas circunscritas Φb y Φc, tripolo de la recta determinada por sus perspectores E=(SC:b²:SA) y F=(SB:SA:c²):

    LongchampsA.png


      Las rectas AA', BB', CC' concurren en el punto de De Longchamps, X20.

    ( Mostrar/Ocultar figura )
      Longchamps.png
    Descargar fichero GeoGebra


      La tangente, SBy+SCz=0, en A a Φa vuelve a cortar a Φb en

    Ab=(SBSC^2 : -SC(SA(SB-SC)+SBSC) : SB(SA(SB-SC)+SBSC)),

    y a Φc en

    Ac=(SB^2SC : -SC(SA(SB-SC)-SBSC) : SB(SA(SB-SC)-SBSC).


      Los puntos Bc, Ba, Ca y Cb, se definen cíclicamente.

      Los puntos Ab, Ac, Bc, Ba, Ca y Cb están en una misma cónica, de centro:

    LongchampsCentro.png

    con (6,9,13)-número de búsqueda en ETC: 0.4464942066184852070174993, y está alineado (Peter Moses) con los centros del triángulo:

    X112, X376, X577, X1249, X1294, X3163, X3184.



      El triángulo delimitado por las rectas BaCa, CbAb, AcBc es perspectivo con ABC con centro de perspectividad el punto de De Longschamps.
      El triángulo delimitado por las rectas BcCb, CaAc, AbBa es perspectivo con ABC con centro de perspectividad el punto

    HG070714p.png
    con (6,9,13)-número de búsqueda en ETC: 7.114153709376457534791226568

  • viernes 27 de junio del 2014

    Centros de semejanza y puntos de Beltrami

      Sea ABC un triángulo y P un punto sobre su circunferencia circunscrita Γ. Se denotan por A', B', C' las reflexiones de P en las mediatrices de BC, CA, AB, respectivamente; A'B'C' está inscrito en Γ.

    •   Se designa por A+ el centro de la semejanza directa que transforma el segmento BC en el C'A', por B+ el centro de la semejanza directa que transforma el segmento CA en el A'B' y por C+ el centro de la semejanza directa que transforma el segmento AB en el B'C'.

      El lugar geométrico de los centros de semejanza A+, B+, C+ son tres circunferencias Γ‍+a, Γ‍+b, Γ‍+c, con un punto común B1 (primer punto de Beltrami, inverso del primer punto de Brocard Ω1, respecto a la circunferencia circunscrita, Γ).
      La circunferencia Γ+t circunscrita a A+B+C+ pasa por B1 y el circuncentro, O.

    ( Mostrar/Ocultar figura )
      beltrami1.png


    Podemos expresar un punto en la circunferencia circunscrita en la forma (Barry Wolk, Anopolis #317):
    circumcircle.png

    Entonces, el centro de la semejanza que transforma el segmento BC en C'A' y la ecuación de la circunferencia que describe, cuando P recorre la circunferencia circunscrita Γ, son:
    Bertrami centros semejanzas+.png

    Γ+a es la inversa, respecto a Γ, de la circunferencia que pasa por B y es tangente a AC en C. Es decir, Γ‍+a es la circunferencia que pasa por B y es tangente en C a la circunferencia circunscrita al triángulo ACO.

    Procediendo cíclicamente, sobre los lados de ABC, se obtienen otras dos circunferencias:
    Γ+b: c^2 x y + b^2 x z + a^2 y z - a^2 c^2 y (x + y + z)/(c^2-b^2) =0,
    Γ+c: c^2 x y + b^2 x z + a^2 y z - a^2 b^2 z (x + y + z)/(a^2 - c^2)=0.
    El punto común a estas tres circunferencias es el primer punto de Beltrami:

    B1 = ( a^2(a^2-b^2) : b^2(b^2-c^2) : c^2(c^2-a^2)).




    Cambiamos la orientación de los segmentos que determinan las semejanzas:

    •   Se designa por A- el centro de la semejanza directa que transforma el segmento CB en B'A', por B- el centro de la semejanza directa que transforma el segmento BA en A'C' y por C- el centro de la semejanza directa que transforma el segmento A'B' en el BC.

      El lugar geométrico de los centros de semejanza A‍-, B‍-, C‍- son tres circunferencias Γ‍-a, Γ‍-b, Γ‍-c, con un punto común B2 (segundo punto de Beltrami, inverso del segundo punto de Brocard Ω2, respecto a la circunferencia circunscrita, Γ).
      La circunferencia Γ‍-t circunscrita a A‍-B‍-C‍- pasa por B2 y el circuncentro O.

    ( Mostrar/Ocultar figura )
      beltrami2.png
    El centro de la semejanza que transforma el segmento CB en B'A' y la ecuación de la circunferencia que describe, cuando P recorre la circunferencia circunscrita Γ, son:
    Bertrami centros semejanzas+.png

    Γ-a es la inversa, respecto a Γ, de la circunferencia que pasa por C y es tangente a AB en B. Es decir, Γ‍-a es la circunferencia que pasa por C y es tangente en B a la circunferencia circunscrita al triángulo ABO.

    Procediendo cíclicamente, sobre los lados de ABC, se obtienen otras dos circunferencias:
    Γ-b: c^2 x y + b^2 x z + a^2 y z - a^2 c^2 y (x + y + z)/(a^2-b^2) =0,
    Γ-c: c^2 x y + b^2 x z + a^2 y z - a^2 b^2 z (x + y + z)/(b^2 - c^2)=0.
    El punto común a estas tres circunferencias es el segundo punto de Beltrami:

    B1 = ( a^2(a^2-c^2) : b^2(b^2-a^2) : c^2(c^2-b^2)).




    OTRAS CONSIDERACIONES:

    ♦   Los centros O‍+t y O‍-t de las circunferencias Γ‍+t y Γ‍-t están en las mediatrices de los segmentos OB1 y OB2, cuyo punto de intersección Q tiene primera coordenada baricéntrica:

    (a^2(-a^8 + a^6(b^2+c^2) + a^4(b^4+5b^2c^2+c^4) - a^2(2b^6+3b^4c^2+3b^2c^4+2c^6) + b^8 + c^8),

    con (6,9,13)-número de búsqueda en ETC: 1.99844087285917963875

      El segundo punto de intersección O' de las circunferencias Γ‍+t y Γ‍-t está en la recta de Lemoine, polar trilineal del simediano.
    ( Mostrar/Ocultar figura )
      beltrami semejanzas.png



    ♦   La envolvente de la recta determinada por los centros O‍+t y O‍-t de las circunferencias Γ‍+t y Γ‍-t es la parábola de foco el circuncentro y directriz la recta de Lemoine.
    El punto medio Mt de O‍+t y O‍-t queda en la tangente en el vértice V de esta parábola. Para t=0 (P(0) el punto de Steiner), M0 es el vértice (punto medio de X3 y X187, éste es el punto medio de los puntos de Beltrami):

    V= (a^2(4a^6 - 9a^4(b^2+c^2) + 2a^2(4b^4+b^2c^2+4c^4) - 3b^6+b^4c^2+ b^2c^4-3c^6 ) : ... : ...),

    con (6,9,13)-número de búsqueda en ETC: 2.47073957830213664904597
    ( Mostrar/Ocultar figura )
      BeltramiParabola.png


  • martes 24 de junio del 2014

    Construcción de triángulo

    ( Anopolis #1704, Antreas P. Hatzipolakis)

      Construir un triángulo ABC, dado A'B'C' el triángulo ceviano de algún punto, tal que A' es el pie de bisectriz en A y B' es el pie de la mediana por B.


    (Publicado en la matemática griega. revista Euclides, 12, 1973) [the translation of "Published in the Greek mathematical magazine Euclid, Dec. 1973"]

    Construcción analítica:

      Tomemos un punto arbitrario P en el plano del triángulo dado A'B'C', que será el de referencia para un sistema de coordenadas baricéntricas.
      Si P fuera el vértice del triángulo a construir (opuesto al lado que pasan por A'), otro vértice será la reflexión R de P en B'. El tercer vértice Q ha de ser RA'∩PC'.
    ( Mostrar/Ocultar figura )
      Anopolis1704.png

      Si (u:v:w) son las coordenadas baricéntricas de P respecto a A'B'C', R=(-u : 2u+v+2w : -w) y Q=(-u(2u+v+2w) : -v(2u+v+2w) : vw). Como A'B'C' debe ser el triángulo ceviano de un cierto punto en PQR, se tiene que verificar que las rectas PA', QB' RC' sean concurrentes; es decir,

    4 u w (u + w) (u + v + w)=0

      Luego, una condición necesaria para construir el triángulo es que el punto P esté sobre la recta δ: x+z=0, paralela a A'C' por B'.

      Debemos localizar la posición del punto P, en la recta δ, tal que la recta PR' (R' la reflexión de R en PA') pase por C'. Al variar P sobre δ, la recta PR' envuelve la circunferencia Γ de centro en A' y tangente a δ. (Nikos Dergiades, ADEGEOM #493):

    PR': w (c'^2 v (v + w) + w (-a'^2 v + b'^2 (v + w)))x -w (-w (a'^2 (u - v + w) + b'^2 (-u + v + w)) + c'^2 (w (v + w) + u (2 v + w)))y + c'^2 v (u v + w (v + w)) - w (a'^2 v (-v + w) + b'^2 (v^2 + u w + v w))z=0.

    Γ: a'^4 x^2 - 2 a'^2 b'^2 x^2 + b'^4 x^2 - 2 a'^2 c'^2 x^2 - 2 b^2 c'^2 x^2 + c'^4 x^2 + 2 a'^4 x y - 4 a'^2 b'^2 x y + 2 b'^4 x y - 4 a'^2 c'^2 x y - 4 b'^2 c'^2 x y + 2 c'^4 x y + a'^4 y^2 - 2 a'^2 b'^2 y^2 + b'^4 y^2 - 2 a'^2 c'^2 y^2 + 2 b'^2 c'^2 y^2 + c'^4 y^2 + 2 a'^4 x z - 4 a'^2 b'^2 x z + 2 b'^4 x z - 4 a'^2 c'^2 x z - 4 b'^2 c'^2 x z + 2 c'^4 x z + 2 a'^4 y z - 8 a'^2 b'^2 y z + 6 b'^4 y z - 4 a'^2 c'^2 y z + 2 c'^4 y z + a'^4 z^2 - 2 a'^2 b'^2 z^2 + 5 b'^4 z^2 - 2 a'^2 c'^2 z^2 - 2 b'^2 c'^2 z^2 + c'^4 z^2 =0.

      Por tanto, un lado del triángulo a construir es una tangente a Γ desde C'.

    Construcción:

    ( Mostrar/Ocultar figura )
      Anopolis1704c.png
      Trazamos la paralela δ a A'C' por B' y la circunferencia Γ con centro en A' y tangente a δ. Si C' es exterior a Γ, una tangente desde C' a Γ corta a δ en A, el vértice C es la reflexión de A en B' y el vértice C=AC'∩CA'.
      Una segunda tangente desde C' permite construir un segundo triángulo A1B1C1.
      En uno de estos triángulos A' será el pie de la bisectriz externa desde el vértice opuesto.
      Si C' es interior a la circunferencia Γ no hay solución.

  • sábado 14 de junio del 2014

    Propiedad de una hipérbola equilátera

    ( ADGEOM #1327, Dao Thanh Oai)

      Sean ABC un triángulo, P un punto en su plano y ℵP la hipérbola equilátera circunscrita a ABC y que pasa por P (pasa también por el ortocentro H). Una recta δ que pasa por P, vuelve a cortar a la hipérbola en un punto Q. Denotamos por A1, B1, C1 los puntos en los que las rectas por P y perpendiculares a AQ, BQ, CQ cortan a los lados BC, CA, AB, respectivamente.

    Los puntos A1, B1, C1 están alineados sobre una recta δ1 perpendicular a PQ.

    ( Mostrar/Ocultar figura )
      ADGEOM1327.png
      Si (u:v:w) son las coordenadas baricéntricas de P, respecto al triángulo ABC, la ecuación de la hipérbola equilátera circunscrita ℵP es (notación de Conway):

    u(SBv-SCw)yz + v(SCw-SAu)xz + w(SAu-SBv)xy = 0.


      A la recta δ, ((1-t)v - t w)x + (t-1)uy + tuz=0, le corresponde el segundo punto Q de intersección con la hipérbola ℵP:

    ((1-t)tu(SBv-SCw) : -t(SAu-SCw)((t-1)v+ tw) : (t-1)(SAu-SBv)((t-1)v+tw)) ).

      La envolvente de las rectas δ1, cuando δ gira alrededor de P, es una cónica (es fácil ver que la correspondencia B1↦ C1 entre las rectas AC y AB es una proyectividad) tangente a los lados de ABC y a la recta del infinito (párabola inscrita ℘p). Por tanto, su directriz pasa por H (es la recta PH) y su foco Fp está en la circunferencia circunscrita (es el conjugado isogonal del punto del infinito determinado por la dirección perpendicular a la recta PH).
      A toda hipérbola equilátera circunscrita a ABC, con uno de sus puntos sobre una recta que pasa por el ortocentro, le corresponde la misma parábola envolvente de las rectas δ1.

      Otra forma de construir el foco Fp es hallando la interseción de las rectas H1(PH∩BC), H2(PH∩CA), H3(PH∩AB), siendo H1H2H3 el triángulo circunceviano de H.

      Directriz PH:   SA(SBv-SCw)x + SB(SCw-SAu)y + SC(SAu-SBv)z=0.
      Foco Fp:   (a^2/(a^2(w-v)+(b^2-c^2)(v+w)) : ... : ... ).
      Ecuación de la parábola ℘p:

    El cuarto punto Qp de intersección de la hipérbola ℵp con la circunferencia circunscrita es el segundo punto de intersección de la hipérbola con la recta PFp.


    Las cordenadas de Qp son: (1/(b^2(SAu-SBv)w + c^2(SAuv-SCvw)) : ... : ... ).

    Este resultado nos da un método para construir el foco de una parábola, inscrita en un triángulo, de la que conocemos su directriz (que ha de pasar por el ortocentro):
    •  Construimos la hipérbola circunscrita al triángulo, que pasa por el ortocentro y por un punto P cualquiera de la directriz de la parábola.
    •  Construimos el cuarto punto Qp de intersección de tal hipérbola y la circunferencia circunscrita.
    •  El foco de la parábola es el segundo punto de intersección de la recta PQp con la circunferencia circunscrita.
    ( Mostrar/Ocultar figura )
       FocoParabolaInscrita.png


  • miércoles 11 de junio del 2014

    Circunferencias tangentes y cúbica de Lucas

    ( ADGEOM #1323, Emmanuel José García)

      Sean ABC un triángulo y P un punto en su plano. Las reflexiones de P en los vértices del triángulo ceviano XaXbXc de un punto X, forman un triángulo QaQbQc. Los tres puntos que resultan de reflejar este triángulo en los lados de ABC determinan una circunferencia Γ(X,P).

    ( Mostrar/Ocultar figura )
      ADGEOM1322.png


      Usando coordenadas baricéntricas, respecto al triángulo ABC, si ponemos X=(u:v:w) y P=(p:q:r), la reflexión del punto Qa en el lado BC es:

    ADGEOM1322pto.png

    Y la condición analítica para que el punto P esté en la circunferencia Γ(X,P) es:

    ADGEOM1322cond.png

      • Cuando P queda en el infinito (primera condición) la construcción de Γ(X,P) no es posible.
      • El último factor nos indica que si X=(u:v:w) está en la cúbica de Lucas (K007), P queda sobre Γ(X,P), para cualquier posición de P en el plano.
      • Los restantes tres factores que figuran en esta condición, mirados como ecuaciones en las variables p,q,r, reprentan las rectas perpendiculares a los lados de ABC en los pies de las cevianas del punto X. Por lo tanto, P está sobre Γ(X,P) si está sobre alguna de estas perpendiculares.

    P está sobre la circunferencia Γ(X,P) si y solo si X está sobre la cúbica de Lucas o sobre las rectas perpendiculares a los lados de ABC en los pies de las cevianas del punto X.


      Tomemos un punto L1 sobre la curva de Lucas y un punto P arbitrario del plano, entonces P está en la circunferencia Γ(L1,P) cuyo centro es el punto D1 (en la cúbica de Darboux, K004), tal que el triángulo pedal de D1 es el triángulo ceviano de L1.
      La recta D1P corta a la cúbica de Darboux en otros dos puntos D2 y D3 (reales o imaginarios). Si L2 y L3 son los puntos (en la cúbica de Lucas) tales que sus triángulos cevianos son los triángulos pedales de D2 y D3, respectivamente, se verifica que las tres circunferencias Γ(L1,P), Γ(L2,P) y Γ(L3,P) son tangentes en P.

    ( Mostrar/Ocultar figura )
      ADGEOM1322b.png


      Sean dos puntos X y X' conjugados isotómicos sobre la cúbica de Lucas y sean Y e Y' los puntos (sobre la cúbica de Darboux) tales que sus triángulos pedales son los triángulos cevianos de X y X', respectivamente. Entonces, todo punto P pertenece a las circunferencias Γ(X,P) y Γ(X',P), además, sus centros son Y e Y', respectivamente.
    Se tiene que la recta YY' pasa por el circuncentro de ABC y la recta XY pasa por el punto de De Longchamps L (simétrico del ortocentro H respecto el circuncentro O), para todo punto X sobre la cúbica de Lucas..

    Las circunferencias Γ(X,P) y Γ(X',P) son tangentes en P si y solo si P está en la recta YY'.


    En estas circunstancias, si G es el baricentro, también la circunferencia Γ(G,P), con centro en el circuncentro, es tangente en P a las circunferencias Γ(X,P) y Γ(X',P).

    En particular (Peter Moses):
    Si X=X7 = punto de Gergonne y X'=X8 = punto de Nagel, las circunferencias Γ(X7,P) y Γ(X8,P) son tangentes en P si y solo si P queda en la recta que pasa por O=X3, I=X1 and X40.

    ( Mostrar/Ocultar figura )
      ADGEOM1322Moses.png


  • martes 10 de junio del 2014

    Una caracterización de la cuártica de Euler-Morley

    ( Anopolis #1645, Antreas P. Hatzipolakis)

      Sean ABC un triángulo y P, Q dos puntos conjugados isogonales.
    Ap es el punto de intersección, distinto de P, de la recta AP con la circunferencia circunscrita a BPC; Aq es el punto de intersección, distinto de Q, de la recta AQ con la circunferencia circunscrita a BQC.
    Similarmente se definen los puntos Bp, Bq, Cp y Cq.
      Denotamos por R el centro radical de las circunferenecias cicunscritas a los triángulos AApAq, BBpBq y CCpCq.
      Si las coordenadas baricéntricas de P son (u:v:w),
    Rp =(a^2u(c^2v^2+b^2w^2) : b^2v(c^2 u^2+a^2w^2) : c^2(b^2u^2+a^2v^2)w).

    El lugar geométrico de los puntos P tales que el circuncentro, P y R están alineados es la cuártica de Euler-Morley.

    ( Mostrar/Ocultar figura )
      Anopolis1645.png


  • lunes 9 de junio del 2014

    Caracterización de la cúbica de Darboux en términos de la circunferencia pedal

    ( Anopolis #1648, Antreas P. Hatzipolakis)

      Sean ABC un triángulo, P, P* dos puntos conjugados isogonales y A'B'C', A"B"C" los triángulos pedales de P, P*, respectivamente.
    Se denota por A1 el punto de intersección (distinto de A) de la recta AA' con la circunferencia pedal de P, y por A* el punto de intersección de la recta B"C" con la tangente a la circunferencia pedal de P en A1.
    De forma similar se define los puntos B* y C*.

    El lugar geométrico de los puntos P tales que su circunferencia pedal está definida y los puntos A*, B* y C* están alineados es la cúbica de Darboux.

    ( Mostrar/Ocultar figura )
      Anopolis1648.png
    Si P=P*=I, incentro, la recta que contiene a los puntos A*, B*, C* es la tripolar de X7, punto de Gergonne.

    (Art of Problem Solving, http://www.artofproblemsolving.com/Forum/viewtopic.php?f=47&t=592844 ) Si P es el circuncentro (P* el ortocentro) , A*, B* y C* están en la recta de ecuación baricéntrica:
    (-a^2 + b^2 + c^2) (a^4 (b^2 + c^2) - b^2 c^2 (b^2 + c^2) + a^2 (b^4 - b^2 c^2 + c^4))x+... =0.

  • viernes 6 de junio del 2014

    Hipérbolas de asíntotas paralelas a los lados de un triángulo

    ( Anopolis #1605, Antreas P. Hatzipolakis)

      Sean p, q, r tres recta fijas y D un punto fijo sobre p. Una recta variable δ a través de D interseca a q en Q y δ', la reflexión de δ en p, corta a r en R. Sea M el punto medio de QR.
      El lugar geométrico de M, cuando δ gira alrededor de D, es una hipérbola de asíntotas paralelas a las rectas q y r, y que pasa por los puntos medios de B=p∩r y C=p∩q (cuando δ=p), de A=q∩r y q∩δ' (cuando δ pasa por A=q∩r) y de A=q∩r y r∩δ' (cuando δ pasa por A=q∩r).

    ( Mostrar/Ocultar figura )
      Anopolis1605.png
      Si hacemos variar el punto D sobre la recta p, se genera un haz de hipérbolas, cuyos cuatro puntos base son los puntos del infinito de las rectas q y r, el punto medio Ma de B=p∩r y C=p∩q, y H'a (conjugado armónico de la proyección ortogonal de A=q∩r sobre p).
      Las hipérbolas degeneradas son los pares de rectas que se obtienen al unir dos a dos los puntos base del haz.
    ( Mostrar/Ocultar figura )
      Anopolis1605haz.png


    Aplicación a la geometría del triángulo:

      Sean ABC un triángulo, P un punto y DEF el triángulo PEDAL de P. Consideremos las hipérbolas ℵa, ℵb y ℵc, según la construcción anterior, procediando cíclicamente sobre los lados del triángulo ABC y tomando, sucesivamente, los puntos D, E y F en sus lados opuestos.
    Si P tiene coordenadas baricéntricas (u:v:w), la ecuación de ℵa es:
    hipApedal.png

    Los centros de las hipérbolas ℵa, ℵb y ℵc están alineados si y sólo si P está sobre la hipérbola de Kiepert.

    ( Mostrar/Ocultar figura )
      Anopolis1605tr.png


      Sean ahora un triángulo ABC, un punto P y PaPbPc el triángulo CEVIANO de P. Consideremos las hipérbolas ℵa, ℵb y ℵc, según la construcción anterior, procediando cíclicamente sobre los lados del triángulo ABC y tomando, sucesivamente, los puntos Pa, Pb y Pc en sus lados opuestos.
    Si P tiene coordenadas baricéntricas (u:v:w), la ecuación de ℵa es:
    hipAceva.png

    Los centros de las hipérbolas ℵa, ℵb y ℵc están alineados si y sólo si P está sobre la ortopivotal cúbica de ortopivote X5, K060.

    ecuK060.png
    ( Mostrar/Ocultar figura )
      Anopolis1605ceva.png
      Para los centros en K060, P=X4, X30, X80, X265, la recta que contiene a los centros de las hipérbolas ℵa, ℵb y ℵc son las tripolares de X253, X69, X7, X264, respectivamente.

  • martes, 3 de junio del 2014

    Caracterizaciones de las cúbicas de Darboux, Lucas y Simson

      Dados un triángulo ABC y un punto P, sean MaMbMc el triángulo medial y DEF el triángulo pedal de P.
      Consideremos el lugar geométrico del punto medio de los puntos en que una recta variable, que pasa por D, corta a las rectas AB y AC.

    ( Mostrar/Ocultar figura )
      HG030614lg.png
    Se trata de una hipérbolaa, que pasa por A, D, Ma con asíntotas paralelas a los lados AB y AC. El centro de ℵa es el punto medio de AD. Denotamos por ta la tangente en A a la hipérbola ℵa

      Si P tiene coordenadas baricéntricas (u:v:w), la ecuación de ℵa y de ta son:
    HG030614hA.png

      Similarmente, se definen las tangentes tb y tb a las correspondientes hipérbolas ℵb y ℵc.
    ( Mostrar/Ocultar figura )
      HG030614conicas.png

    El triángulo A'B'C' delimitado por las tangentes ta, tb y tc es perspectivo con ABC si y solo si P está sobre la circunferencia circunscrita o sobre la cúbica de Darboux.
    El centro de perspectividad Q de ABC y A'B'C', cuando P recorre la cúbica de Darboux está sobre la cúbica de Lucas.

    Ocurre que, en este caso, los vértices D, E, F del triángulo pedal de P está en las rectas AA', BB', CC'.
    ( Mostrar/Ocultar figura )
      HG030614d.png
      Parejas de centros {P,Q}, P en la cúbica de Darboux y Q en la cúbica de Lucas:
    {X1,X7}, {X3,X2}, {X4,X4}, {X20,X69}, {X40,X8}, {X64,X253}, {X84,X189}, {X1490,X329}, {X1498,X20}, {X3345,X1034}, {X3346,X1032}.

    Cuando P queda sobre la circunferencia circunscrita a ABC, los vértices de su triángulo pedal quedan sobre la recta de Simson s(P) de P, y las tres tangentes ta, tb y tc concurren en el tripolo de s(P), sobre la cúbica de Simson.

    ( Mostrar/Ocultar figura )
      HG030614c.png


  • domingo 1 de junio del 2014

    Triángulos con su ortocentro en una recta paralela a la de Euler

    ( Anopolis #1584, Antreas P. Hatzipolakis)

    Dado un triángulo ABC y una recta d. Se consideran los triángulos ABaCa, con Ba en AB, Ca en AC y el ortocentro Ha sobre la recta d.
    1.1. El circuncentro Oa de ABaCa recorre la mediatriz ma de ASa. donde Sa es el punto "Anti-Steiner" (denominado así por Darij Grinberg en "Anti-Steiner point with respect to a triangle") de la recta d respecto al triángulo ABaCa.
    1.2. El centro de la circunferencia de los nueve puntos Na de ABaCa recorre una recta na.

    El punto Sa (en la circunferencia es circunscrita a ABaCa) es la intersección de las refleciones de d en AB y AC.
    Dos puntos particulares de la recta na son los puntos medios de EFa y FEa, siendo E y F las intersecciones de d con los lados AB y AC, respectivamente; y Ea y Fa las intersecciones de ma con los lados AB y AC, respectivamente.

    ( Mostrar/Ocultar figura )
      HG010614.png

    Denotamos por Ad el punto de intersección de ma y na. Similarmente, procediendo cíclicamente sobre los vértices de ABC, se definen los puntos Bd y Cd.

     Cuando la recta d es paralela a la recta de Euler de ABC, las rectas AAd, BBd, CCd concurren en el punto de Kosnita, X54.
      Existe una única recta d0, paralela a la recta de Euler, para la cual los puntos A0, B0 y C0 están alineados.

    ( Mostrar/Ocultar figura )
      HG010614d0.png

    La ecuación baricéntrica de la recta de d0 es:
    HG010614Ecd0.png
    Esta recta no contiene centros del triángulo, que figuren actualmente en ETC, salvo su punto en el infinito, X30.

  • sábado, 31 de mayo del 2014

    Recta de Euler e hipérbola asociada

    Dado un triángulo ABC, su recta de Euler corta a los lados BC, CA, AB en los puntos D, E, F, respectivamente. La circunferencia de centro en F y que pasa por A vuelve a cortar a AC en Ab. La circunferencia de centro en E y que pasa por A vuelve a cortar a AB en Ac. La recta AbAc (que pasa por el ortocentro) corta a BC en Aa.
    Similarmente, se definen los puntos Ba, Bb, Bc, Ca, Cb, Cc.

    Los puntos Aa, Bb y Cc están en la recta que corta a la circunferencia de Euler en los centros de las hipérbolas rectangulares de Jerabek y circunscrita que pasa por el centro X93.

    ( Mostrar/Ocultar figura )
      recta Euler Circunferencias.png
    Esta recta corta a la recta de Euler en el punto de coordenadas baricéntricas:
    HG310514P.png

    con número de búsqueda en ETC: 3.6723293499380232572548885

    Los seis puntos Ab, Ac, Ba, Bc, Ca y Cb están en una hipérbola que pasa por X107 (en la circunferencia circunscrita), X648 (tripolo de la recta de Euler).



    Las reflexiones de la recta AbAc en los lados de ABC se cortan en un punto Sa de la circunferencia circunscrita (Teorema de Collings), denomonado por Darij Grinberg "Anti-Steiner point" de la recta AbAc.
    Análogamente, se construyen los puntos Sb y Sc.

    Las rectas ASa, BSb, CSc son parelelas, con punto del infinito X523, conjugado isogonal del foco de la parábola de Kiepert.



  • martes, 27 de mayo del 2014

    Triángulo ceviano y ejes radicales

    ( Anopolis #1574, Antreas P. Hatzipolakis)

    Sean ABC un triángulo, P un punto y A'B'C' su triángulo ceviano.
    Conideremos los siguentes puntos: Ma el punto medio de AH, M'a el punto medio de A'H, y M1 el punto medio de MaM'a. Similarmente, se definen M2 y M3.
    Denotamos por:
    Ra el eje radical de las circunferencias M2(M2B') y M3(M3C').
    Rb el eje radical de las circunferencias M3(M3C') y M1(M1A').
    Rc el eje radical de las circunferencias M1(M1A') y M2(M2B').

      Las paralelas La, Lb, Lc a Ra, Rb, Rc a través de A, B, C, respectivamente, son concurrentes si y sólo si P está en la cúbica de Lucas. El punto de concurrencia Q queda sobre la cúbica de Darboux.

    ( Mostrar/Ocultar figura )
      Anopolis1574g.png
      Si P(u:v:w), en coordenadas baricéntricas,
    Q(a^4u(4u+v+w) - 2a^2(v-w)(b^2v-c^2w) - (b^2-c^2)(v+w)(b^2(u+2v)-c^2(u+2w)) : ... : ...).

      Parejas de centros {P,Q}, P en la cúbica de Lucas y Q en la cúbica de Darboux:
    {X2,X4}, {X4,X64}, {X7,X84}, {X8,X1}, {X20,X20}, {X69,X3}, {X189,X3345}, {X253,X3346}, {X329,X40}, {X1032,X3348}, {X1034,X3347}.

      La primera componente de las coordenadas baricéntricas del centro radical R de las circunferencias M1(M1A'), M2(M2B') y M3(M3C') es:
    a^4 v (u+v) w (u+w) (2 u^3+(v-w)^2 (v+w)+2 u (v+w)^2)-a^2 (c^2 v (2 u^5 (v+w)+v (v-w) w^2 (v+w)^2+u^4 v (v+3 w)+2 u v w^2 (v^2-w^2)-2 u^3 (v^3-v w^2)-u^2 (v^4+v^3 w-2 w^4))+b^2 w (2 u^5 (v+w)-v^2 (v-w) w (v+w)^2+u^4 w (3 v+w)+2 u^3 w (v^2-w^2)+u (-2 v^4 w+2 v^2 w^3)+u^2 (2 v^4-v w^3-w^4)))-(b^2-c^2) u (v+w) (b^2 w (u^3 (2 v-w)+2 u^2 v (v+w)+v (v^3+v^2 w+v w^2+w^3)+u (v^3+4 v^2 w+v w^2+w^3))-c^2 v (-u^3 (v-2 w)+2 u^2 w (v+w)+w (v^3+v^2 w+v w^2+w^3)+u (v^3+v^2 w+4 v w^2+w^3))).


      Los puntos P, Q y R están alineados si y sólo si P está en una curva algebraica de grado diez, que pasa por A, B, C, X2, X4, X20, los puntos medios de los lados (donde es tangente a los lados), pies de las cevianas de X69 y por los puntos del infinto de los lados.


      Si P es el baricentro, Q es el ortocentro, y R es X3830, que divide al segmento X2X4 en la razón 3:-1.


      Los puntos La∩Rb, La∩Rc, Lb∩Rc, Lb∩Ra, Lc∩Ra, Lc∩Rb quedan sobre una misma cónica de centro el punto medio de Q y R.



  • martes, 13 de mayo del 2014

    Construcción triángulo (AB AC IH)

    ( Art of Problem Solving, jrrbc)

    Construcción de un triángulo conocidas las rectas determinadas por dos de sus lados y la recta que pasa por el incentro y el ortocentro.

    Del triángulo ABC a determinar, admitamos que el vértice A sea el punto de intersección de las dos rectas lb y lc dadas, que coinciden con lados del triángulo. El incentro I de ABC ha de estar en una de las dos bisectrices de tales rectas y sobre la recta r dada, que ha de contener también al ortocentro H.

    Denotamos por Y y Z las proyecciones ortogonales de I sobre los lados conocidos lb y lc, estos son los puntos de contacto de la circunferencia inscrita Γ del triángulo buscado ABC con los lados AC y AB, respectivamente. Nos falta determinar el punto X de contacto Γ con el lado BC.
    ( Mostrar/Ocultar figura )
      jrrbc.png

    Esta es una construcción con regla y compás de posibles soluciones:
    (Ver detalles en el caso de construcción de triángulos: AB AC IH)

    Por el punto de intersección M de r con la perpendicular a lb por A, se traza la perpendicualar a lc, que corta a lb en Ba.
    Por el punto de intersección N de r con la perpendicular a lc por A, se traza la perpendicualar a lb, que corta a lc en Ca.
    Sean P el punto de interseccion de las rectas BaCa y YZ, y Q el punto de intersección de r con la reflexión de YZ respecto a I.
    Los puntos de corte de la recta PQ con Γ dan los puntos de tangencia del lado BC con la circunferencia inscrita.
    En consecuancia, al considerar la otra bisectriz de las rectas lb y lc, PUDIERAN EXISTIR HASTA CUATRO SOLUCIONES.

    Mostrar/Ocultar Un caso con tres soluciones
       gtr2531ec.png


  • sábado, 10 de mayo del 2014

    Conjugado isotómico del conjugado isogonal

    Sean ABC un triángulo, P un punto y DEF su triángulo circunceviano. Denotamos por (Ab) la circunferencia que pasa por D y es tangente a AB en B, y por (Ac) la circunferencia que pasa por D y es tangente a AC en C.
    Sea A' el otro punto de intersección de estas circunferencias (está sobre BC). Similarmente, se definen los puntos B' y C'. (ver Estrofoide de Jerabek)

    A'B'C' es el triángulo ceviano de tgP, conjugado isotómico del conjugado isogonal de P.

    ( Mostrar/Ocultar figura )
      tgP.png


  • viernes, 9 de mayo del 2014

    Proyectividad en la recta de Euler

     Sean ABC un triángulo y P un punto. El triángulo circunceviano de P es perspectivo con el triángulo ceviano del conjugado isotómico del conjugado isogonal de P si y solo si P está en la recta de Euler.
     El perspector P' también está en al recta de Euler y la correspondencia P ↦ P' es una proyectividad, con puntos dobles los de intersección de la recta de Euler con la circunferencia circunscrita.

    ( Mostrar/Ocultar figura )
      ProyectividadRectaEuler.png
    Si P es un punto del plano del triángulo ABC con coordenadas baricentricas (u:v:w), las coordenadas de los vértices de su triángulo circunceviano DEF (puntos en los que las cevianas de P vuelven a cortar a la circunferencia circunscrita) son:

    D(-a^2vw : v(c^2v+b^2w) : w(c^2v + b^2w)), E(u(c^2u+a^2w) : -b^2uw : w(c^2u+a^2w)), F(u(b^2u+a^2v) : v(b^2u+a^2v) : -c^2uv).

    Y las coordenadas de los vértices del triángulo ceviano de tgP:

    L(0 : c^2v : b^2w), M(c^2u : 0 : a^2w), N(b^2u : a^2v : 0).

    Los triángulos DEF y LMN son perspectivos si y sólo si
    ((-a^2b^2+b^4+a^2c^2-c^4)u - (a^4+a^2b^2-b^2c^2+c^4)v + (a^4-b^4-a^2c^2+b^2c^2)w) (c^2uv+b^2uw+a^2vw)=0.
    Es decir, si P está en la circunferencia circunscrita (DEF degenera en un punto) o en la recta de Euler.

    Si P(u:v:w) está en la recta de Euler el centro de prespectividad de DEF y LMN es:

    P' (2b^2c^2SAu^2 + a^2(b^2c^2+a^2(SA-a^2))vw + b^2(b^2c^2+a^2(SA-b^2))wu + c^2(b^2c^2+a^2(SA-c^2))uv : ... : ...)


    Pares de puntos homólogos (P↦P'), figurando ambos actualmemtes en ETC:
    (X2↦X22), (X3↦X20), (X4↦X3), (X22↦X1370), (X23↦X858), (X24↦X4), (X25↦X2), (X27↦X4184), (X28↦X21), (X29↦X4225), (X30↦X2071), (X186↦X30), (X199↦X3151), (X237↦X401), (X378↦X376), (X403↦X186), (X419↦X237), (X436↦X418), (X451↦X2915), (X468↦X23), (X1113↦X1113), (X1114↦X1114), (X1593↦X3522), (X1598↦X3523), (X2070↦X3153), (X2073↦X5196), (X2074↦X1325), (X2409↦X4230), (X3144↦X3145), (X3145↦X3152), (X3515↦X3146), (X3517↦X3091), (X3518↦X5), (X3520↦X550), (X3542↦X24), (X4183↦X1817), (X4185↦X4189), (X4186↦X4188), (X4213↦X199), (X4222↦X404), (X4227↦X4221), (X4230↦X4226), (X4231↦X4220), (X4232↦X1995), (X4233↦X4228), (X4238↦X4236), (X4241↦X4243), (X4244↦X4238), (X4246↦X3658), (X4247↦X4234), (X4249↦X4237), (X5136↦X4216), (X5200↦X1599), (X6240↦X3520), (X6353↦X25), (X6622↦X3515).

    Siendo los puntos dobles X1113 y X1114, y los puntos límites X186 y X2071.

    En términos de la referecia proyectiva sobre la recta de Euler {O,H;N} (de puntos base el circuncentro y el ortocentro y punto unidad el centro de la circunferencia de los nueve puntos), la correspondencia P↦P' se expresa por:
    proyectividadRectaEulerEc..png

    Los puntos dobles se obtienen para los valores de λ raíces del polinomio
    8a^2b^2c^2SASBSCC - 2a^2b^2c^2λ + λ²=0.
    λ12 = a^2b^2c^2(1 ± (a^2b^2c^2 - 8SASBSC)½).

    Obteniéndose los puntos de coordenadas homogéneas, en la referencia {O,H;N},
    (-abc : abc ± (a^2b^2c^2 - 8SASBSC)½).

    Que corresponde a los puntos X1113 y X1114, de interseccción de la recta de Euler con la circunferencia circunscrita.


    Adicionalmente podemos notar que:

    Cuando P varía en al recta de Euler, las rectas DL, EM, DN pasan por puntos fijos (sobre la circunferencia circunscrita) que son los vértices del triángulo circunceviano del punto de De Longchamps, respecto al triángulo A'B'C', antipodal de ABC.



  • jueves, 8 de mayo del 2014

    Estrofoide de Jerabek

    Sean ABC un triángulo, P un punto y DEF su triángulo circunceviano. Denotamos por Ab el centro de la circunferencia que pasa por D y es tangente a AB en B, y por Ac el centro de la circunferencia que pasa por D y es tangente a AC en C.
    Similarmente, se definen los puntos Bc, Ba y Ca, Cb.

    Las rectas AbAc, BcBa y CaCb no son concurrentes y el triángulo A'B'C' delimitado por ellas es perspectivo con ABC si y solo si P queda en la estrofoide de Jerabek (K039).

    ( Mostrar/Ocultar figura )
      estrofoideJerabekPropiedad.png

    Si (u:v:w) son las coordenadas baricentricas de P se tiene que:
    Ab = (-a^2b^2(a^2-b^2+c^2)w : -a^4c^2v-(b^2-c^2)^2(c^2v+b^2w)+ a^2(b^2+c^2)(2c^2v+b^2w) : 2a^2b^2c^2w),
    Ac = (-a^2c^2(a^2+b^2- c^2)v : 2a^2b^2c^2v : -a^4b^2w-(b^2-c^2)^2(c^2v+ b^2w)+a^2(b^2+c^2)(c^2v+2b^2w))

    La recta AbAc es:
    (a^4b^2c^2vw+(b^2-c^2)^2(c^2v+b^2w)^2- a^2(b^2+c^2)(c^2v+b^2w)^2)x + (-a^2b^2w(c^4v-b^4w+ b^2(a^2w+c^2(-v+w))))y + (-a^2c^2v(a^2c^2v+(b^2- c^2)(c^2v+b^2w)))z=0.

    Las rectas AbAc, BcBa y CaCb son concurrentes si y solo si P queda en una séxtica con puntos dobles en los vértices de ABC

    ( Mostrar/Ocultar figura )
      estrofoideJerabekPropiedad.png

    Notificación de Bernard Gibert:

    Los triángulos ABC y A'B'C' son ortológicos si y solo si P está sobre la estrofoide de Jerabek.   En este caso, el centro de ortología de ABC respecto a A'B'C' describe la estrofoide de Ehrmann (K025).

    ( Mostrar/Ocultar figura )
      estrofoideJerabekOrtologico.png



    Alguna información sobre la estrofoide de Jerabek

    •  Una estrofoide de foco F, de punto doble D y eje d0 (pasando por D) es el lugar geométrico de los puntos M de una recta variable d pasando por F y tal que PM=PD, donde P es el punto de intersección de la recta d0 con d
    ( Mostrar/Ocultar figura )
      estrofoide.png

    •  La estrofoide de Jerabek (K039) tiene punto doble el circuncentro, foco el inverso en la circunferencia circunscrita de X263 y eje la recta que une el circuncentro con X49.
    ( Mostrar/Ocultar figura )
      estrofoideJerabek.png

    •  La estrofoide de Jerabek es la imagen de la hipérbola de Jerabek en la inversión respecto a la circunferencia circunscrita.

    •  La coordenadas baricéntricas del foco de la estrofoide de Jerabek son:
    focoEstrofoideJerabek.png
    con número de búsqueda en ETC: -0.286262733091678965975303639

    •  La ecuación baricéntrica de su asíntota de la estrofoide de Jerabek es:
    (b^2-c^2)(a^4SA^4- 2a^2SBSCSA^3-((b^2-c^2)^4+2SBSC(SB^2+SC^2))SA^2- 2a^2SBSC((b^2-c^2)^2-SBSC)SA-(b^2-c^2)^2SB^2SC^2)x/(a^2SA) + ... =0.

    Cuyo punto del infinito:
    (a^2(a^6(b^2+c^2)-3a^4(b^4+c^4)+ a^2(3b^6-b^4c^2-b^2c^4+3c^6)-(b^4-c^4)^2): ... : ... ),
    con número de búsqueda en ETC: 1.022945264181160305475445435.

    La asíntota vuelve a cortar a la estrofoide de Jerabek en el punto S con primera coordenada baricéntrica:
    (a^2SA(a^2(SA^2-SBSC)-(b^2-c^2)^2SA) / (a^4SA^4- 2a^2SBSCSA^3-((b^2-c^2)^4+2SBSC(SB^2+SC^2))SA^2- 2a^2SBSC((b^2-c^2)^2-SBSC)SA-(b^2- c^2)^2SB^2SC^2), con número de búsqueda en ETC: 7.9946323139539709651682014

    •  La estrofoide de Jerabek es el lugar geométrico de los pies de las perpendiculares, trazadas desde el circuncentro a las tangentes a la parábola de directriz el eje d0 de la estrofoide y de foco F', la reflexión del circuncentro en el inverso (respecto a la circunferencia circunscrita) F del antipodal X265 del circuncentro en la hipérbola de Jerabek.

    Esta parábola pasa por X647 (centro de perspectividad del triángulo ABC y el triángulo tangencial de la hipérbola de Jerabek) y dirección del eje dada por el punto del infinito X924.
    Las coordenadas de su foco son:
    F' = (a^2(a^2-b^2-c^2) (a^8-a^6(b^2+c^2) + a^4b^2c^2 - a^2(b^2-c^2)^2(b^2+c^2) + (b^2-c^2)^2(b^4-b^2c^2+c^4)) : ... :...).
    con número de búsqueda en ETC: -7.354888360379703466465294356
    ( Mostrar/Ocultar figura )
      estrofoideJerabekParabola.png


  • viernes, 02 de mayo del 2014

    Lugares geométricos y triángulos circuncevianos

    ( Anopolis #1416, Antreas P. Hatzipolakis)

    Sean ABC un triángulo, P un punto y A'B'C' su triángulo circunceviano.
    Denotamos:
    Ab el centro de la circunferencia pasando por C' y tangente a AB en B,
    Ac el centro de la circunferencia pasando por B' y tangente a AC en C.
    Similarmente se definen los puntos Bc,Ba y Ca,Cb.
    Sean además Ma, Mb, Mc los puntos medios de los segmentos AbAc, BcBa, CaCb.

    •  El lugar geométrico de los puntos P tales que el triángulo delimitado por las rectas AbAc, BcBa, CaCb es nodegenerado y perspectivo con ABC es la cúbica de Neuberg, K001 = pK(X6, X30),

    ( Mostrar/Ocultar figura )
      Anopolis1416a2.png


    •  El lugar geométrico de los puntos P tales que las rectas AbAc, BcBa, CaCb son concurrentes es una séxtica con puntos dobles en los vértices de ABC, pasa por los centros X2, X3 y X1139 y por los puntos, Oa, Ob, Oc, donde las cevianas del circuncentro vuelven a corta a al cúbica de Neuberg.

    ( Mostrar/Ocultar figura )
      Anopolis1416a2.png
    Cuando P=X1139 (conjugado isogonal del punto de reflexión de Parry, X399), las rectas AbAc, BcBa, CaCb son paralelas, con punto del infinito X523, isogonal conjugado del foco de la parábola de Kiepert.
    Otros puntos comunes con la cúbica de Neuberg son Oa, Ob, Oc:
    Oc=(a^2SA:b^2SB:2SASB), ...


    •  El lugar geométrico de los puntos P tales que los triángulos ABC y MaMbMc son ortológicos es es la cúbica de Thomson, K002 = pK(X6, X2),

    Si P recorre la cúbica de Thomson, el centro ortológico Q de MaMbMc con respecto a ABC recorre la cúbica imagen de la cúbica de Darboux mediente la homotecia de centro el circuncentro y razón 1/2.
    ( Mostrar/Ocultar figura )
      Anopolis1416b1.png
    Algunos centros ortológicos R de ABC con respecto a MaMbMc de puntos P en la cúbica de Thomson: {X1, X80}, {X3, X1294}, {X4, X265}, {X6, X671}, {X57, X3254}
    ( Mostrar/Ocultar figura )
      Anopolis1416b2.png


    •  El lugar geométrico de los puntos P tales que las mediatrices de los segmentos AbAc, BcBa, CaCb son concurrentes es una nónica con puntos cuadruples en los vértices de ABC, pasa por los centros X1, X2, X3, X13, X14, X1113, X1114, y X1138 y por los pies de las cevianas del circuncentro.

    ( Mostrar/Ocultar figura )
      Anopolis1416c.png


  • martes, 22 de abril del 2014

    Punto de reflexión de Parry y otros asociados

    ( Anopolis #1382, Antreas P. Hatzipolakis)

    Sean ABC un triángulo, da, db, dc tres rectas paralelas a través de A, B, C, respectivamente, y d'a, d'b, d'c las reflexiones de da, db, dc en BC, CA, AB, respectivamente.

    Si las recta da, db, dc son paralelas a la recta de Euler de ABC, las rectas d'a, d'b, d'c concurren en X399 (punto de reflexión de Parry)

    ( a^2(a^8 - 4a^6(b^2+c^2) + a^4(6b^4+b^2c^2+6c^4) + a^2(-4b^6+b^4c^2+b^2c^4-4c^6) + (b^2-c^2)^2(b^4+4b^2c^2+c^4) : ... : ...)


    Cuando las rectas da, db, dc giran alrededor de A, B, C, permaneciendo paralelas, la recta de Euler de A'B'C' pasa por un punto fijo Q, cuyas coordenadas baricéntricas han sido calculadas por J. F. García Capitán, Hyacinthos message 15827 (November 19, 2007).

    El eje radical de las circunferencias circunscrita y de Euler de A'B'C' pasa por un punto fijo Y, cuando las rectas da, db, dc giran alrededor de A, B, C, permaneciendo paralelas.

    Y = (a^10(b^2+c^2)
    -a^8(3b^4+4b^2c^2+3c^4)
    +a^6(2b^6+5b^4c^2+5b^2c^4+2c^6)
    +2a^4(b^8-3b^6c^2+b^4c^4-3b^2c^6+c^8)
    -a^2(b^2-c^2)^2(3b^6-4b^4c^2-4b^2c^4+3c^6)
    +(b^2-c^2)^6: ... : ...)

    con número de búsqueda en ETC: -12.444306554956166449865828
    ( Mostrar/Ocultar figura )
      Anopolis1372.png
    Los puntos Y y Q son antipodales en la circunferencia que describe el punto de intersección la recta de Euler de A'B'C' y el eje radical de las circunferencias circunscrita y de Euler de A'B'C'. Esta circunferencia pasa por X399.

  • jueves, 17 de abril del 2014

    Perpendiculares a cevianas y lugares geométricos

    ( Anopolis #1354, Antreas P. Hatzipolakis)

    •  Sean ABC un triángulo, I su incentro y P un punto. La perpendicular por P a AI corta a AB y AC en Ab y Ac, respectivamente. Se denota por r1 el eje radical de las circunferencias circunscritas a los triángulos ABC y AAbAc.
    Los ejes radicales r2 y r3 se definen de forma cíclica.

    El lugar geométrico de los puntos P tales que el triángulo ABC es perspectivo con el triángulo A'B'C', delimitado por los ejes radicales r1, r2 y r3, está formado por la hipérbola equilátera que pasa por el incentro (hipérbola de Feuerbach) y por la cúbica circunscrita a ABC de ecuación baricéntrica:
    x (c (a - b) (a - b + c) y^2 + b (a - c) (a + b - c) z^2) +
    y (a (b - c) (a + b - c) z^2 + c (b - a) (-a + b + c) x^2) +
    z (b (c - a) (-a + b + c) x^2 + a (c - b) (a - b + c) y^2 ) - 2 a b c x y z = 0.


    Cuando P se mueve sobre la hipérbola de Feuerbach el centro de perspectividad Q de los triángulos ABC y A'B'C' recorre la cúbica generalizada de Lemoine K(X1)=K360.
    ( Mostrar/Ocultar figura )
      Anopolis1352Feuerbach.png




    •  Sean ABC un triángulo y P un punto. La perpendicular por P a AP corta a AB y AC en Ab y Ac, respectivamente. Se denota por r1 el eje radical de las circunferencias circunscritas a los triángulos ABC y AAbAc.
    Los ejes radicales r2 y r3 se definen de forma cíclica.

    El lugar geométrico de los puntos P tales que el triángulo ABC es perspectivo con el triángulo A'B'C', delimitado por los ejes radicales r1, r2 y r3, está formado por la circunferencia circunscrita a ABC y por la cúbica de Darboux K004



    Cuando P se mueve sobre la circunferencia circunscrita los ejes radicales r1, r2 y r3 son concurrentes en un punto Q, que describe la isocúbica no pivotal nK(X(577),X(394),X(3))=cK(#X(3),X(394)) (conico-pivotal isocubic, Jean-Pierre Ehrmann and Bernard Gibert.- Special Isocubics in the Triangle Plane, §8), que tiene al circuncentro como punto aislado y pasa por X878 (que corresponde a P=X98, punto de Tarry).
    La "cónica de contacto" de cK(#X(3),X(394)) es la cónica circunscrita de Johnson.
    (Otros ejemplos de "conico-pivotal isocubics" )

    ( Mostrar/Ocultar figura )
      Anopolis1352POR.png


  • miércoles, 16 de abril del 2014

    Triángulos con misma área que el triángulo órtico

    ( TriangulosCabri #707, Ricardo Barroso)

    Dado un triángulo ABC, el lugar geométrico de los puntos P para los cuales su triángulo pedal tiene la misma área que el triángulo órtico es la circunferencia de centro el circuncentro y que pasa por el ortocentro.

    Las coordenadas baricentricas de los vértices del triángulo pedal de un punto P=(x:y:z) son:
    D = (0 : (b^2-c^2)x+a^2(x+2y) : (-b^2+c^2)x+a^2(x+2z)),
    E = ((a^2-c^2)y+b^2(2x+y) : 0 :-a^2y+c^2y+b^2(y+2 z)),
    F = {(a^2-b^2)z+c^2(2x+z) : (-a^2+b^2)z+c^2(2y+z) : 0).
    El área de DEF coincide con el área de triángulo órtico (pedal del ortocentro) si y sólo si las coordenadas de P satisfacen a la ecuación:
    a^6 x^2 - a^4 b^2 x^2 - a^2 b^4 x^2 + b^6 x^2 - a^4 c^2 x^2 + 2 a^2 b^2 c^2 x^2 - b^4 c^2 x^2 - a^2 c^4 x^2 - b^2 c^4 x^2 + c^6 x^2 + 2 a^6 x y - 2 a^4 b^2 x y - 2 a^2 b^4 x y + 2 b^6 x y - 3 a^4 c^2 x y + 6 a^2 b^2 c^2 x y - 3 b^4 c^2 x y + c^6 x y + a^6 y^2 - a^4 b^2 y^2 - a^2 b^4 y^2 + b^6 y^2 - a^4 c^2 y^2 + 2 a^2 b^2 c^2 y^2 - b^4 c^2 y^2 - a^2 c^4 y^2 - b^2 c^4 y^2 + c^6 y^2 + 2 a^6 x z - 3 a^4 b^2 x z + b^6 x z - 2 a^4 c^2 x z + 6 a^2 b^2 c^2 x z - 2 a^2 c^4 x z - 3 b^2 c^4 x z + 2 c^6 x z + a^6 y z - 3 a^2 b^4 y z + 2 b^6 y z + 6 a^2 b^2 c^2 y z - 2 b^4 c^2 y z - 3 a^2 c^4 y z - 2 b^2 c^4 y z + 2 c^6 y z + a^6 z^2 - a^4 b^2 z^2 - a^2 b^4 z^2 + b^6 z^2 - a^4 c^2 z^2 + 2 a^2 b^2 c^2 z^2 - b^4 c^2 z^2 - a^2 c^4 z^2 - b^2 c^4 z^2 + c^6 z^2 = 0.

    Esta ecuación se puede poner en la forma:
    a^2yz+b^2zx+c^2xy-(-a^2+b^2+c^2)(a^2-b^2+c^2)(a^2+b^2-c^2)/((a+b+c)(-a+b+c)(a-b+c)(a+b-c)) (x+y+z)^2=0.
    Se trata de la circunferencia de centro en el circuncentro y que pasa por el ortocentro.
    ( Mostrar/Ocultar figura )
      RB707.png



  • martes, 8 de abril del 2014

    Una propiedad del cuadrado baricéntrico del simediano

    Sean ABC un triángulo y L un punto variable sobre la paralela a BC por A. El lugar geométrico de los ortocentros de los triángulos LBC es una parábola; denotamos por ta la tangente en su vértice. Similarmente y de forma cíclica se consideran la tangentes tb y tc.

    El cento de homotecia de los triángulos ABC y el delimitado por ta, tb y tc es el X32.

    ( Mostrar/Ocultar figura )
      HG080414.png

    Sea L=(t:1:-1) un punto variable sobre la recta paralela a BC por A. Las coordenadas baricéntricas del ortocentro del triángulo LBC son:
    HL = ( (a^2(t-2)+(c^2-b^2)t)((b^2-c^2)t+a^2(2+t)) : -(a^2(t-1)-c^2(t-1)-b^2(1+t))((b^2-c^2)t+a^2(2+t)) : -(a^2(t-2)+(c^2-b^2)t)(-c^2(t-1)+a^2(1+t)-b^2(1+t)) ).

    Este punto describe la parábola ℘a de ecuación:
    a: -a^2SAx^2 + 4a^4yz+(3a^4+b^4-4a^2c^2-2b^2c^2+c^4)zx + (3a^4-4a^2b^2+b^4-2b^2c^2+c^4)xy = 0.

    Cuya tangente ta en su vértice tiene por ecuación:
    (2a^4-2a^2b^2+b^4-2a^2c^2-2b^2c^2+c^4)x + a^4y + a^4z = 0.
    Permutando cíclicamente, se obtienen las tangentes tb y tc en los vértices de las correpondientes parábolas ℘b y ℘c. El punto de interseccion de estas últimas es:
    A' = (-a^4+2a^2(b^2+c^2)-2(b^4-b^2c^2+c^4) : b^4 : c^4).
    Y silarmente resultan las coordenadas de B'=tc∩ta y C' = ta∩tb. En consecuencia, las rectas AA', BB' y CC' concurren en el punto X32 = (a^4:b^4:c^4).

    Adicionalmente tenemos:

    •  El centro de homotecia de los triángulos ABC y el delimitado por las directrices de las tres parábolas ℘a, ℘b y ℘c tiene coordenadas baricentricas:

    (a^4+S^2 : b^4+S^2 : c^4+S^2)

    con número de búsqueda en ETC: 0.64658408249618255603004536878
    ( Mostrar/Ocultar figura )
      HG080414d.png
    La directriz de la parábola ℘a es (a^4-3S^2)x+(a^4+S^2)y+(a^4+S^2)z=0.

    •  El centro de homotecia de los triángulos ABC y el delimitado por las perpendiculares por los focos a los ejes de las tres parábolas ℘a, ℘b y ℘c tiene coordenadas baricentricas:

    (a^4-S^2 : b^4-S^2 : c^4-S^2)

    con número de búsqueda en ETC: -0.2507364947244949174662
    ( Mostrar/Ocultar figura )
      HG080414f.png
    La perpendicular al eje de la parábola ℘a por su foco es (a^4-5S^2)x+(a^4-S^2)y+(a^4-S^2)z=0.

    •  El ortocentro X4 de ABC es común a las tres parábolas. La tangente en X4 a la parábola ℘a corta a los lados AB y AC en Ac y Ab, respectivamente. Similarmente se determina los puntos Ba, Bc, Cb y Ca.
    Estos seis puntos quedan en una misma cónica, cuyo centro tiene coordenadas baricéntricas (notación de Conway):

    (SASBSC + 2(b^2SB^2+c^2 SC^2) : SASBSC + 2(c^2SC^2+a^2SA^2 ) : SASBSC + 2(a^2SA^2+b^2SB^2)),

    con número de búsqueda en ETC: 1.8894135371622742397888827245

    Los puntos Aa, Bb y Cc de intersección de las tangentes en el ortocentro a las parábolas ℘a, ℘b y ℘c con los lados BC, CA u AB, respectivamente, están en la recta SA^2x+SB^2y+SC^2z=0, polar trilineal de X393.
    ( Mostrar/Ocultar figura )
      HG080414a.png


  • lunes, 7 de abril del 2014

    Conjugado isogonal respecto al triángulo antipedal

    ( Anopolis #1309, Antreas P. Hatzipolakis)

    Sean ABC un triángulo, P un punto y A'B'C' el triángulo antipedal de P. El lugar geométrico de los puntos P tales que P, el conjugado isogonal de P respecto a ABC y el conjugado isogonal de P respecto a A'B'C' están alineados es cúbica de McKay, K003 = pK(X6, X3).

    ( Mostrar/Ocultar figura )
      Anopolis1309.png

    Si P(u:v:w), su conjugado isogonal es P*(a^2vw:b^2wu:c^2uv) (en coordenadas baricéntricas).
    El conjugado isogonal Q de P respecto a A'B'C' es la intersección de las rectas AD', BE', CF', siendo D', E' , F' las reflexiones de P respecto a los puntos medios D, F, F de los lados de ABC (ver §4.2Cojugado isogonal).
    Q=(SASC(v+w) + SB(SA(v+w)-SC u) : ... : ... ).


    Si P está sobre la cúbica de McCay, su conjugado isogonal Q respecto a A'B'C' es la reflexión de P respecto al circuncentro.

  • miércoles, 2 de abril del 2014

    El centro de congruencia de Yff

    Sean ABC un triángulo, P un punto, A1 y A2 los centros de semejanza interno y externo de las circunferencias con centro en B y C que pasan por P. Similarmente, y de forma cíclica, se definen los puntos B1, B2, C1, A2. Entonces, las rectas AA1, BB2 y CC1 concurren en un punto Q, cuya polar trilineal pasa por A2, B2 y C2.

    ( Mostrar/Ocultar figura )
      HG020414.png

    Esta construcción de los puntos A1 y A2 conincide con la generalización dada por Peter Moses en ETC, ya que PA1 es la bisectriz del ángulo ∠BPC:
    "Let ABC be a triangle and P a point. Let D be the point on side BC such that ∠BPD = ∠DPC), and likewise for point E on side CA and point F on side AB. If P = (p : q : r) (trilinears), then the lines AD, BE, CF concur in the point K(P) = f(p,q,r,A) : f(q,r,p,B) : f(r,p,q,C), where f(p,q,r,A) = (q^2 + r^2 + 2qr cos A)-1/2. Moreover, if P* is the inverse of P in the circumcircle, then K(P*) = K(P). [Peter Moses, Feb. 1, 2010, based on Seiichi Kirikami's construction of X(174)]"

    Si P=(u:v:w), en coordenadas baricéntricas, el punto de intersección de las rectas AA1, BB2 y CC1 es:
    Yff.png

    Al punto P', inverso de P en la circunferencia circunscrita, le corresponde el mismo punto Q que a P.

    Si P=X1 es el incentro, Q=X174 es centro de congruencia de Yff:
    Iff
    ( Mostrar/Ocultar figura )
      HG020414X1.png

    Parejas de puntos {P,Q}:
    {X1,X174}, {X3,X2}, {X13,X13}, {X15,X1}, {X16,X1}, {X36,X174}.

    Cuando P es el baricentro, Q=(1/Sqrt[-a^2+2b^2+2c^2]:1/Sqrt[2a^2-b^2+2c^2]:1/Sqrt[2a^2+2b^2-c^2]), con número de búsqueda en ETC: 1.62327927239083782493133901

  • martes, 1 de abril del 2014

    Triángulos ortológicos asociados al baricentro

    ( Anopolis #1286, Antreas P. Hatzipolakis)

    Sean ABC un triángulo y A'B'C' el triángulo medial.
    Adoptamos las siguientes notaciones:

    Ab y Ac los circuncentros de los triángulos AGB' y AGC', resp.
    Bc y Ba los circuncentros de los triángulos BGC' y BGA', resp.
    Ca y Cb los circuncentros de los triángulos CGA' y CGB', resp.
    Ma, Mb y Mc los puntos medios de AbAc, BcBa, CaCb, resp.


    En coordenadas baricéntricas:
    Ma
    Mb
    Mc

    Los triángulos ABC y MaMbMc son ortológicos.
    El centro de ortología de MaMbMc con respecto a ABC es el centro de la circunferencia de los nueve puntos, X5.
    El centro de ortología de ABC con respecto a MaMbMc es:

    Ortologico1.png
    ( Mostrar/Ocultar figura )
      Anopolis1283.png


    Sean M1, M2 y M3 los simétricos de Ma, Mb y Mc respecto a las medianas AA', BB' y CC'. Los triángulos ABC y M1M2M3 son ortológicos


    En coordenadas baricéntricas:
    M1
    M2
    M3

    El centro de ortología de M1M2M3 y ABC es X2845:
    (a^2SA +10SBSC: b^2SB + SCSA : c^2SC + SASB).

    El centro de ortología de ABC y M1M2M3 es:
    Ortologico2.png
    ( Mostrar/Ocultar figura )
      Anopolis1283.png


  • lunes, 31 de marzo del 2014

    Ejemplos de triángulos perspectivos y ortológicos

    ( Anopolis #1273, Antreas P. Hatzipolakis)

    Sean ABC un triángulo, P un punto y A'B'C' el triángulo pedal de P.
    Adoptamos las siguientes notaciones:

    Ab y Ac son las proyecciones ortogonales de A' sobre PB' y PC', respectivamente.
    A3+ es la intersección de la paralela por B' a AC con la paralela por Ac a A'B'.
    A3- es la intersección de la paralela por B' a AC con la paralela por A' a B'Ac.
    Es decir, la circunferencia de centro en B' y radio A'Ac interseca a la paralela por B' a A'Ac en los dos puntos A3+ y A3-.

    A2+ es la intersección de la paralela por C' a AB con la paralela por Ab a A'C'.
    A2- es la intersección de la paralela por C' a A'B con la paralela por A' a C'Ab.

    M1+ y M1- son los puntos medios de A2+A3+ y A2-A3-, respectivamente.

    ( Mostrar/Ocultar figura )
      Anopolis1273b.png

    Similarmente y de forma cíclica, se definen los puntos:
    B3+, B1+, C1+, C2+, B3-, B1-, C1-, C2- y M2+, M2-, M3+, M3-.

    1. El lugar geométrico de los puntos P tales que las mediatrices de los segmentos A2+A3+, B3+B1+, C1+C2+ son concurrentes es una quártica circunscrita a ABC.

    ( Mostrar/Ocultar figura )

    Información sobre esta cuártica:
      Pasa por el circuncentro y ortocentro. Tres de sus asíntotas son perpendiculares a los lados de ABC y delimitan un triángulo perspectivo con ABC, con centro de perspectividad el foco de la parábola de Kiepert. Las ecuaciones baricéntricas de estas asíntotas (notación de Conway) son:
    (b^2-c^2)SBSBx + a^2SBy + a^2SCz =0   y cíclicamente.
      La cuarta asíntota tiene como punto del infinito a X523, conjugado isogonal del foco de la parábola de Kiepert, X110.

    NOTA: Si P es el circuncentro o está en el infinito, la configuración no tiene sentido. Cuando P es el ortocentro, las mediatrices de los segmentos A2+A3+, B3+B1+, C1+C2+ concurren en el punto de primera coordenada baricéntrica:

    a^2(a^6(b^2+c^2) - 3a^4(b^4+c^4) + a^2(3b^6+b^4c^2+b^2c^4+3c^6) - (b^2-c^2)^2(b^4+4b^2c^2+c^4)),

    con número de búsqueda en ETC: -1.91350459625470323717507806


    2. El lugar geométrico de los puntos P tales que las mediatrices de los segmentos A2-A3-, B3-B1-, C1-C2- son concurrentes es una quártica circunscrita a ABC.

    ( Mostrar/Ocultar figura )

      Cuando P es el ortocentro, las mediatrices de los segmentos A2-A3-, B3-B1-, C1-C2- concurren en el centro de la circunferencia de Taylor, X389. Además, en este caso el triángulo M1-M2-M3- es perspectivo con ABC (centro de perspectividad el semidiano) y su circunferencia circunscrita es concéntrica con la circunferencia de Taylor.
    ( Mostrar/Ocultar figura )



    3. El lugar geométrico de los puntos P tales que los triángulos ABC y M1+M2+M3+ son perspectivos es una cúbica circunscrita a ABC.
    Estos triángulos son ortológicos si solo si P está sobre la recta de Euler.

    ( Mostrar/Ocultar figura )


    Los centros de ortología y de perspectividad de los triángulos ABC y M1+M2+M3+ correspondientes a los puntos X4 y X361 son colineales:
    ( Sondat's Theorem. If two triangles are mutually orthologic and perspective, the centres of orthology and perspectivity are collinear. Further, it is perpendicular to the perspectrix.
    An Application of Sondat's Theorem, Ion Patrascu and Florentin Smarandache
    ):

    •  P=X4
    ( Mostrar/Ocultar figura )
      El centro de perspectividad de los triángulos ABC y M1+M2+M3+ es el simediano, X6.
      El centro de ortología de ABC respecto a M1+M2+M3+ coordenadas baricéntricas:

    O31 = ( SA/(a^4 - 2a^2(b^2+c^2) + b^4+6b^2c^2+c^4) : ... : ...),

    con número de búsqueda en ETC: 10.2893367499920063626210307
      El centro de ortología de M1+M2+M3+ respecto a ABC coordenadas baricéntricas:

    O32 = ( 2a^10 - a^8(b^2+c^2) - 8a^6(b^4-b^2c^2+c^4) + 10a^4(b^2-c^2)^2(b^2+c^2) - 2a^2(b^4-c^4)^2 - (b^2-c^2)^4(b^2+c^2): ... : ...),

    con número de búsqueda en ETC: 7.3799576190510080677803629

    •  P=X631
    ( Mostrar/Ocultar figura )
    En este caso los triángulos ABC y M1+M2+M3+ son homotéticos.
      El centro de homotecia de los triángulos ABC y M1+M2+M3+ es el punto de coordenadas baricéntricas:

    Q = ( a^2(a^4 - 2a^2(b^2+c^2) + b^4+6b^2c^2+c^4 ) : ... : ...),

    con número de búsqueda en ETC: 3.08200205691896287503357243
      El centro de ortología de ABC respecto a M1+M2+M3+ es el ortocentro.
      El centro de ortología de M1+M2+M3+ respecto a ABC es el punto de coordenadas baricéntricas:

    Y = ( 2a^10 - 5a^8(b^2+c^2) + 4a^6(b^2-c^2)^2 - 2a^4(b^6+7b^4c^2+7b^2c^4+c^6) + 2a^2(b^2-c^2)^2(b^4+14b^2c^2+c^4) - (b^2-c^2)^4(b^2+c^2): ... : ...),

    con número de búsqueda en ETC:-1.5600580790622942195564665


    4. El lugar geométrico de los puntos P tales que los triángulos ABC y M1-M2-M3- son perspectivos es una cúbica circunscrita a ABC.
    Estos triángulos son ortológicos si solo si P está sobre la recta de Euler.

    ( Mostrar/Ocultar figura )
    Esta cúbica pasa por el ortocentro, donde es tangente a la recta de Euler, por X1498 y el tercer punto E común con la recta de Euler tiene por coordenadas:

    E = ( a^2(a^8 - 2a^6(b^2+c^2) - 12a^4b^2c^2+ 2a^2(b^2+c^2)^3 - (b^2-c^2)^2(b^4-6b^2c^2+c^4)): ... : ...),



    Casos en que los triángulos ABC y M1-M2-M3- son simultáneamente perspectivos y ortológicos:

    •  P=X4
    ( Mostrar/Ocultar figura )
    En este caso los triángulos ABC y M1-M2-M3- son homotéticos.
      El centro de homotecia de los triángulos ABC y M1-M2-M3- es el simediano X6.
      El centro de ortología de ABC respecto a M1-M2-M3- es el ortocentro.
      El centro de ortología de M1-M2-M3- respecto a ABC es el punto de coordenadas baricéntricas:

    O42 = ( 2a^10 - 5a^8(b^2+c^2) + 4a^6(b^2+c^2)^2 - 2a^4(b^2-c^2)^2(b^2+c^2) + 2a^2(b^2-c^2)^4- (b^2-c^2)^4(b^2+c^2): ... : ...),

    con número de búsqueda en ETC:-1.34146146000198321561337218

    •  P=E
    ( Mostrar/Ocultar figura )

      El centro de perspectividad de los triángulos ABC y M1-M2-M3- es el punto de coordenadas baricéntricas:

    Z = ( a^2SA /(b^2+c^2-3a^2) : b^2SB /(c^2+a^2-3b^2) : c^2SC /(a^2+b^2-3c^2) ),

    con número de búsqueda en ETC: -2.28208807333641855014440139
      El centro de ortología de ABC respecto a M1-M2-M3-es el punto de coordenadas baricéntricas:

    U = ( a^2/(5a^10 - 11a^8(b^2+c^2) + 2a^6(b^4+22b^2c^2+c^4) + 2a^4(5b^6-13b^4c^2-13b^2c^4+5c^6) - a^2(b^2-c^2)^2(7b^4+18b^2c^2+7c^4)+ (b^2-c^2)^4(b^2 + c^2)) : ... : ...),

    con número de búsqueda en ETC:-16.11015588490167262757688
      El centro de ortología de M1-M2-M3- respecto a ABC es el punto de coordenadas baricéntricas:

    V = ( a^2(a^12(b^2+c^2) - 2a^10(2b^4+7b^2c^2+2c^4) + 5a^8(b^6+5b^4c^2+5b^2c^4+c^6) - 12a^6b^2c^2(b^4+6b^2c^2+c^4) + a^4(-5b^10+11b^8c^2+26b^6c^4+26b^4c^6+11b^2c^8-5c^10) + 2a^2(b^2-c^2)^4(2b^4-3b^2c^2+2c^4) - (b^2-c^2)^4(b^6-7b^4c^2-7b^2c^4+c^6)) : ... : ... ),

    con número de búsqueda en ETC: -12.933974089689188258369107


    5. El lugar geométrico de los puntos P tales que los triángulos M1+M2+M3+ y M1-M2-M3- son perspectivos es la recta de Euler. Estos triángulos son ortológicos para todo punto P del plano.

    ( Mostrar/Ocultar figura )

    Los centros de ortología O1, O2 están alineados con Q, por el Teorema de Sondat.

    Cuando P recorre la recta de Euler, el centro de perspectividad Q de los triángulos M1+M2+M3+ y M1-M2-M3- recorre la recta X2X6, que pasa por el baricentro y semidiano. La envolvente de las rectas PQ es una parábola (t_P_PCp) tangente a la recta de Euler en X376 (baricentro del triángulo antipedal de X2) y a la recta X2X6 en X597 (punto medios del baricentro y simediano).
    el punto del infinito de esta parábola es:

    ( 10a^6 - a^4(b^2+c^2) - 4a^2(b^4+c^4) - 5(b^2-c^2)^2(b^2+c^2): ... : ... ),

    con número de búsqueda en ETC: 1.0956343251291829068665975668

  • jueves, 27 de marzo del 2014

    Recta de Euler del triángulo antipedal conteniendo al circuncentro

    ( Anopolis #1268, Antreas P. Hatzipolakis)

    Sean ABC un triángulo, P un punto y A'B'C' el triángulo antipedal de P. El lugar geométrico de P tal que la recta de Euler de su triángulo antipedal pasa por el circuncentro es una séptica que pasa por: X(1), X(4), X(6), X(13), X(14), X(74), los excentros, por los pies de las alturas.

    ( Mostrar/Ocultar figura )


  • miércoles, 26 de marzo del 2014

    Cuártica lugar de centros de homotecia

    ( Anopolis #1265, Antreas P. Hatzipolakis)

    Sean ABC un triángulo, P un punto y A'B'C' el triángulo pedal de P.
    Se denota por Ab, Ac las proyecciones ortogonales de A' sobre PB',PC', respectivamente.
    l1 es el eje radical de las circunferencias Ab(AbB') y Ac(AcC'), y m1 es el eje radical de las circunferencias Ab(AbC') y Ac(AcB'). l1 y m1 son paralelos, ya que son perpendiculares a AbAc.
    Similarmente, se consideran los ejes radicales l2, l3, m2 y m3.
    Consideremos el triángulo LaLbLc delimitado por l1, l2 y L3 y el triángulo MaMbMc delimitado por m1, m2 y m3, ellos son homotéticos.

    El lugar geométrico de los puntos P que coinciden con el centro de homotecia de los triángulos LaLbLc y MaMbMc es la cuártica:

    ( Mostrar/Ocultar figura )

    Esta cuártica pasa por el baricentro (X2), por los puntos isodinámicos (X15, X16), por los puntos del infinito de la hipérbola de Kiepert (X3412, X3413) y por los pies Va, Vb, Vc y Wa, Wb, Wc de las bisectrices interiores y exteriores.

  • sábado, 22 de marzo del 2014

    Una caracterización de la cúbica de McCay

    ( Anopolis #1248, Antreas P. Hatzipolakis)

    Sean ABC un triángulo, P un punto y PaPbPc el triángulo antipedal de P.
    Se denota por Ha, Hb y Hc los ortocentros de los triángulos PBC, PCA y PAB, respectivamente.
    Y por H1, H2 y H3 los ortocentros de los triángulos PaBC, PbCA y PcAB, respectivamente.

    El lugar geométrico de los puntos P tales que los triángulos HaHbHc y H1H2H3 sean perspectivos es la cúbica de McKay,

    ( Mostrar/Ocultar figura )

    En coordenadas baricéntricas, si P=(u:v:w):
    Ha = ((SBv+SC(u+v))(SCw+SB(u+w)) : (SBv+SC(u+v))(SAu-SCw) : (SAu-SBv)(SCw+SB(u + w))),
    H1 = (-u:u+w:u+v).
    Las coordenadas de los ortocentros Hb, Hc, H2, H3 se obtienen por permutación cíclica.

    Cuando el punto P se mueve sobre la cúbica de McKay el centro de perspectividad Q de los triángulos HaHbHc y H1H2H3 está sobre la tercera cúbica de Musselman (K028).

  • viernes, 21 de marzo del 2014

    Puntos asociados a los triángulos duales de los triángulos cevianos de los puntos de Fermat

    ( DC(X(13)) y DC(X(14)), Encyclopedia of Triangle Centers)

    En ETC, justo antes del centro X2979, se da el concepto de triángulo dual:
      Sea UVW un triángulo en plano del triángulo ABC. U' el conjugado isogonal del punto del infinito de la recta VW; se definen V' y W' cíclicamente. Al triángulo U'V'W' se le denomina dual de UVW.
    Y se define una aplicación entre puntos del plano (en coordenadas baricéntricas) por:

    P=(u:v:w)   →   DC(P) = ( a^2vw/(v+w) : b^2wu/(w+u) : c^2uv/(u+ v) )
    DC(P) es el punto tal que su triángulo circunceviano es el triángulo dual del triángulo ceviano de P.

    En particular:

    DC(X13) = X2981, conjugado isogonal de X396 = punto medio de X13 y X15.
    DC(X14) (no incluido actualmente en ETC), conjugado isogonal de X395 = punto medio de X14 y X16.



    Otra interpretación geométrica de los centros DC(X13) y DC(X14):

     Dado un triángulo ABC, sea DDbDc el triángulo equilátero INSCRITO en ABC, con D, Db y Dc sobre BC, CA y AB, respectivamente, y DbDc paralelo a BC. Cíclicamente se define los triángulos equiláteros inscritos EEcEa y FFaFb.
     Las rectas DbDc, EcEa y FaFb delimitan un triángulo A'B'C' homotético a ABC y el centro de homotecia es DC(X13)= X2981:

    DC(X(13))
    ( Mostrar/Ocultar figura )

    El triángulo DEF es el ceviano de X13. Las coordenadas de los puntos Db y Dc son, respectivamente:
    (a^2(3SC+√3 S) : 0 : 2(a^2SA+SC(SB+√3 S))),   (a^2(3SB+√3 S) : 2(a^2SA+SB(SC+√3 S)) : 0).

    Los centros Oa, Ob, Oc de los triángulos equiláteros inscritos DDbDc, EEbEc, FFbFc, respectivamente, están alineados y las rectas AOa, BOb, COc concurren en X17.

    Los seis puntos Db, Dc, Ec, Ea, Fa y Fb están en una misma cónica.



     Dado un triángulo ABC, sea DDbDc el triángulo equilátero EXINSCRITO en ABC, con D, Db y Dc sobre BC, CA y AB, respectivamente, y DbDc paralelo a BC. Cíclicamente se define los triángulos equiláteros exinscritos EEcEa y FFaFb.
     Las rectas DbDc, EcEa y FaFb delimitan un triángulo A'B'C' homotético a ABC y el centro de homotecia es DC(X14):

    DC(X(14))
    ( Mostrar/Ocultar figura )

    El triángulo DEF es el ceviano de X14. Las coordenadas de los puntos Db y Dc son, respectivamente:
    (a^2(3SC-√3 S) : 0 : 2(a^2SA+SC(SB-√3 S))),   (a^2(3SB-√3 S) : 2a^2SA+2SB(SC-√3 S), 0) : 0).

    Los centros Oa, Ob, Oc son los centros de los triángulos equiláteros exinscritos DDbDc, EEbEc, FFbFc, respectivamente, están alineados y las rectas AOa, BOb, COc concurren en X18.

    Los seis puntos Db, Dc, Ec, Ea, Fa y Fb están en una misma cónica.

  • lunes, 17 de marzo del 2014

    Una interpretación de las circunferencias de Tucker

    ( ADGEOM #1165, Emmanuel José García)

    Dados un triángulo ABC y un número real t, las paralelas al lado BC a la distancia |ta| cortan a AB en Cb y C'b (hacia el exterior e interior del triángulo, respectivamente), y al lado AC en Bc y B'c (hacia el exterior e interior del triángulo, respectivamente).
    Procediendo cíclicamente, tomando las distancias tb y tc, se definen los puntos Ca, Ac, Ab y Ba, y los puntos C'a, A'c, A'b y B'a.
    Los puntos Cb, Bc, Ca, Ac, Ab y Ba están en una misma circunferencia Γt; así mismo, los puntos C'b, B'c, C'a, A'c, A'b y B'a están en una misma circunferencia Γ‍′t. Estas son circunferencias de Tucker, cambiando t por (-t), se pasa de una circunferencia a otra.

    Para obtener las coordenadas baricéntricas de los puntos anteriores, tomamos la circunferencia de centro en el punto medio de BC y radio ta:
    c^2xy+b^2xz+a^2yz +(1/4) (x+y+z) ((a^2-2b^2-2c^2+4a^2t^2)x + (-a^2+4a^2t^2)y + (-a^2+4a^2t^2)z) = 0.
    La ecuación conjunta de las tangentes a esta circunferencia paralelas a BC es:
    (a^4-2a^2b^2+b^4-2a^2c^2-2b^2c^2+c^4+4a^4t^2)x^2 + 4a^4t^2y^2 + 4a^4t^2z^2 + 8a^4t^2yz + 8a^4t^2zx + 8a^4t^2xy = 0.
    Y sus intersecciones con los lados AC y AB son:
    Ba=(2at : 0 : -bc-2at),   Ca=(2at : -bc-2at : 0),   B'a=(2at : 0 : bc-2at),   C'a=(2at : bc-2at : 0).
    Por permutación cíclica obtenemos las coordenadas del resto de los puntos.
    ( Mostrar/Ocultar figura )
    La ecuación baricéntrica de la circunferencia Γt es:
    Tucker Circle
    Se trata de una circunferencia de Tucker; en efecto, la antiparalela por Ac a BC, respecto a AB y AC es c(bc+2at)x+2actya(ab+2ctz=0, que corta a AC en Ca= (2at : -bc-2at : 0 ), y sucesivamente.

    La ecuación de la circunferencia Γ‍′t, resulta de la anterior, cambiando t por -t.
    El eje radical de las circunferencias Γt y Γ‍′t, para todo t, es la polar del simediano respecto a la circunferencia circunscrita, que corta perpendicularmente al eje de Brocard en X187, punto medio de los puntos isodinámicos.

    Casos particulares de circunferencia de Tucker:

    Circunferencia de Apolonio.   [t = (a+b+c)/4=s/2]

    Tucker Circle
    La circunferencia que toca las tres circunferencias exinscritas de un triángulo y los abarca a menudo se conoce como circunferencia de Apolonio (Clark Kimberling,- "Triangle Centers and Central Triangles.". 1998, p. 102).
    Darij Grinberg; Paul Yiu.- "The Apollonius Circle as a Tucker Circle." Forum Geom. 2(2002), 175-182.

    Circunferencia circunscrita   [t=0]

    a²yz + b²zx + c²xy = 0.


    Primera circunferencia de Lemoine  [t= -(a b c)/(2(a^2+b^2+c^2))]

    Tucker Circle
    Las paralelas a los lados de un triángulo por su simediano, cortan a sus lados es seis puntos cocíclicos situados en la denominada primera circunferencia de Lemoine.

    Circunferencia coseno o Segunda circunferencia de Lemoine [t=-(a b c)/(a^2 + b^2 + c^2)]

    Tucker Circle
    Cada antiparalela por el simediano a un lado, respecto a los otros dos, corta a estos en dos puntos. Los seis puntos así determinados son cocíclicos estando en la que se conoce como circunferencia coseno.

    Circunferencia de Gallatly  [t=-abc(a^2+b^2+c^2)/(4(a^2b^2+b^2c^2+c^2a^2))]

    Tucker Circle
    Se llama circunferencia de Gallatly de un triángulo ABC a la circunferencia pedal de sus puntos de Brocard.

    Circunferencia de Kenmotu  [t=(abc Sqrt[(-a+b+c)(a+b- c)(a-b+c)(a+b+c)] - abc(a^2+b^2+c^2)/(2(a^4+b^4+c^4))]
    Si en un triángulo ABC se inscriben tres cuadrados iguales con un vértice común, las diagonales de los cuadrados, que une vértices que están en los lados de ABC, son antiparalelas a los lados de ABC. Por eso estos seis vértices pertenecen a una circunferencia de Tucker, circunferencia de Kenmotu.

    Circunferencia de Taylor  [t=-S^2/(2abc)]

    Tucker Circle
    La circunferencia de Taylor de un triángulo es la que pasa por las proyecciones de los pies de las alturas sobre los lados.

    Circunferencia de Tucker biceviana con centro en AH  [t=-bc/(2a)]

    Tucker Circle
    Una de las seis circunferencias de Tucker que se obtiene cuando el centro está sobre la altura desde el vértice A, pasa por este vértice.
    Bernard Gibert.- "Bicevian Tucker Circles" Forum Geom. 7 (2007) 87–97.

    Circunferencia de Tucker biceviana con centro en la mediatriz de los puntos medios de AB y AC  [t=-bc/(4a)]

    Tucker Circle
    Una de las seis circunferencias de Tucker que se obtiene cuando el centro está sobre la mediatriz del lado del triángulo medial con vértices en los puntos medios de AB y AC, pasa por estos puntos medios.

  • jueves, 13 de marzo del 2014

    Triple reflexión en los lados de un triángulo

    ( Anopolis #1197, Seiichi Kirikami)

    Dados un triángulo ABC y un punto P, se denotan las reflexiones de P en BC, CA y AB por P1, P2 y P3, respectivamente.
    Sean P12 y P13 las reflexiones de P1 en CA y AB, respectivamente. Se definen P23, P21 y P31, P32 cíclicamente.
    Finalmente, se denotan las reflexiones de P21 y P31 en CA y AB por P212 P213 y P312, P313 respectivamente. Y se definen cíclicamente los puntos P323, P321, P123, P121 y P131, P132, P231, P232.

    Los puntos P1, P21, P31, P231 y P321 están en una misma circunferencia Γa, cuyo centro Oa no depende de la posición del punto P y tiene coordenadas baricéntricas (-a^2:2SC:2SB).

    De forma similar se consideran las circunferencias Γb y Γc y sus respectivos centros Ob y Oc.
    ( Mostrar/Ocultar figura )


    Si D = P321P123∩P132P231 ∈ Γa, E = P132P231∩P213P312 ∈ Γb y F = P213P312∩P321P123 ∈ Γc, los triángulos DEF son todos iguales, para cualquier P, y se tiene que:
    Area(DEF) = |SASBSC|/(a^2b^2c^2) Area(ABC)

    Los triángulos ABC y DEF son perspectivos si y solo si P queda en la hipérbola
    ( a^4 b^2 x^2-2 a^2 b^4 x^2+b^6 x^2-a^4 c^2 x^2+b^4 c^2 x^2+2 a^2 c^4 x^2-b^2 c^4 x^2-c^6 x^2-a^6 x y+3 a^4 b^2 x y-3 a^2 b^4 x y+b^6 x y-a^4 c^2 x y+b^4 c^2 x y+2 a^2 c^4 x y-2 b^2 c^4 x y-a^6 y^2+2 a^4 b^2 y^2-a^2 b^4 y^2-a^4 c^2 y^2+b^4 c^2 y^2+a^2 c^4 y^2-2 b^2 c^4 y^2+c^6 y^2+a^6 x z+a^4 b^2 x z-2 a^2 b^4 x z-3 a^4 c^2 x z+2 b^4 c^2 x z+3 a^2 c^4 x z-b^2 c^4 x z-c^6 x z+2 a^4 b^2 y z-a^2 b^4 y z-b^6 y z-2 a^4 c^2 y z+3 b^4 c^2 y z+a^2 c^4 y z-3 b^2 c^4 y z+c^6 y z+a^6 z^2+a^4 b^2 z^2-a^2 b^4 z^2-b^6 z^2-2 a^4 c^2 z^2+2 b^4 c^2 z^2+a^2 c^4 z^2-b^2 c^4 z^2 = 0)
    (H) equilátera que pasa por el ortocentro, por el simediano, por la reflexión del circuncentro en el ortocentro (X382), por el punto de reflexión de Parry (X399) y de puntos del infinito X2574 y X2575, conjugados isogonales de los puntos en los que la recta de Euler corta a la circunferencia circunscrita.

    ( Mostrar/Ocultar figura )

    El centro de perspectividad de los triángulos ABC y DEF, cuando P recorre (H), queda sobre la hipérbola de Jerabek (hipérbola equilátera circunscrita a ABC que pasa por el circuncentro).
    El centro de la hipérbola (H) es el punto de coordenadas baricéntricas:

    (a^2 (a^8(b^2+c^2) - 2a^6(b^4+c^4) + a^4b^2c^2(b^2+c^2) + a^2(b^2-c^2)^2(2b^4+b^2c^2+2c^4) - b^10 + b^8c^2 + b^2c^8 - c^10) : ... : ...),

    que tiene (6-9-13)-número de búsqueda en ETC: -6.618666655241975536408988331.

    Peter Moses obtiene que este punto está en las rectas determinadas por los pares de centros del triángulo con número de orden: {6,1205}, {23,110}, {51,125}, {52,3627}, {74,389}, {113,5562}, {185,1986}, {265,5446}, {1204,2935}, {1614,2914}, {3060,3448}. Y que se puede expresar como:
    3 X(51) - 2 X(125),   3 X(51) - 4 X(1112),   3 X(3060) - X(3448).


    Los triángulos OaObOc y DEF son perspectivos si y solo si P queda en la hipérbola de Jerabek. El centro de perspectividad queda sobre la hipérbola equilátera (H).

    ( Mostrar/Ocultar figura )


    Las tangentes en D, E y F a las circunferencias Γa, Γa y Γc son concurrentes si y solo si P recorre la cónica circunscrita a ABC con centro X5 (centro de la circunferencia de los nueve puntos).

    ( Mostrar/Ocultar figura )


    Las tangentes en D, E y F a las circunferencias Γa, Γa y Γc delimitan un triángulo no degenerado perspectivo con DEF si y solo si P recorre la cúbica de Darboux del triángulo órtico, K044 = pK(X216,X4).

    ( Mostrar/Ocultar figura )


    Las tangentes en D, E y F a las circunferencias Γa, Γa y Γc delimitan un triángulo no degenerado perspectivo con ABC si y solo si P recorre una cuártica circunscrita a ABC, que pasa por el ortocentro (doble), por X265 (reflexión del circuncentro en el centro de la hipérbola de Jerabek) y por X1986 (punto de reflexión de Hatzipolakis) y sus dos asíntotas (ortogonales) tienen la dirección de los puntos X2574 y X2575 (conjugados isogonales de los puntos de intersección de la recta de Euler con la circunferencia circunscrita).

    ( Mostrar/Ocultar figura )


    Las tangentes en D, E y F a las circunferencias Γa, Γa y Γc delimitan un triángulo no degenerado perspectivo con OaObOc si y solo si P recorre una cuártica circunscrita a ABC, que pasa por el circuncentro, ortocentro y X54 (punto de Kosnita) y sus dos asíntotas (ortogonales) tienen la dirección de los puntos X2574 y X2575 (conjugados isogonales de los puntos de intersección de la recta de Euler con la circunferencia circunscrita).

    ( Mostrar/Ocultar figura )


  • miércoles, 12 de marzo del 2014

    Triángulos ceviano y pedal de misma área

    ( Problema 700, Ricardo Barroso)

    Lugar geométrico de los puntos del plano del triángulo ABC tales que sus triángulos ceviano y pedal tiene la misma área

    Si P=(u:v:w) son las coordenadas baricéntricas de un punto del plano, las áreas de sus triángulos ceviano PaPbPc y pedal DEF son, respectivamente:
    2uvw/((u+v)(u+w)(v+w)) Area(ABC),
    (-a+b+c)(a+b-c)(a-b+c)(a+b+c)(c^2uv+b^2uw + a^2vw)/(4a^2b^2c^2(u+v+w)^2) Area(ABC)
    Estas dos cantidades son iguales si P está en la quíntica, con puntos dobles en los vértices de ABC (pudiendo ser aislados) y asíntotas los lados del triángulo antimedial, de ecuación:
    a^4 c^2 x^3 y^2 - 2 a^2 b^2 c^2 x^3 y^2 + b^4 c^2 x^3 y^2 - 2 a^2 c^4 x^3 y^2 - 2 b^2 c^4 x^3 y^2 + c^6 x^3 y^2 + a^4 c^2 x^2 y^3 - 2 a^2 b^2 c^2 x^2 y^3 + b^4 c^2 x^2 y^3 - 2 a^2 c^4 x^2 y^3 - 2 b^2 c^4 x^2 y^3 + c^6 x^2 y^3 + a^4 b^2 x^3 y z - 2 a^2 b^4 x^3 y z + b^6 x^3 y z + a^4 c^2 x^3 y z + 4 a^2 b^2 c^2 x^3 y z - b^4 c^2 x^3 y z - 2 a^2 c^4 x^3 y z - b^2 c^4 x^3 y z + c^6 x^3 y z + a^6 x^2 y^2 z - a^4 b^2 x^2 y^2 z - a^2 b^4 x^2 y^2 z + b^6 x^2 y^2 z + 8 a^2 b^2 c^2 x^2 y^2 z - 3 a^2 c^4 x^2 y^2 z - 3 b^2 c^4 x^2 y^2 z + 2 c^6 x^2 y^2 z + a^6 x y^3 z - 2 a^4 b^2 x y^3 z + a^2 b^4 x y^3 z - a^4 c^2 x y^3 z + 4 a^2 b^2 c^2 x y^3 z + b^4 c^2 x y^3 z - a^2 c^4 x y^3 z - 2 b^2 c^4 x y^3 z + c^6 x y^3 z + a^4 b^2 x^3 z^2 - 2 a^2 b^4 x^3 z^2 + b^6 x^3 z^2 - 2 a^2 b^2 c^2 x^3 z^2 - 2 b^4 c^2 x^3 z^2 + b^2 c^4 x^3 z^2 + a^6 x^2 y z^2 - 3 a^2 b^4 x^2 y z^2 + 2 b^6 x^2 y z^2 - a^4 c^2 x^2 y z^2 + 8 a^2 b^2 c^2 x^2 y z^2 - 3 b^4 c^2 x^2 y z^2 - a^2 c^4 x^2 y z^2 + c^6 x^2 y z^2 + 2 a^6 x y^2 z^2 - 3 a^4 b^2 x y^2 z^2 + b^6 x y^2 z^2 - 3 a^4 c^2 x y^2 z^2 + 8 a^2 b^2 c^2 x y^2 z^2 - b^4 c^2 x y^2 z^2 - b^2 c^4 x y^2 z^2 + c^6 x y^2 z^2 + a^6 y^3 z^2 - 2 a^4 b^2 y^3 z^2 + a^2 b^4 y^3 z^2 - 2 a^4 c^2 y^3 z^2 - 2 a^2 b^2 c^2 y^3 z^2 + a^2 c^4 y^3 z^2 + a^4 b^2 x^2 z^3 - 2 a^2 b^4 x^2 z^3 + b^6 x^2 z^3 - 2 a^2 b^2 c^2 x^2 z^3 - 2 b^4 c^2 x^2 z^3 + b^2 c^4 x^2 z^3 + a^6 x y z^3 - a^4 b^2 x y z^3 - a^2 b^4 x y z^3 + b^6 x y z^3 - 2 a^4 c^2 x y z^3 + 4 a^2 b^2 c^2 x y z^3 - 2 b^4 c^2 x y z^3 + a^2 c^4 x y z^3 + b^2 c^4 x y z^3 + a^6 y^2 z^3 - 2 a^4 b^2 y^2 z^3 + a^2 b^4 y^2 z^3 - 2 a^4 c^2 y^2 z^3 - 2 a^2 b^2 c^2 y^2 z^3 + a^2 c^4 y^2 z^3 = 0.
    Esta curva pasa por el ortocentro H, ya que sus triángulos medial y pedal coinciden.
    La tangente en el ortocentro (punto ordinario de la curva) tiene la dirección determinada por el punto del infinito X924 (conjugado isogonal de X925).
    Si el triángulo es acutángulo, el ortocentro es interior al triángulo ABC. Así existen infinitos puntos, por continuidad, en el interior de un triángulo acutángulo tales que las áreas de sus triángulos ceviano y pedal coinciden.
    ( Mostrar/Ocultar figura )


  • martes, 25 de febrero del 2014

    Centros de ortología de triángulos cevianos y antipedales

    ( r2040, Quim Castellsaguer)

    Dados un triángulo ABC y un punto Q, el lugar geométrico de los puntos P tales que los triángulos antipedal de P y ceviano de Q son ortológicos es la cónica circunscrita que pasa por Q y por su complemento.

    ( Mostrar/Ocultar figura )
    El centro de ortología X del triángulo ceviano QaQbQc de Q, respecto al triángulo antipedal DEF de P, describe la cónica circunscrita a QaQbQc, que pasa por Q y por el "crosspoint" C(X2,Q) de X2 y Q.

    Las ecuaciones de las cónicas citadas son, si (p:q:r) son las coordenadas baricéntricas de Q:
    p(q^2-r^2)yz + q(r^2-p^2)zx + r(p^2-q^2) = 0,
    qr(q-r)x^2 + rp(r-p)y^2 + pq(p-q)z^2 + p^2(q-r)yz + q^2(r-p)zx + r^2(p-q)xy = 0.
    Estas cónicas son homotéticas (¿centros de homotecia?), tienen en común los puntos Q y C(X2,Q) , y se verifica que P, X y C(X2,Q) están alineados.
    El "crosspoint" C(X2,Q) de X2 y Q, tiene esta otra interpretación geométrica: "The lines tangent to the conic {A,B,C,P,U} at P and U intersect in X. (Randy Hutson, September 10, 2012)".

    Un caso particular:
    Cuando Q es el incentro (resultado r2040 en Todo Triángulo Web de Joaquín Castellsaguer), la cónica que pasa por A, B, C, Q, cQ tiene de ecuación baricéntrica:
    a(b2-c2)yz + b(c2-a2)zx +c(a2-b2)xy = 0,

    que contiene a los centros del triángulo de números de orden: 1, 10, 19, 37, 65, 75, 82, 91, 158, 225, 267, 596, 759, 775, 876, 897, 921, 969, 994, 1247, 1581, 1910, 2153, 2154, 2166, 2168, 2186, 2190, 2214, 2216, 2217, 2218, 2219, 2363, 2588, 2589, 2652, 2962, 3668, 4674.


    Dados un triángulo ABC y un punto P, el lugar geométrico de los puntos Q tales que los triángulos antipedal de P y ceviano de Q son ortológicos es la isocúbica pivotal pK(G,P), de polo el baricentro y pivote el conjugado isotómico de P.

    Si P=(u:v:w),   pK(G,P) : vwx(y^2-z^2) + wuy(z^2-x^2) + uvz(x^2-y^2 = 0.
    ( Mostrar/Ocultar figura )
    El centro de ortología Y del triángulo ceviano QaQbQc de Q, respecto al triángulo antipedal DEF de P, describe (cuando Q recorre pK(G,P)) la isocúbica pivotal de polo el "crosspoint" C(X2,Q) de X2 y Q, y pivote el anticomplemento aP de P.

    Un caso particular:

    Dados un triángulo ABC, el lugar geométrico de los puntos Q tales que los triángulos antipedal (excentral) del incentro y ceviano de Q son ortológicos es la isocúbica pivotal K034 = pK(X2,X75), de polo el baricentro y pivote el conjugado isotómico del incentro.

    ( Mostrar/Ocultar figura )

    El centro de ortología Y, del triángulo ceviano QaQbQc de Q respecto al triángulo excentral IaIbIc, describe (cuando Q recurre pK(X2,X75)) la isocúbica pivotal K033 = pK(X37,X8).

  • jueves, 20 de febrero del 2014

    El punto X275 como centro de homotecia

    Dado un triángulo ABC, sean A' el punto diametralmente opuesto a A, D el punto de intersección de AA' con BC y A" la reflexión de A' en D.
    Los puntos Ba y Ca son las proyecciones ortogonales de A" sobre AC y AB, respectivamente. Se definen similarmente los puntos Cb, Ab, Ac y Bc.

    El triángulo UVW, delimitado por las rectas BaCa, CbAb y AcBc, es homotético a ABC y su centro de homotecia es X275.

    X275 = (1/(SA (SB SC + S^2)) : 1/(SB (SC SA + S^2)) : 1/(SC (SA SB + S^2))).


    El centro de homotecia de UVW y A'B'C' (que ha de estar en la recta X3X275) es el punto de coordenadas baricéntricas:

    Y = (SB SC (3 a^4 SA^4 - SA^3 (SB^3 - 3 a^2 SB SC + SC^3) - SA^2 SB SC (7 SB^2 + 16 SB SC + 7 SC^2) - 8 a^2 SA SB^2 SC^2 - 2 SB^3 SC^3):...:...),

    con número de búsqueda en ETC: 25.2481990527062715026144
    ( Mostrar/Ocultar figura )


    Además, los seis puntos Ba, Ca, Cb, Ab, Ac y Bc están en una misma cónica de ecuación baricéntrica:


  • miércoles, 19 de febrero del 2014

    Dos triángulos de misma área

    Dado un triángulo ABC, los triángulos A1B1C1 y A2B2C2 tales que

    A1B1/B1A = b/c,   B1C1/C1B = c/a,   C1A1/A1C = a/b,  
    A2B2/B2A = c/b,   B2C2/C2B = a/c,   C2A2/A2C = b/a,  
    tienen la misma área igual a:
    abc/(ab(a+b)+bc(b+c)+ca(c+a)+abc)  área(ABC)

    ( Mostrar/Ocultar figura )



  • miércoles, 12 de febrero del 2014

    Construcción del triángulo: O Ma Hb

    ( Construcción de triángulos #176) 4th Irish Mathematical Olympiad 4 May 1991, Paper 1

    Construir un triángulo conociendo su circuncentro y los pies de la mediana y la altura desde dos vértices distintos.

    ( Mostrar/Ocultar figura )

    Utilizando un sistema de coordenadas respecto al que O(0,0) y Ma(0,-m), sean Hb(α,β) y un punto variable B'(t,-m) en la perpendicular por Ma a OMa.
    La perpendicular por Hb a HbB' corta a la circunferencia de centro en O y radio OB' en los puntos A' y B', que describen un lugar geométrico que corta a la perpendicular por Ma a OMa en los vértices B y C del triángulo ABC a construir. Las coordenadas de estos puntos son (±(α2+(m+β)2)½,-m). Así, pueden ser construidos con regla y compás. Una vez construidos B y C, la construcción del vértice A es inmediata. Existen DOS SOLUCIONES.

  • jueves, 30 de enero del 2014

    Perspectividad con los triángulos medial y órtico

    (ADGEOM #1085 Emmanuel Garcia)

      Dado un triángulo ABC, con longitud de lados a=BC, b=CA y c=AB, sean |at|, |bt| y |ct| las distancias de los lados de ABC a los lados del triángulo A'B'C', tal que at=bt=ct (t número real). El circuncentro y el ortocentro son los únicos puntos del plano tales que sus triángulo pedales son perspectivos con A'B'C', para todo número real t.

    Relativo al triángulo ABC, sean I el incentro, r el radio de la circunferencia inscrita y LMN el triángulo de contacto interior. La recta pa situada a una distancia |at| del lado BC es la paralela a dicho lado por el punto que resulta de aplicar una homotecia de centro en I y razón (r+at)/r al punto L. Esta recta pa tiene ecuación baricéntrica (similarmente, las rectas pb y pc):

    pa:  (a+b+c)r+a^2t)x + a^2ty + a^2tz = 0
    pa:  b^2tx + (a+b+c)r+b^2t)y + b^2tz = 0
    pa:  c^2tx + c^2ty + (a+b+c)r+c^2t)y = 0.

    Los vértices del triángulo A'B'C' delimitado por las rectas pa, pb y pc son:

    A' = ((a+b+c)r + (b^2+c^2)t : -b^2t : -c^2t),
    B' = (-a^2 t : (a+b+c)r + (c^2+a^2)t : -c^2t},
    C' = (-a^2t : -b^2 t : (a+b+c)r + (a^2+b^2)t).

      Nótese que el centro de homotecia de ABC y A'B'C' es el simediano (a^2:b^2:c^2). La razón de homotecia es:
    1 + (a^2+b^2+c^2)t / ((a+b+c)r).
    lo que nos da otra vía para obtener los triángulos A'B'C'


    Dado un punto P(u:v:w) los vértices de su triángulo pedal son:

    D = (0 : (b^2-c^2)u + a^2(u+2v) : (c^2-b^2)u + a^2(u+2 w)),
    E = ((a^2-c^2)v + b^2(v+2u) : 0 : (c^2-a^2)v + b^2(v+2w)),
    F = ((a^2-b^2)w + c^2(w+2u), (b^2-a^2)w + c^2(w+2v) : 0).

    Imponiendo que los triángulos A'B'C' y DEF sean perspectivos se llega a que las coordenadas de P deben satisfacer a un polinomio de tercer grado en la variable t. La anulación de los coeficientes de este polinomio nos permite afirmar que P ha de estar en la cúbica de Darboux (para que el término independiente se anule), sobre la recta de Euler (para que el coeficiente del término de segundo grado se anule).
    Los único puntos comunes de la recta de Euler y la cúbica de Darboux son el circuncentro y el ortocentro.
    Como los puntos P que anulan al coeficiente del término de primer grado del polinomio están en una cónica que pasa por el circuncentro y el ortocentro, queda establecida la afirmación inicial. Es decir:

      Los únicos triángulos pedales que son perspectivos a todos los triángulos homotéticos a ABC, mediante las homotecias de centro en el simediano, son los triángulos medial y órtico.



    • El lugar geométrico de los centros de perspectividad del triángulo órtico y cada triángulo imagen de ABC en una homotecia de centro en el simediano, es la hipérbola equilátera circunscrita al triángulo órtico y que pasa por el ortocentro.
    Esta hipérbola, con centro en X(1112), pasa por los puntos X(4), X(6), X(52), X(113), X(155), X(185), X(193), X(1162), X(1163), X(1829), X(1839), X(1843), X(1858), X(1986), X(2574), X(2575), X(2904), X(2905), X(2906), X(2907), X(2914), X(3574), X(5095).
    ( Mostrar/Ocultar figura )


    • El lugar geométrico de los centros de perspectividad del triángulo medial y cada triángulo imagen de ABC en una homotecia de centro en el simediano, es la recta que pasa por el baricentro y simediano.
    ( Mostrar/Ocultar figura )


  • miércoles, 29 de enero del 2014

    El centro X(468)

      Dado un triángulo ABC y un punto variable P sobre la recta de Euler, el lugar geométrico de los centros de la hipérbola equilátera circunscrita al triángulo pedal de P y que pasa por P es una recta, que corta a la recta de Euler en X468.

    ( Mostrar/Ocultar figura )
    La ecuación baricéntrica de la recta lugar geométrico de los centros de las hipérbolas que se tratan es:

    (b^2-c^2)(-a^10 + 2a^8(b^2+c^2) - 5a^6b^2c^2 + a^4(-2b^6+3b^4c^2+3b^2c^4-2c^6) + a^2(b^2-c^2)^2(b^4-b^2c^2+c^4) + 3b^2c^2(b^2-c^2)^2(b^2 + c^2))x+ ... = 0.

    Esta recta pasa por los centros X(468) (sobre la recta de Euler), por X(1112) y cuyo punto del infinito es X(511) (conjugado isogonal del punto de Tarry, cuarto punto de intersección de la circunferencia circunscrita y la hipérbola de Kiepert).

    X(1112) es el centro de la hipérbola cuando P es el ortocentro.
    X(468) es el centro de la hipérbola cuando P es X(186), el inverso del ortocentro en la circunferencia circunscrita



  • martes, 28 de enero del 2014

    El centro X(2165)

      Dado un triángulo ABC, sean TaTbTc su triángulo tangencial y DEF el triángulo circunceviano del ortocentro. La tangente en D a la circunferencia circunscrita corta a TaTb en Db y a TaTc en Dc.
      Los puntos Tb, Tc, Db y Dc están en una circunferencia Γa y denotamos por da el eje radical de esta circunferencia y la circunferencia circunscrita.
      Similarmente se definen los ejes radicales db y dc.

    ABC y el triángulo A'B'C', delimitado por da, db y dc, son perspectivos, con centro de perspectividad en X2165.

    ( Mostrar/Ocultar figura )


  • jueves, 23 de enero del 2014

    La isocúbica pK(X2, X6)

      Dado un triángulo ABC y un punto P, sean Pr y Pl sus brocardianos (Paul Yiu.- Introduction to the Geometry of the Triangle, 8.4 pag, 106), PcbPacPba y PbcPcaPab los triángulos cevianos de Pr y Pl, respectivamente.

    La isocúbica pivotal pK(X2, X6) es el lugar geométrico de los puntos P tales que las mediatrices de PcbPbc, PacPca y PbaPab son concurrentes.

    Esta caracterización geométrica de pK(X2, X6) le ha motivado a Bernard Gibert a asignarle el número K659 es su catálogo de cúbicas asociadas a un triángulo.
    ( Mostrar/Ocultar figura )
    Esta cúbica, de ecuación baricéntrica:

    a^2 x (y^2 - z^2) + b^2 y (z^2 - x^2) + c^2 z (x^2 - y^2) = 0,

    pasa por los vértices de ABC y de su triángulo antimedial y por los centros X(2), X(6), X(76), X(194), X(2998).

    NOTAS sobre cónicas bicevianas de los brocardianos de un punto.

    Construcción de los brocardianos:
    Si PaPbPc es el triángulo ceviano de P, el punto Pcb es la intersección de BC con la paralela a AC por Pc y el punto Pbc es la intersección de BC con la paralela a AB por Pb. Los puntos Pac, Pca, Pba y Pab se definen similarmente.

    •   La cónica biceviana de los brocardianos de un punto P es una circunferencia solo cuando P= X194.

    Este hecho geométrico está descrito en el párrafo 2.3 de Bernard Gibert.- Tucker cubics and Bicentric Cubics.
    Si (u:v:w) son las coordenadas baricéntricas de P, la ecuación de la cónica biceviana de los brocardianos de P es:

    vwx^2 + wuy^2 + uvz^2 -(u^2 + vw)yz -(v^2 + wu)zx -(w^2 + uv)xy = 0.

    Una cónica de ecuación baricéntrica f x^2 + g y^2 + h z^2 + 2 p y z + 2 q z x + 2 r x y = 0 es una circunferencia cuando

    (g + h -2p)/a^2 = (h + f -2q)/b^2 = (f + g -2r)/c^2.

    Así, la cónica biceviana que nos ocupa es circunferencia cuando:

    (u + v)(u + w)/a^2 = (v + w)(v + u)/b^2 = (w + u)(w + v)/c^2.

    Resolviendo estas ecuaciones obtenemos que las coordenadas (u:v:u) son proporcionales a (a^2(b^2+c^2)-b^2c^2 : b^2(c^2+a^2)-c^2a^2 : c^2(a^2+b^2)-a^2b^2), que son las el centro X194.
    La circunferencia biceviana de los brocardianos de X194 tiene centro con coordenadas:

    (a^2 (a^4 (b^4-4 b^2 c^2+c^4)-a^2 (b^6-4 b^4 c^2-4 b^2 c^4+c^6)+b^2 c^2 (b^4-4 b^2 c^2+c^4)) : ... : ...),

    que tiene (6-9-13)-número de búsqueda en ETC: 0.1830166211743119241902625514

    •   La cónica biceviana de los brocardianos de un punto P tienen su centro en P si y sólo si P está en la elipse circunscrita de Steiner.
    ( Mostrar/Ocultar figura )

    Si P está sobre la elipse circunscrita de Steiner sus brocardianos están en la recta del infinito.

    •   Los puntos A' = BPac∩CPab, B' = CPba∩APbc, C' = APca∩BPca están alineados si y solo si P está sobre la elipse circunscrita de Steiner
    ( Mostrar/Ocultar figura )

    Cuando P está sobre la elipse circunscrita de Steiner, la recta p que contiene a los puntos A', B', C' también pasa por el baricentro. Si tP es la tangente en P a la elipse y Q = p∩tP, el área de ABC es cuatro veces el área del triángulo ceviano de Q. Por tanto, Q describe la cúbica T(1/4) = K015 (ver párrafo 3.1 de Bernard Gibert.- Tucker cubics and Bicentric Cubics.)

  • lunes, 20 de enero del 2014

    Conjeturas

      Dado un triángulo ABC y un punto P, existen DOS triángulos isósceles con baricentro A, el ángulo desigual de amplitud la del ángulo en el vértice A, y los vértices iguales sobre las cevianas BP y CP.
      Sean A1B1C1 y A2B2C2 tales triángulos, con ∠B1A1C1 = ∠B2A2C2 = ∠BAC, B1 y B2 sobre BP, C1 y C2 sobre CP, y A el baricentro de A1B1C1 y de A2B2C2.
      Los triángulos isósceles A1B1C1 y A2B2C2 son inversamente semejantes. Si A'1 es la reflexión de A1 en B1C1, los triángulos A'1B1C1 y A2B2C2 son directamente semejantes; denotamos por A' su centro de semejanza directa. Similarmente se consideran los centros de semejanza B' y C'.

    1)   Para cualquier P, las circunferencias (BB'∩CC',B,C), (CC'∩AA',C,A) y (AA'∩BB',A,B) son concurrentes en un punto Q.

    ( Mostrar/Ocultar figura )

    2)   Si P recorre la circunferencia circunscrita a ABC, las circunferencias (BB'∩CC',B,C), (CC'∩AA',C,A) y (AA'∩BB',A,B) concurren en un punto fijo Q0, con (6-9-13)-número de búsqueda en ETC: 19.52051544033555

    ( Mostrar/Ocultar figura )

    3)   Existe un único punto P0 sobre la circunferencia circunscrita tal que AA', BB' y CC' son concurrentes (en el punto Q0).

    ( Mostrar/Ocultar figura )


  • jueves, 02 de enero del 2014

    Rectas perpendiculares por un punto que cortan a dos rectas dadas

    (ADGEOM #1014 Dao Thanh Oai)

    Sean un punto D y dos rectas p y q, entonces la recta que pasa por los puntos medios de los dos segmentos que pares de rectas perpendiculares por D determinan sobre p y q, pasa por un mismo punto al variar las rectas perpendiculares.

    ( Mostrar/Ocultar figura )

    El punto fijo X asociado a D, p, q es el punto de intersección de las mediatrices de los segmentos que las rectas perpendiculares por D a p y q, determinan sobre éstas.

    Damos dos aplicaciones de este resultado a la geometría del triángulo:

    1a) Dado un triángulo ABC y un punto P, los puntos medios de los segmentos que dos rectas perpendiculares variables que pasan por A determinan al cortar cada una a las rectas PB y PC, definen un recta que pasa por un punto fijo Pa.

    Pa es el punto de intersección de la mediatriz del segmento que la perpendicular por A a PB determina sobre las rectas PB y PC con la mediatriz del segmento que la perpendicular por A a PC determina sobre las rectas PB y PC.
    Se define de forma similar los puntos Pb y Pc, procediendo cíclicamente sobre los vértices de ABC.

    El lugar geométrico de los puntos P tales que ABC y PaPbPc son perspectivos es la hipérbola de Kiepert, y el centro de perspectividad Q es el punto donde la paralela por P a la recta de Euler vuelve a cortar a la hipérbola de Kiepert.

    ( Mostrar/Ocultar figura )
    Si P=(1/(SA+t) : 1/(SB+t) : 1/(SC+t)) es un punto genérico de la hipérbola de Kiepert, entonces el centro de perspectividad Q tiene coordenadas baricéntricas

    Q=(1/(S²+SAt) : 1/(S²+SBt) : 1/(S²+SCt)).

    Pares de centros {P,Q} sobre la hipérbola de Kiepert que figuran actualmente en ETC, expresados por sus correspondientes índices {i,j}, P=Xi, Q=Xj:
    {2, 4}, {4, 2}, {13, 13}, {14, 14}, {98, 671}, {262, 598}, {485, 1327}, {486, 1328}, {598, 262}, {671, 98}, {1327, 485}, {1328, 486}, {3424, 5485}, {3429, 4052}, {4052, 3429}, {5485, 3424}

    Algún par {P,Q}, con P y Q centros en la hipérbola de Kiepert, tales que Q no figura actualmente en la ETC:

    {X10, (1/(2 S^2-3(a^2+b^2+c^2+2(ab+ac+bc))SA):...:...)}, (6-9-13)-número de búsqueda en ETC: -7.75319920991533803152915369

    {X17, (1/(S+3Sqrt[3]SA):...:...)}, (6-9-13)-número de búsqueda en ETC: -1.860007742164222715329129330

    {X18, (1/(S-3Sqrt[3]SA):...:...)}, (6-9-13)-número de búsqueda en ETC: 126.1499312579877710386653603

    {X76, (1/(3(SA²-SBSC)+2S²):...:...)}, (6-9-13)-número de búsqueda en ETC: -16.87874108766941076512556987

    {X83, (1/(3(SA²-SBSC)+4S²):...:...)}, (6-9-13)-número de búsqueda en ETC: -2.887934280286545741662150280

    {X94, (1/((S² - 3 SA²) (S² + 3 SBSC)):...:...)}, (6-9-13)-número de búsqueda en ETC: 0.3900113547977998375532972499

    { (1/(3(SA²-SBSC)-2S²):...:...)} (1/(3 a^2 - 2 b^2 - 2 c^2):...:...), }, (6-9-13)-números de búsqueda del segundo en ETC: 4.63650295237392362290626430 y 4.1268565215227781921162084412, respectivamente,

    { (1/(3(SA²-SBSC)-4S²):...:...), (1/(a^2-6SA):...:...) }, el primero con (6-9-13)-números de búsqueda en ETC: 2.98578607000337615936536037 y el segundo es el conjugado isotómico de X3630.

    { (1/(a^2 + 6 SA):...:...), (1/(5 S^2 + 3 (SA+SB+SC)SA):...:...)}, el primero es el conjugados isotómico de X3631 y (6-9-13)-número de búsqueda del segundo en ETC: 0.0063415316860299433902901912.

    { (1/(5 S^2 - 3 (SA+SB+SC)SA):...:...), (1/(3 a^2 - 2 b^2 - 2 c^2)):...:...), }, (6-9-13)-números de búsqueda del segundo en ETC: 4.63650295237392362290626430 y 4.1268565215227781921162084412, respectivamente,

    2a) Dado un triángulo ABC y un punto P, los puntos medios de los segmentos que dos rectas perpendiculares variables que pasan por P determinan al cortar cada una a las rectas AB y AC, definen un recta que pasa por un punto fijo Pa.

    Pa es el punto de intersección de la mediatriz del segmento que la perpendicular por P a AB determina sobre las rectas AB y AC con la mediatriz del segmento que la perpendicular por P a AC determina sobre las rectas AB y AC.
    Se define de forma similar los puntos Pb y Pc, procediendo cíclicamente sobre los vértices de ABC.

    El lugar geométrico de los puntos P tales que ABC y PaPbPc son perspectivos es la cúbica de Darboux (K004), y el centro de perspectividad Q queda sobre la cúbica de Thomson (K002).

    ( Mostrar/Ocultar figura )


    OTRA FORMA DE ENUNCIAR ESTA PROPIEDAD GEOMÉTRICA DE LA CÚBICA DE DARBOUX:

    Sean ABC un triángulo, un punto P y DEF su triángulo pedal. La recta PE corta a AB en E' y PF corta a AC en F'. Los puntos E, F, E' y F' están en una circunferencia de centro Pa. Se definen similarmente los puntos Pb y Pc. El lugar geométrico de los puntos P tales que ABC y PaPbPc son perspectivos es la cúbica de Darboux (K004), y el centro de perspectividad Q queda sobre la cúbica de Thomson (K002).

    Si P=(p:q:r) es un punto de la cúbica de Darboux, entonces el centro de perspectividad Q, en la cúbica de Thomson, tiene coordenadas baricéntricas

    Q=(a^4(p^2-2(q-r)^2+p(q+r)) + 2a^2(b^2(p(3r-q)+(q+r)^2)+c^2(p(3q-r)+(q+r)^2)) - (b^2-c^2)^2p(p-q-r) : ... : ... ).

    Pares de centros {P,Q} sobre las cúbicas de Darboux y Thomson que figuran actualmente en ETC, expresados por sus correspondientes índices {i,j}, P=Xi, Q=Xj:
    {1, 1}, {3, 6}, {4, 2}, {20, 4}, {40, 57}, {64, 3}, {84, 9}, {1490, 282}, {1498, 1073}, {2130, 3349}, {2131, 3350}, {3182, 3342}, {3183, 3344}, {3345, 223}, {3346, 1249}, {3347, 3341}, {3348, 3343}, {3353, 3352}, {3354, 3351}, {3355, 3356}, {3472, X?}, {3473, X?}, {3637, X?}.
    Página Personal